Clinical Surgery Pearls 01

 













































































































































































01..............................152


CLINICAL SURGERY PEARLS


CLINICAL SURGERY PEARLS

R Dayananda Babu MS MNAMS

Professor and Head

Department of Surgery

Sree Gokulam Medical College and Research Foundation

Venjaramoodu, Thiruvananthapuram, Kerala, India

JAYPEE BROTHERS MEDICAL PUBLISHERS (P) LTD.

New Delhi • London • Philadelphia • Panama

®

SECOND EDITION

Foreword

Mathew Varghese

Headquarters

Jaypee Brothers Medical Publishers (P) Ltd

4838/24, Ansari Road, Daryaganj

New Delhi 110 002, India

Phone: +91-11-43574357

Fax: +91-11-43574314

Email: jaypee@jaypeebrothers.com

Jaypee Brothers Medical Publishers (P) Ltd

Overseas Offices

J.P. Medical Ltd

83, Victoria Street, London

SW1H 0HW (UK)

Phone: +44-2031708910

Fax: +02-03-0086180

Email: info@jpmedpub.com

Jaypee Brothers Medical Publishers (P) Ltd

17/1-B Babar Road, Block-B, Shaymali

Mohammadpur, Dhaka-1207

Bangladesh

Mobile: +08801912003485

Email: jaypeedhaka@gmail.com

Jaypee-Highlights Medical Publishers Inc.

City of Knowledge, Bld. 237, Clayton

Panama City, Panama

Phone: +507-301-0496

Fax: +507-301-0499

Email: cservice@jphmedical.com

Jaypee Brothers Medical Publishers (P) Ltd

Shorakhute, Kathmandu

Nepal

Phone: +00977-9841528578

Email: jaypee.nepal@gmail.com

Website: www.jaypeebrothers.com

Website: www.jaypeedigital.com

© 2013, Jaypee Brothers Medical Publishers

All rights reserved. No part of this book may be reproduced in any form or by any means without the prior permission of the publisher.

Inquiries for bulk sales may be solicited at: jaypee@jaypeebrothers.com

This book has been published in good faith that the contents provided by the author contained herein are original, and is intended for

educational purposes only. While every effort is made to ensure accuracy of information, the publisher and the author specifically disclaim

any damage, liability, or loss incurred, directly or indirectly, from the use or application of any of the contents of this work. If not specifically

stated, all figures and tables are courtesy of the author. Where appropriate, the readers should consult with a specialist or contact the

manufacturer of the drug or device.

Clinical Surgery Pearls

First Edition: 2010

Second Edition: 2013

ISBN 978-93-5090-396-4

Printed at

Jaypee Brothers Medical Publishers, Ltd

The Bourse

111 South Independence Mall East

Suite 835, Philadelphia, PA 19106, USA

Phone: + 267-519-9789

Email: joe.rusko@jaypeebrothers.com

®

Dedicated to

My late parents for their love and affection –

Mr Raghavan and Mrs Mallakshy

My only sister – late Ms Damayanthy

My wife – Professor (Dr) Geetha Bhai and

to my beloved son Deepak D Babu

for their moral support

My teachers for their wisdom

My patients for their trust and support

My students for their assistance


Professor R Dayananda Babu is known to me for the past forty years. I have great admiration for his wealth

of knowledge in the subject of surgery.

He has written the book Clinical Surgery Pearls with careful and persistent effort. The overriding goal has

been the mobilization of information relative to the science and skills of surgery. In addition to defining

the frontiers of surgical knowledge, it affords the student to assimilate the fundamentals in an easy way.

This book will be an enormous help to those who are studying surgery at both undergraduate and

postgraduate levels.

I wish the book a great success.

Professor (Dr) Mathew Varghese

MS FRCS Ed

Emeritus Professor of Surgery

Government Medical College

Kottayam, Kerala, India

Foreword


Preface to the Second Edition

The first edition of this book was published in 2010. It is gratifying to note the wide acceptance of this

book as an exam cracker by undergraduates and postgraduates alike; and, therefore, I was forced to bring

out the second edition within 2 years of the initial publication. I am happy to note that now this book is

recommended by many universities.

There is no need to stress the importance of refreshing a book like this. I was forced to spend many hours

in rectifying the errors which have crept up in the first edition. The old chapters have been thoroughly

revised and updated. The new American Joint Committee on Cancer (AJCC), 7th edition, has been used for

staging and management, instead of the 6th edition of AJCC as used in the first edition. At the end of some

of the important cases, colored boxes have been used under the title “What is new—For postgraduates,

the unique unorthodox style, the student-oriented approach and the question-answer format are still

retained.”

I am grateful to Professor John S Kurian, who is Professor of Surgery at Government Medical College,

Kottayam, Kerala, India, for the effort he has taken to find out the errors and for coming up with suggestions

for improvement. I also thank Dr Deepak George, for his valuable suggestions for improvement of many

of the chapters.

I also thank the publisher M/s Jaypee Brothers Medical Publishers (P) Ltd, New Delhi, India, for bringing

out a high-quality second edition book quickly.

R Dayananda Babu


Preface to the First Edition

This book is the final result of my continuous teaching and learning process with my undergraduate and

postgraduate students in surgery. Whenever I interact with my students, I realize their problems and

deficiencies and find out the solutions, so that it reaches them. Whenever I read a chapter, a series of

questions will come to my mind and then I will try to answer those questions. That is exactly the reason

why this book is in question-answer format. The flow charts and tables in this book are evolved in the

classrooms and bedside teaching area.

Whenever I read a topic, I try to define the condition. I feel that when you define something, half the

problem is solved; and, therefore, the first chapter is devoted to definitions. There are more than 100

definitions in this book.

Another important aspect of any learning process is to find out the concepts behind the disease process

and management. These concepts are converted to an easily digestible capsule form in this book for the

students. As an examiner at undergraduate and postgraduate levels, I realized that most of the time the

students miss many important clinical points during case presentation, not because they do not know

them but because they do not have a checklist. Therefore, I have given the checklist for all clinical cases.

The questions for the postgraduate (PG) students are marked as PG in brackets so that the undergraduate

students can skip them if they feel so.

More than 50 clinical cases are discussed in this book (both long ones and short ones). Each case starts

with a clinical capsule and questions are formulated based on the clinical capsule. There is a separate

chapter for radiology and imaging and about 32 skiagrams are discussed. Important tables and charts

are included as a separate chapter for ready reference.

This is a clinical book of definitions, checklists, tables, flow charts, questions and answers. All my classes

are distilled into a book and the title is Clinical Surgery Pearls. The preparation of this book took seven long

years of hard work, and I completed this book single handedly. All the clinical photographs are taken by

me with a small Kodak digital camera. The highlighted boxes and charts in this book will make it easily

readable. I am sure, the unique style and the student-oriented approach will make the learning process

a pleasant experience.

R Dayananda Babu


Acknowledgments

I am grateful to:

All my patients, for permitting me to take clinical photographs.

My favorite student Dr Suraj Rajan, who has drawn the medical illustrations in Adobe photoshop and

who is now working in the US. He also read the first “raw copy” and gave suggestions from the “student

point of view”, which is incorporated as student review. I am short of words to thank him.

All my Professors and teachers in surgery. I remember my great teachers like Professor CKP Menon,

Professor KJ Jacob, Professor Mathew Varghese, Professor Balsalam, Professor Mohankumar, Professor

KY Roy and Professor CK Bahuleyan.

My wife Dr Geetha Bhai, who helped me in proofreading and editing this book and without her help

this could not have been possible.

All my postgraduate and undergraduate students in surgery.

Shri Jitendar P Vij (Group Chairman), Mr Ankit Vij (Managing Director) and Mr Tarun Duneja (DirectorPublishing) of M/s Jaypee Brothers Medical Publishers (P) Ltd, New Delhi, India.

Mr PM Sebastian (Branch Manager,Jaypee Brothers, Kochi) and MrArun Kumar(Senior Sales Executive,

Jaypee Brothers, Kochi) and all the staff of Kochi Branch for bringing out this book in time.

Finally, Mr Subramanian, for spending time with me and doing the DTP work of this book.


SECTION 1: Definitions

Definitions..................................................................................................................................................................................... 3

SECTION 2: Long Cases

Case 01: Toxic Goiter................................................................................................................................................................21

Case 02: Solitary Thyroid Nodule (STN-Nontoxic).........................................................................................................45

Case 03: Papillary Carcinoma Thyroid with Lymph Node Metastases...................................................................52

Case 04: Multinodular Goiter ...............................................................................................................................................68

Case 05: Early Breast Cancer.................................................................................................................................................74

Case 06: Advanced Breast Cancer.......................................................................................................................................95

Case 07: Epigastric Lump.................................................................................................................................................... 106

Case 08: Right Hypochondrial Lump without Jaundice.......................................................................................... 119

Case 09: Right Iliac Fossa Mass (Suspected Ileocecal Tuberculosis).................................................................... 128

Case 10: Suspected Carcinoma of the Cecum............................................................................................................. 134

Case 11: Appendicular Mass.............................................................................................................................................. 146

Case 12: Obstructive Jaundice.......................................................................................................................................... 152

Case 13: Varicose Veins........................................................................................................................................................ 168

Case 14: Peripheral Occlusive Vascular Disease.......................................................................................................... 188

Case 15: Lymphoma ............................................................................................................................................................. 207

Case 16: Renal Swelling....................................................................................................................................................... 224

Case 17: Pseudocyst of Pancreas..................................................................................................................................... 235

Case 18: Retroperitoneal Tumor....................................................................................................................................... 241

Case 19: Testicular Malignancy......................................................................................................................................... 248

Case 20: Portal Hypertension............................................................................................................................................ 261

Case 21: Mesenteric Cyst .................................................................................................................................................... 278

Contents

xvi

Clinical Surgery Pearls

SECTION 3: Short Cases

Case 22: Non-thyroid Neck Swelling .............................................................................................................................. 285

Case 23: Tuberculous Cervical Lymph Node................................................................................................................ 288

Case 24: Cervical Metastatic Lymph Node and Neck Dissections........................................................................ 296

Case 25: Carcinoma Tongue with Submandibular Lymph Node ......................................................................... 308

Case 26: Carcinoma of Gingivobuccal Complex (Indian Oral Cancer) ............................................................... 319

Case 27: Parotid Swelling.................................................................................................................................................... 324

Case 28: Submandibular Sialadenitis............................................................................................................................. 335

Case 29: Ranula, Plunging Ranula, Sublingual Dermoid and Mucous Cyst...................................................... 340

Case 30: Thyroglossal Cyst, Lingual Thyroid, Ectopic Thyroid, Subhyoid Bursa and

 Carcinoma Arising in Thyroglossal Cyst....................................................................................................... 343

Case 31: Branchial Cyst, Branchial Fistula, Cystic Hygroma.................................................................................... 349

Case 32: Soft Tissue Sarcoma ............................................................................................................................................ 355

Case 33: Neurofibroma, von Recklinghausen’s Disease........................................................................................... 364

Case 34: Lipoma (Universal Tumor)................................................................................................................................. 370

Case 35: Sebaceous Cyst/Epidermoid Cyst/Wen/Dermoid Cyst........................................................................... 373

Case 36: Ulcer.......................................................................................................................................................................... 378

Case 37: Malignant Melanoma ......................................................................................................................................... 388

Case 38: Basal Cell Carcinoma/Rodent Ulcer............................................................................................................... 402

Case 39: Squamous Cell Carcinoma—SCC (Epithelioma)....................................................................................... 407

Case 40: Carcinoma Penis................................................................................................................................................... 414

Case 41: Congenital Arteriovenous Fistula/Hemangioma/Compressible Swelling...................................... 422

Case 42: Unilateral Lower Limb Edema ......................................................................................................................... 432

Case 43: Hydrocele of Tunica Vaginalis Sac (Epididymal Cyst, Spermatocele,

 Varicocele, Hematocele, Chylocele, etc.) .................................................................................................... 441

Case 44: Inguinal Hernia/Femoral Hernia..................................................................................................................... 450

Case 45: Incisional Hernia (Ventral Hernia, Postoperative Hernia)...................................................................... 469

Case 46: Epigastric Hernia (Fatty Hernia of the Linea Alba)................................................................................... 474

Case 47: Paraumbilical Hernia, Umbilical Hernia in Adults and Children.......................................................... 477

Case 48: Desmoid Tumor, Interparietal Hernia (Interstitial) and Spigelian Hernia ....................................... 483

Case 49: Gynecomastia/Male Breast Carcinoma........................................................................................................ 486

Case 50: Fibroadenoma/Cystosarcoma/Breast Cyst/Fibroadenosis/Fibrocystic

 Disease/ Mastalgia/Mastopathy/Chronic Mastitis................................................................................... 494

Contents

xvii

SECTION 4: Radiology and Imaging

Radiology Questions and Answers................................................................................................................................. 503

SECTION 5: Important Tables and Charts

General...................................................................................................................................................................................... 559

Trauma ...................................................................................................................................................................................... 567

Burns ...................................................................................................................................................................................... 574

Neck ...................................................................................................................................................................................... 578

Breast ...................................................................................................................................................................................... 579

Abdomen ................................................................................................................................................................................. 583

Vascular..................................................................................................................................................................................... 594

Limbs ...................................................................................................................................................................................... 599

Anorectal.................................................................................................................................................................................. 604

Index ...................................................................................................................................................................................607

1. Take up one idea

Make that one idea your life

Think of it, dream of it, live on it

Let the brain, muscle, nerves and every part of your body be full of that idea

Leave the other ideas alone.

—Swami Vivekananda

2. Give the best you have and the best shall come back to you.

—Holy Bible

3. Reading maketh a full man, conference a ready man and writing an exact man.

—Francis Bacon

4. All truth passes through three stages

First, it is ridiculed

Second, it is violently opposed

Third, it is accepted as being self-evident.

—Schopenhauer

5. The world is not divided into the rich and poor, the successes and failures,

but into learners and non-learners.

—Benjamin Barber

Sayings of the Great

S e c t i o n

1

Definitions


1. Abdominal Apoplexy

Spontaneous hemorrhage into the peritoneal cavity.

Causes:

a. Tumors – Hepatoma

 – Spleen

 – Other organs

b. Arteriosclerotic lesion in older individuals

 – Superior mesenteric artery:

 Mesenteric apoplexy (spontan-

 eous rupture)

 – Right colic artery

 – Branches of celiac.

c. Hemorrhage from congenital aneurysm in

young patients — bleeding from splenic artery

aneurysm in pregnancy.

2. Abscess, Cold Abscess

Abscess: It is a localized collection of pus in a

pathological space lined by granulation tissue.

Cold Abscess: Soft fluctuant swelling without signs

of inflammation, which is mistaken for a cyst. This

is lined by granulation tissue and caseous material.

It is due to tuberculous infection and contains

tubercle bacilli. It is not hot. Brawny induration,

edema and tenderness are absent.

3. Acute Abdomen

Any sudden spontaneous nontraumatic disorder

affecting the abdomen for which urgent operation

may be necessary and undue delay in diagnosis may

adversely affect the outcome.

4. Activities of Daily Living (Adl)

It is critical to assess the functional status of the

prospective older candidate for surgery prior to

scheduling an operation.

The activities are:

1. Feeding oneself

2. Bathing

3. Toileting (continence)

4. Transferring from bed to chair

5. Dressing

6. Grooming.

Instrumental ADLs are more complex:

a. Food preparation

b. Shopping

c. Blanching.

5. Agenesis/Atresia

Agenesis: Failure of the development of an organ

or structure.

Atresia: Failure to canalize viscera.

Definitions

Clinical Surgery Pearls

4

6. Amylase

A serum amylase level four times above the normal

is indicative of acute pancreatitis.

7. Ankyloglossia

Inability to protrude the tongue due to involvement

of the muscles of tongue by carcinoma. The tongue

deviates to the affected side.

8. Apathetic Hyperthyroidism

Asymptomatic mild hyperthyroidism occurring in

the elderly recognized only by laboratory findings.

9. Arc of Riolan (Meandering Mesenteric Artery)

The left colic artery near the splenic flexure

bifurcates; one of the branches passes to the right

in the transverse mesocolon to anastomose with

a similar branch of middle colic artery to form

the Arc of Riolan. This has got important role in

supplementing the marginal artery (Fig. 10.1).

10. Bacteremia, Pyemia, Septicemia

Bacteremia: Circulating bacteria in the blood

without producing disease.

Pyemia: Circulating infective emboli composed of

masses of organisms, vegetations and infected clots

in the bloodstream.

Septicemia: Circulation of bacteria in blood

producing disease.

11. Barrett’s Esophagus

It is a metaplasia of the lower esophageal mucosa

due to replacement of the squamous epithelium,

by columnar epithelium, endoscopically having

salmon pink appearance replacing the whitish

squamous epithelium pathologically showing

intestinal type of epithelium with goblet cells.

12. Biliary Colic, Cholecystitis

The term colic is inaccurate for gallbladder. It

produces constant pain in most cases as a result

of obstruction to cystic duct. The pain last for 1–5

hours, and rarely shorter than 1 hour duration (Right

upper quadrant pain radiating to right upper back,

right scapula or between the scapulas). Pain lasting

beyond 24 hours suggests acute inflammation—

Cholecystitis.

13. Boil, Furuncle, Furunculosis, Folliculitis,

Carbuncle

Folliculitis: Affection of the root of one hair follicle

alone by Staphylococcus is called folliculitis.

Boil/Furuncle: Infection of the root of the hair follicle

with perifolliculitis caused by Staphylococcus is

called Boil/Furuncle.

Furunculosis: Multiple boils with intervening normal

tissue is called furunculosis.

Carbuncle: Infective gangrene of skin and subcutaneous tissue caused by Staphylococcus

(multiple boils with involvement of intervening

tissue also).

14. Breast Carcinoma—Definitions

Skin tethering and fixity: The skin tethering is due to

early involvement of ligaments of Cooper.

Manifested as puckering of the skin. The underlying

lump can be moved independently of the skin to

some extent.

Definitions

5

Skin fixity: It is because of invasion of carcinoma

along the ligaments of Cooper to the skin.

The lump and the skin cannot be moved

separately.

Retraction (Recent) of nipple: Extension of growth

along the lactiferous duct and subsequent fibrosis.

Peau d’ Orange appearance is due to blockage

of the lymphatics draining the skin—cutaneous

lymphedema. The hair follicles are more firmly

fixed to the subcutaneous tissue than the rest of

the skin. The hair follicles appear to be retracted

and the between areas swell giving the orange

peel appearance.

Terminal Duct Lobular Unit (TDLU): The functional

unit of the breast is the terminal duct lobular

unit. All cancers of the breast and most benign

conditions arise within TDLU (Fig. 5.4).

Skin Involvement: T4b

Edema (including peau d’ orange) or ulceration of the

skin of the breast or satellite skin nodules confined to

the breast. Dimpling of the skin and nipple retraction

is not considered skin involvement.

Inflammatory carcinoma breast: It is a clinicopathological entity characterized by diffuse erythema and

edema (peau d’ orange) of the breast without an

underlying palpable mass, involving the majority

of the skin of the breast. This is due to tumor

emboli within dermal lymphatics. The biopsy should

demonstrate cancer within the dermal lymphatic or

in breast parenchyma itself. Neglected LABC (locally

advanced breast cancer) is not inflammatory Ca.

Extensive in situ component: If more than 25% of the

main tumor mass contains in situ disease and there

is in situ cancer in the surrounding breast tissue,

the cancer is classified as having an extensive in

situ component.

Chest wall infiltration: Chest wall includes ribs,

intercostal muscles and serratus anterior muscle but

not the pectoral muscle.

Supraclavicular nodes: These are seen in a triangle

defined by the omohyoid muscle and tendon,

internal jugular vein (medial border) and the clavicle

and subclavian vein (lower border). Adjacent nodes

outside this triangle are considered to be lower

cervical nodes (M1).

Multifocal: Tumor foci in the same quadrant are

called multifocal.

Multicentric: Tumor foci in different quadrants are

called multicentric.

Microinvasion: (Ti mic): Microinvasion of 0.1 cm or

less in greatest dimension.

Micrometastasis: Tumor deposits greater than 0.2

mm, but not greater than 2 mm in largest dimension

having histologic evidence of malignant activity

namely proliferation or stromal reaction.

Isolated tumor cells: Single cell or small clusters of

cells not greater than 0.2 mm in largest dimension

with no histologic evidence of malignant activity.

15. Bruit

It is the sound produced by the turbulent blood

flow through a stenotic arterial segment which is

transmitted distally along the course of the artery.

When a bruit is heard over the peripheral vessel,

stenosis is present at or proximal to that level.

It is heard loudest during systole and with greater

stenosis may extend into diastole. The pitch of the bruit

rises as the stenosis becomes more marked. Absence

of bruit does not indicated absence of occlusion.

Clinical Surgery Pearls

6

When the vessel becomes completely occluded, the

bruit may disappear.

16. Burns, Scald, Fat Burn

Burns: Injury by dry heat.

Scald: Injury by moist heat.

Fat burn: Injury by boiling oil.

17. Bursae: Bunion, Clergyman’s Knee, Golfer’s

Elbow, Students Elbow, Housemaid’s Knee,

Tennis Elbow

Bursae: These are fluid-filled cavities lined with

flattened endothelium similar to synovium. Usually

seen in relation to joints. When they develop over

pressure points, they are called adventitious

bursae (see examples). They prevent friction

during movement. Fluctuation, fluid thrill and

transillumination are positive.

Housemaid’s knee: It is a subcutaneous bursa

between patella and skin.

Clergyman’s knee: It is a subcutaneous bursa

between skin and ligamentum patella.

Students elbow: It is a subcutaneous bursa between

skin and olecranon.

Golfer’s elbow: It is medial epicondylitis Tenderness

can be elicited at the medial epicondyle at the

common flexor origin.

Tennis elbow: It is lateral epicondylitis (Common

extensor origin at the lateral epicondyle is affected).

Bunion: It is a subcutaneous bursa between skin and

head of 1st metatarsal bone.

18. Carbuncle

Read boil.

19. Cellulitis, Erysipelas

Cellulitis: Spreading inflammation of subcutaneous and fascial tissue caused by Streptococcus

pyogenes. Commences in a trivial infected wound.

It has “No edge, No fluctuation, No pus and No limit”.

Morison’s aphorism: Cellulitis occurring in children is

never primary in the cellular tissue, but secondary

to an underlying bone infection.

Cellulitis of the scrotum: Always rule out extravasation

of urine.

Erysipelas: It is cuticular lymphangitis.

Milian’s ear sign: Facial erysipelas spreads

and involves the pinna because it is cuticular

lymphangitis. Subcutaneous inflammations stop

short for the pinna because of close adherence of

the skin to the cartilage.

20. Claudication, Rest Pain

Claudication: (I limp). Claudication is the cramp

like muscle pain which appears following exercise

when there is an inadequate arterial blood flow.

It must fulfil three criteria

1. It is a cramp like muscle pain (usually the calf)

2. Pain develops only when the muscle is exercised

3. The pain disappears when the exercise stops.

Rest pain: It is the continuous pain caused by severe

ischemia. This pain is present at rest throughout the

day and the night. The pain is relieved by putting

the leg below the level of the heart.

Definitions

7

21. Clergyman’s Knee

Read bursae.

22. Cold Abscess

Read abscess.

23. Compressibility, Reducibility

Compressibility: When the contents of a swelling can

be emptied by squeezing but the swelling reappear

spontaneously on release of pressure.

Reducibility: When the contents of a swelling can

be emptied by squeezing but does not return

spontaneously. This requires additional force such

as cough or effect of gravity. For example, Hernia.

24. Compound Palmar Ganglion

Compound palmar ganglion: It is a tuberculous

affection of ulnar bursae, with a swelling in the

hollow of the palm, extending to the lower forearm. Cross fluctuation can be elicited between the

palm and lower forearm.

25. Constipation, Obstipation

Constipation: A bowel frequency of less than one

every 3 days. (Fewer than two per week).

Obstipation: (Absolute constipation): Absence of

passage of both stool and flatus.

26. Cough Impulse

Cough Impulse: Expansile impulse seen or felt over

a swelling when the patient coughs, cries or strains.

27. Crepitus

Crepitus: (Grating or crackling sensation imparted

to the examining fingers) may be present when the

joint contain loose bodies. May communicate with

joint. It is also seen in the following conditions:

Subcutaneous emphysema (surgical emphysema)—gas is present in the subcutaneous tissue.

Four types:

a. Traumatic: Fracture ribs, injury to nasal fossa,

breach of continuity of larynx, tracheostomy,

fracture skull involving sinuses

b. Infective: Gas gangrene

c. Extraneous: After fluid administration, closure

of surgical wound, etc.

d. Complicating rupture of esophagus

Fracture of bones

Extravasation of gas in pneumoperitoneum

Pseudo gas gangrene (air entrapped in the

subcutaneous tissue after laparotomy).

28. Cyst

Cyst: It is a pathological fluid-filled sac bound by a

wall. It may be true or false, congenital or acquired.

True cyst: It is one in which the sac is lined with cells

of epithelial origin.

False cyst: It is a walled off fluid collection not lined

by epithelium. False cyst may be inflammatory or

degenerative.

Examples of false cyst

Dental/Radicular cyst

Encysted pleural effusion

Pseudocyst of pancreas

Cystic degeneration of tumors

Brain cyst.

29. Dermoid

Dermoid: Cyst formation due to sequestration of

epithelium deep to the skin surface.

Clinical Surgery Pearls

8

30. Dietl’s Crises

Dietl’s crisis: This is because of intermittent hydronephrosis. After an attack of acute renal pain, a

swelling is found in the loin due to the hydronephrosis. Following the passage of large volume of

urine some hours later, the pain is relieved and the

swelling will disappear.

31. Diverticulum, Diverticulosis

Diverticulum: Abnormal external projection from

a hollow viscus external to the serosa is called

diverticulum. It may be true or false, congenital

or acquired. Congenital is true and acquired is false

(one meaning of diverticulum is a wayside house

of ill-fame).

True diverticulum: Containing all the layers of the

bowel wall.

False diverticulum: There is no muscle coat, but all

other layers (herniation of mucosa or submucosa

through the muscular coat).

Pulsion diverticulum: The diverticulum is pushed out

by intraluminal pressure.

Traction diverticulum: Diverticulum develops as a

result of external traction.

Diverticulosis: Presence of multiple false diverticulae.

32. Diarrhea

Diarrhea: If stools contain more than 300 mL fluid

daily.

33. Edema

Edema: It is an imbalance between capillary

filtration and lymphatic drainage (this does

not mean that all edemas are lymphedemas).

This will occur only when the lymphatic system

fails to drain the tissue fluid produced by normal

capillary filtration.

34. Empyema

Empyema: It is collection of pus in a physiological

space.

35. Erysipelas (Read Cellulitis)

Erysipelas: Spreading cuticular lymphangitis caused

by Streptococcus pyogenes. It has a sharply defined

margin unlike cellulitis. The vesicles contain serum.

Milian’s ear sign—Erysipelas can spread to the

pinna.

36. Erythroplakia, Leukoplakia

Erythroplakia: Any lesion of the oral mucosa that

presents as bright red velvety plaques that cannot

be characterized clinically or pathologically as any

other recognizable condition.

Leukoplakia: Any white patch or plaque that cannot

be characterized clinically or pathologically as any

other disease.

37. Exotoxin, Endotoxin

Exotoxin: Toxin liberated by living bacteria.

Endotoxin: Toxin liberated after death of bacteria,

being a part of the organism itself.

38. Evidence—Levels

Levels of evidences: Agency for health care policy

and research grading system for evidence and

recommendation.

Definitions

9

Recommendation of Strength:

A – Directly based on category I evidence.

B – Directly based on category II evidence or

extrapolated recommendation from category

I evidence.

C – Directly based on category III evidence or

extrapolated recommendation from category

I or II evidence.

D – Directly based on category IV evidence or

extrapolated recommendation from category

I, II, or III evidence.

Levels of evidences: Pragmatic grading (only three

grades).

Evidence Description

I a Evidence from meta-analysis of randomized controlled trials RCT

I b Evidence from at least one RCT

II a Evidence from at least one controlled study without randomization

II b Evidence from at least one other type of quasi-experimental study

III Evidence from nonexperimental descriptive studies, such as comparative studies and case

control studies

IV Evidence from expert committee reports or opinions or clinical experience of respected

authorities or both.

Levels of Evidences Recommendations

I. Beyond reasonable doubt, high quality RCT,

systematic reviews, high quality synthesized

evidence

A. Strong recommendations which should be followed

II. On the balance of probabilities

Evidence of best practice from high quality review

of literature

B. Based on evidence of effectiveness that may need

interpretation in the light of other factors like local

facilities, audit, etc.

III. Unproven in sufficient evidence upon which to

base a decision or contradictory evidence

C. When there is inadequate evidence

39. Fistula, Sinus

Fistula: It is a communicating tract between two

epithelial surfaces lined with granulation tissue.

It may be a communication between the skin and

hollow viscera or between two hollow viscerae

(Internal fistula).

Sinus: Sinus is a blind track leading from the surface

down to the tissue lined by granulation tissue/

epithelium.

Fistula-in-ano: The pathology of fistula-in-ano is

‘cryptoglandular infection’ (Infection of the anal

glands in the crypt).

Clinical Surgery Pearls

10

40. Flail Chest

Flail chest: Three or more ribs fractured in 2 or more

places.

Bilateral costochondral separation will result in

flail sternum.

41. Folliculitis

Read boil.

42. Ganglion

Ganglion: Cystic, myxomatous degeneration of

fibrous tissue. They are not pockets of synovium

protruding from joints. It may be multilocular

occasionally.

Content—Viscous gelatinous material.

Disappear underneath adjacent structure during

certain movements.

Fluctuation is present if not tense.

43. Gangrene, Necrosis, Infarction, Slough

Gangrene: Macroscopic death of tissue with

putrefaction.

Necrosis: Microscopic death of tissue.

Infarction: Ischemic necrosis is called infarction.

Slough: A piece of dead tissue separated from living

tissue.

44. Early Gastric Cancer

Early gastric cancer: Cancer of the stomach confined

to the mucosa and submucosa irrespective of the

nodal status.

45. Gastrinoma

Gastrinoma: A basal gastric acid output more than

15 mmol/HR and a fasting gastrin level of more than

200 pg/mL is strongly supporting the diagnosis.

46. Gastrinoma Triangle (Passaro’s Triangle)

Gastrinoma triangle: The three points forming the

triangle are:

1. Junction between the head and neck of the

pancreas.

2. Junction of cystic duct with CBD.

3. Junction between 2nd and 3rd parts of the

duodenum.

47. Goiter

Goiter: Any enlargement of thyroid gland is called

goiter.

Grading of goiter:

WHO (1990) Perez Classification

Grade 0 No goiter

Grade I a Not visible, but palpable

Grade I b Visible with neck extended and palpable

Grade II Visible with neck in normal position and

palpable

Grade III Large gland evident from a distance.

WHO classification (1994)

Grade 0 – No palpable or visible swelling

Grade 1 – A mass in the neck that is consistent with

an enlarged thyroid that is palpable, but not visible

when neck is in normal position. It moves upwards

in the neck as the subject swallows

Definitions

11

Grade 2 – A swelling in the neck that is visible when

the neck is in a normal position and is consistent

with an enlarged thyroid when neck is palpated.

Large goiter:

Protrusion of goiter beyond chin or jaw.

Goiter which weighs 80 g or more after excision.

Largest neck circumference crossing the goiter

being 40 cm or more.

Stage III—WHO classification.

48. Granuloma

Granuloma: Tumor-like mass formed in chronic

inflammatory tissue.

49. Hamartoma, Teratoma

Hamartoma: A tumor-like formation of tissues

indigenous to the site due to developmental

aberration.

Teratoma: Tumor-like proliferation of tissues, not

indigenous in origin, containing more than one

germinal layer.

50. Hematemesis, Melemesis, Melena,

Hematochezia

Hematemesis: Vomiting of bright red or dark blood.

Melemesis: Vomiting of altered blood is called

melemesis. Coffee ground vomitus is due to vomiting

of blood that has been in the stomach long enough

for gastric acid to convert Hb to methemoglobin.

Melena: Passage of black or tarry sticky, semisolid,

stools because of the presence of altered blood. It

can be produced by blood entering the bowel at any

point from mouth to cecum. The black color is due

the Hematin (from Heme). 50 to 100 ml of blood

in stomach can produce melena. 1 liter of blood in

stomach will produce melena for 3–5 days.

Hematochezia: Passage of bright red blood from

the rectum (Colon, rectum and anus) is called

hematochezia. Brisk bleeding from upper intestine

with rapid transit can also produce it.

51. Hernia, Prolapse

Hernia: Abnormal protrusion of a viscus or part of a

viscus lined by a sac through a normal or abnormal

opening in the abdominal wall.

Prolapse: Abnormal protrusion of a viscus through

a normal or abnormal opening not lined by a sac.

52. Hurthle Cell Tumor

Hurthle cell tumor: Presence of more than 75%

follicular cells having oncocytic features in thyroid

histology is called Hurthle cell tumor.

53. Hydronephrosis, Dietl’s Crisis (Read Above)

Hydronephrosis: Aseptic dilatation of pelvicalyceal

system due to partial or intermittent obstruction.

54. Hyperparathyroidism

Hyperparathyroidism: The combinations of increased

PTH levels and hypercalcemia without hypocalciuria

(Hypercalciuria of more than 400 mg/24 hour is

diagnostic).

55. Incontinence of Urine

Incontinence of urine: Involuntary evacuation of

urine.

Clinical Surgery Pearls

12

56. Incontinence of Stool

Incontinence of stool: Involuntary evacuation of

stool.

Three Types

a. Incontinence for solid feces

b. Incontinence for liquid feces

c. Incontinence for gas.

57. Infarction

Read gangrene.

58. Inguinal Canal

Inguinal canal: It is an intermuscular slit situated

between the superficial inguinal ring and deep

inguinal ring.

59. Intussusception

Intussusception: Telescoping of proximal intestine

to the distal intestine.

Retrograde intussusception: Telescoping of distal

intestine into the proximal intestine (e.g. jejunogastric intussusception) after gastrojeunostomy).

60. Jaundice

Jaundice: Yellowish discoloration of skin and

mucous membrane due to excessive circulating bile.

61. Karnofsky Performance Status (Kps)

Karnofsky performance status (KPS): The KPS is

reliable independent predictor of survival of

outcome for patients with solid tumors. It is a

required baseline assessment in clinical protocols

in head and neck and other cancers.

The American joint committee on cancer (AJCC)

strongly recommends recording of KPS along with

standard staging information (TNM). It is a method

of measuring co-morbidity. It provides a uniform

objective assessment of an individuals functional

status. The scale in ten point increments from zero

(Dead) to 100 (Normal, no complaints, no evidence

of disease)wasdevisedin1948byDavid A Karnofsky.

Karnofsky Performance Status (KPS)

100 – Normal; no complaints; no evidence of disease.

90 – Able to carry on normal activity; minor signs or

symptoms of disease. 80 – Able to carry on normal

activity with effort; some signs or symptoms of

disease. 70 – Care for self; unable to carry on normal

activity or do active work. 60 – Requires occasional

assistance, but is able to care for most of own needs.

50 – Requires considerable assistance and frequent

medical care. 40 – Disabled; requires special care and

assistance. 30 – Severely disabled, hospitalization is

indicated by although death is not imminent. 20 –

Very sick. Hospitalization necessary. Active supportive

treatment is needed. 10 – Moribund. Fatal process

rapidly progressing. 0 – Dead.

A. Able to carry on normal activity. No special care

is needed (scale 80–100).

B. Unable to work, able to live at home, cares

for most personal needs; a varying amount of

assistance is needed (50–70).

C. Unable to take care of self; requires the equivalent of institutional or hospital care; disease

may be progressing rapidly (scale 10–40).

62. Line of Demarcation

Line of demarcation: Zone of demarcation between

viable and gangrenous tissue indicated by a band of

hyperemia and hyperesthesia on the surface and

separation is achieved by a layer of granulation tissue.

Definitions

13

In dry gangrene the line of demarcation appears

in a matter of days without infection and this is

called “separation by aseptic ulceration”.

In moist gangrene the line of demarcation is

more proximal than dry gangrene and the process

is called “separation by septic ulceration”.

63. Lipoma (Universal Tumor)

Lipoma: It is benign tumor from “adult fat cell“ It is

called “universal tumor” or “ubiquitous tumor” and

hence the aphorism: “when in doubt hedge on fat”.

64. Lower Gi Bleed, Upper Gi Bleed

Lower GI bleed: It is a bleeding from distal to the

ligament of Treitz.

Upper GI bleed: It is a bleeding from proximal to the

ligament of Treitz.

65. Marginal Artery of Drummond, Arc of Riolan

(Read Above)

Marginal artery of Drummond: It is the paracolic

vessel of anastomosis between the superior

mesenteric and inferior mesenteric arterial system.

66. Massive Hemothorax

Massive hemothorax: When 1500 mL or more of

blood is acutely removed from the pleural space,

then it is called massive hemothorax.

67. Massive Blood Transfusion

Massive blood transfusion: The term massive transfusion

implies a single transfusion greater than 2500 mL or

5000 mL transfused over a period of 24 hours.

68. Melena, Melemesis

Read hematemesis.

69. Menarche—Early

Early menarche: Age of menarche before 12 years.

70. Menopause—Late

Late menopause: Menopause after 50 years.

71. Mesentery of Small Intestine — Attachment

Mesentery of small intestine—attachment: The

base of the mesentery attaches to the posterior

abdominal wall to the left of the second lumbar

vertebra and passes obliquely to the right and

inferiorly to the right sacroiliac joint crossing 3rd

part of the duodenum, aorta, IVC and right ureter.

It is 6 inches (15 cm) in length. Remember the small

intestine has got 6 meters length (Fig. 2.1).

72. Mesentery of Sigmoid — Attachment

Mesentery of sigmoid—attachment: It is shaped like

an inverted V. The apex of the V is at the bifurcation

of left common iliac artery crossing the brim. The

right limb descends to the third piece of the sacrum.

The left limb runs along the brim of left side of

pelvis (Fig. 10.2).

73. Mesentery of The Transverse Colon

Mesentery of the transverse colon: It is attached to

the descending part of duodenum to the head

and lower aspect of the body of the pancreas and

placed horizontally to the anterior surface of the

left kidney.

Clinical Surgery Pearls

14

74. Necrosis

Read gangrene.

75. Old Age

Old age: Above 65 years is old age and above 85

years is very old age.

76. Oral Cavity, Buccal Mucosa, Retromolar

Trigone, Trismus, Ankyloglossia

Oral cavity: Starts at skin vermilion junction of

lip anteriorly to circumvallate papillae of tongue,

posterior part of the hard palate, and anterior

pillar of tonsil posteriorly. Oral cavity includes the

following.

Lips

Buccal mucosa

Upper and lower alveolar ridge

Retromolar trigone

Floor of the mouth

Hard palate

Oral tongue.

Buccal mucosa: Extends from the upper alveolar

ridge down to the lower alveolar ridge, and from

the commissure anteriorly to the mandibular ramus

and retromolar region posteriorly.

Retromolar trigone: It is defined as the anterior

surface of the ascending ramus of the mandible. It

is triangular in shape with the base being superior

behind the third upper molar tooth and the apex

inferior behind the 3rd lower molar.

Trismus: (Spasmodic clenching) is inability to open

the mouth.

Causes for Trismus

Oral carcinoma—Involvement of pterygoid,

masseter, temporalis and buccinator muscle.

Inflammatory—Parotitis

Tooth abscess (Dental)

Erupting wisdom tooth

Peritonsillar abscess (Quinsy)

Tetanus — (Painful smiling risus sardonicus).

Ankyloglossia (Read above).

77. Pancreatitis, Pancreatic Necrosis, Pancreatic

Abscess, Pancreatic Ascites, Pancreatic

Effusion, Pseudocyst, Pancreatic Necrosis,

Acute Fluid Collection

Chronic pancreatitis: It is a disease in which there is

irreversible progressive destruction of pancreatic

tissue. Its clinical course is characterized by

dynamic progressive fibrosis of the pancreas.

Acute Pancreatitis

Acute fluid collection: It is fluid collection in or near

the pancreas with ill defined wall occurring early in

acute pancreatitis.

Pancreatitis acute pseudocyst: It is a collection of

pancreatic juice enclosed in a wall of fibrous or

granulation tissue (Requires 4 weeks).

Pancreatic necrosis: Diffuse or focal area of nonviable pancreatic parenchyma. Associated peripancreatic fat necrosis is present.

Infected pancreatic necrosis: Same as above with

infection.

Pancreatic abscess: Circumscribed intra-abdominal

collection of pus in proximity to pancreas. There is

no pancreatic necrosis.

Definitions

15

Pancreatic ascites: Chronic generalized peritoneal

enzyme rich effusion associated with pancreatic

ductal disruption.

Pancreatic effusion: Encapsulated collection of fluid

in the pleural cavity.

78. Papilloma (Benign Papilloma), Polyp,

Polyposis

Benign papillomas: These are hamartomas

consisting of an overgrowth of all skin layers

and its appendages having a central core and

normal sensation. They are well-defined, usually,

pedunculated ranging from few millimeters to a

few centimeters in size, commonly 5 mm across.

The surface may be grooved or deeply fissured.

The complications of papilloma are inflammation,

bleeding ulceration, pigmentation and keratosis.

Polyp: It is a morphological term and no histologic

diagnosis is implied. They are masses of tissue

that project into the lumen of viscera. When the

base is broader than the head, it is called sessile.

When the base is narrower than head, it is called

pedunculated. It may be benign or malignant,

mucosal or sub-mucosal or muscular.

Polyposis: Presence of many polyps.

Classification of polyp

a. Neoplastic

Adenoma Tubular adenoma,

 – Tubulovillous

 – Villous adenoma

Carcinoid

Adenocarcinoma

b. Hamartomatous

Juvenile polyp (associated with malrotation or

 Meckel’s diverticulum)

Peutz–Jeghers polyps

c. Inflammatory (Pseudo-polyp)

Benign lymphoid polyp

d. Hyperplastic polyp (Metaplastic polyp)

Diminutive lesions most often found in leftside

 of the colon

e. Miscellaneous

Lipoma

Leiomyoma.

79. Paralytic Ileus

Paralytic ileus: Defined as a state in which there is

failure of transmission of peristaltic waves in the

intestine secondary to neuromuscular failure [in

the myenteric (Auerbach) and the submucous

(Meissner) plexuses.

80. Paraphimosis, Phimosis

Phimosis: Inability to retract the foreskin to expose

the glans.

Paraphimosis: Inability to reduce a previously

retracted foreskin.

81. Peau D’ Orange

Read breast

82. Perfusion, Transfusion

Perfusion: Artificial passage of fluid through blood

vessel (usually veins).

Transfusion: Intravenous administration of blood or

its components.

83. Prolapse—Read Hernia

Abnormal protrusion of a viscus through a normal

or abnormal opening not lined by a sac.

Contd...

Contd...

Clinical Surgery Pearls

16

84. Pseudo Thyrotoxicosis

Seen in critically ill patients characterized by

increased levels of T4

 and decreased levels of T3

 due

to failure of conversion of T4

 to T3

.

85. Pus

Pus: It is a fluid composed of living and dead

bacteria, dead fixed and free cells (the latter

representing body’s phagocytic response) and

foreign material such as sutures, implants and

splinters.

Color of the pus may give a clue regarding the

organism.

Creamy yellow—Staphylococci

Watery opalescent—Streptococcus

Blue/Green—Pseudomonas

Purplish brown—Amebic liver abscess

Yellow granules—Actinomycosis.

86. Renal Angle

Renal angle: Angle between the 12th rib and the

edge of the erectorspinae muscle. Normally this

is empty and resonant. There should not be any

tenderness.

Rest pain: It is the continuous pain caused by severe

ischemia. This pain is present at rest throughout the

day and the night. The pain is relieved by putting

the leg below the level of the heart.

87. Retention of Urine

Retention of urine: Accumulation of urine in the

bladder with inability to void.

Acute retention: Sudden inability to pass urine with

a painful bladder.

Chronic retention: Retention with a painless bladder.

Size of Urinary Catheter

French or Charriere’s scale

Fr or Ch

3 Fr = 1 mm outer diameter of catheter

Recall Shakespeare’s ‘Seven Ages of

Man’ from As You Like It.

The entire World is a stage

And all the men and women merely players;

They have their exits and their entrances;

And one man in his time plays many parts,

His acts being seven ages. At first the infant,

Mewling and puking in the nurse’s arms.

And then the whining school boy, with his satchel,

And shining morning face, creeping like snail,

Unwillingly to school. And then the lover,

Sighing like furnace, with a woeful ballad

Made to his mistress’ eyebrow. Then a solider,

Full of strange oaths and bearded like the pard,

Jealous in honor, sudden and quick in quarrel,

Seeking the bubble reputation

Even in the cannon’s mouth. And then the justice,

In fair round belly with good capon lin’d,

With eyes severe, and beard of formal cut,

Full of wise saws and modern instances;

And so he plays his part. The sixth age shifts

Into the lean and slipper’d pantaloon,

With spectacles on nose and pouch on side;

His youthful hose well say’d a world too wide

For his shrunk shank; and his big manly voice,

Turning again towards childish treble, pipes

And whistle in his sound. Last scene of all,

That ends this strange eventful history,

Is second childishness, and mere oblivion

Sans teeth, sans eyes, sans taste, sans everything

Definitions

17

Important causes for retention of urine as per the

seven ages are:

1. The infant – Posterior urethral valve

2. The school boy – Enlarged bladder neck

 (Marion’s disease)

 – Obturation by stone

3. The “lover age” – Retention following acute

 urethritis

4. The soldier – Urethral stricture

5. The justice – Benign enlargement of

 the prostate

6. The sixth age – Carcinoma of the prostate

7. The last age – Carcinoma of the prostate

 – Benign enlargement of

 the prostate

Three most important causes for acute retention

in female:

Retrover ted gravid uterus (Do bimanual

palpation of uterus)

Disseminated sclerosis (CNS examination).

Hysteria.

“Bashful bladder”—Cannot pass urine when another

person is in the vicinity.

88. Retromolar Trigone

Read oral cavity

89. Rigidity, Guarding

Reflex contraction of the abdominal wall muscles

secondary to intraperitoneal inflammation.

Rigidity: In rigidity there is contraction even at rest.

Guarding: In guarding it is secondary to provocation

from the examining hand of the physician.

90. Run in, Distal Run off

Distal run off: Patency of the main vessel beyond an

arterial occlusion seen in angiogram.

Run in: Patency of the main vessel proximal to the

site of occlusion in angiogram.

91. Scoliosis

Scoliosis: Rotatolateral deformity of the spine.

92. Screening, Surveillance

Screening: It is defined as testing a group of people

considered to be at normal risk for a disease, to

discover those at increased risk.

Surveillance: It is defined as testing of a group

known to be at increased risk for a disease.

93. Sinus

Read fistula

94. Stricture, Stenosis

Stricture: Narrowing of a length of canal or hollow

organ.

Stenosis: Narrowing of a segment of canal or orifice.

95. Strangury, Tenesmus

Strangury: Painful, frequent, ineffective attempts

at micturition.

Tenesmus: Painful, frequent, ineffective attempts

at defecation.

96. Tension Pneumothorax

Tension pneumothorax: Presence of air in the pleural

cavity with signs of mediastinal shift like: Tracheal

shift or and Shift of Apex beat.

Clinical Surgery Pearls

18

Differences between simple pneumothorax and

tension pneumothorax

 Simple Tension

Tracheal position Normal Displaced

Percussion note Normal Increased

Jugular pressure Normal Elevated (unless

 hypovolemic)

Breath sounds Normal Decreased

Pulse Normal Weak

BP Normal Low

A tension pneumothorax impairs venous return

by caval distortion from mediastinal shift and raised

intrathoracic pressure with compression of the

contralateral lung.

Radiological signs of tension pneumothorax:

1. Tracheal shift

2. Spreading of the ribs (Space between ribs

increased)

3. Lowering of hemidiaphragm.

97. Third Day Fever

Third day fever: If a patient is developing fever on the

third postoperative day of surgery, suspect septic

foci in the IV cannula.

98. Tubercle, Caseous Material, Tuberculous Pus

Tubercle: Microscopically consists of an area of

caseation surrounded by:

a. Giant cells (having 20 or more peripherally

arranged nuclei)

b. Zone of epithelioid cells around giant cells

c. Zone of inflammatory cells—lymphocytes and

plasma cells.

Tubercle is visible to the naked eye towards the

end of second week.

Caseous material: It is a dry, granular and cheese

like material (Granular structureless material

microscopically).

Tuberculous pus: Softening and liquefaction of the

caseous material result in a thick creamy fluid called

tuberculous pus. Liquefaction is associated with

multiplication of bacteria. It is highly infective.

It contains fatty debris in serous fluid with a few

necrotic cells (It is usually sterile).

99. Ulcer

Ulcer: Abnormal breach in the continuity of the skin or

mucous membrane due to molecular death of tissue.

100. Upper GI Bleed

Read Lower GI.

101. Varicose Vein

Varicose vein: (WHO Definition) Abnormally dilated

saccular or cylindrical superficial veins which can be

circumscribed or segmental.

102. Volvulus

Volvulus: Axial rotation of a portion of bowel about

its mesentery. Volvulus can occur in the cecum,

sigmoidcolon and in the stomach.

In the stomach, there are two types of volvulus.

Organoaxial—rotation ofstomach in horizontal

direction (common).

Mesenteroaxial—rotation ofthe stomach in the

vertical direction.

103. WEIGHT LOSS

Weight loss: Loss of more than 10% body weight

over a period of 6 month.

S e c t i o n

2

Long Cases


Case Capsule

A 30-year-old female patient with a thin build has

presented with diffuse enlargement of the thyroid

and palpitation of 6 months duration. She complains

of increased appetite and loss of weight. She is

apparently irritable and says, she is intolerant to

hot weather with excessive sweating. She has a

preference for cold weather. She also complains of

insomnia and loss of concentration ability. She has

diarrhea in addition. She is married and has a baby of

6 months old. She complains of amenorrhea for the

last 3 months. On examination, patient is agitated and

nervous. Examination of the palms revealed that they

are moist and sweaty. She has tachycardia, fine and

fast tremor and protruded eyeballs. There is visible

diffuse enlargement of the thyroid. On auscultation,

there is a systolic bruit heard in the upper pole of the

thyroid. The carotids are felt in the normal position. The

trachea is central. There is no evidence of retrosternal

extension. The cervical lymph nodes are not enlarged.

In all goiters or swelling in the neck assess the

following:

1. What is the anatomical diagnosis—by assessing

the plane—deep to the deep fascia and deep to

the sternomastoid?

2. What is the pathological diagnosis, e.g. nodular

goiter, solitary thyroid nodule, carcinoma, etc.

3. What is the functional diagnosis—whether the

patient is euthyroid, hyperthyroid, hypothyroid?

Checklist for history

Onset related to puberty, pregnancy

Residence: Endemic area or not

Ingestion of goitrogens

Intolerance to hot/cold temperature

Increased appetite with loss of weight (Hyperthyroidism)

Gain in weight (Hypothyroidism)

Change in menstrual cycle

Bowel habit—diarrhea (hyper), constipation

(hypo)

Difficulty in swallowing

Difficulty in breathing

Hoarseness of voice

Postural cough during sleeping (retrosternal

extension)

Historyofpalpitation/shortnessofbreathonexertion

Insomnia, loss of concentration (hyper)

Irritability/nervousness (hyper).

1 Toxic Goiter

Case

Clinical Surgery Pearls

22

Checklist for examination of thyroid

Always check the pulse for tachycardia before examining the thyroid

Look for tremor of hands and tongue before examining the thyroid

Ask the patient to take a sip of water and to hold it in his/her mouth. Then ask the patient to swallow (goiter

moves on swallowing)

Ask the patient to put out the tongue (thyroglossal cyst moves up)

Stand behind the patient and palpate the thyroid (ask the patient to take another sip of water)

Decide whether it is diffuse enlargement,single nodule, multiple nodules and the nature of the surface

Decide the consistency

Look over the top of the head for exophthalmos (look for lid lag, lid retraction and other eye signs)

Check the eye movements, double vision

Now stand in front of the patient for palpation of the trachea for deviation, for assessing the lower limit by

‘getting below’

Assess the plane of the swelling (stretch the deep fascia by extending the neck and see whether it becomes less

prominent, contract the sternomastoid muscle against resistance and see whether it becomes less prominent

Do Pemberton’s test for retrosternal extension

Percuss the manubrium sterni for dullness (seen in retrosternal extension)

Palpate the carotids on both sides

Examine the regional lymph nodes

Feel the skin (dry in hypothyroidism, shiny skin in hyperthyroidism)

Look for pretibial myxedema (hyperthyroidism)

Assess the build of the patient (Thin—hyperthyroidism, obese—hypothyroidism)

Examine the palms—warm, moist and changes of acropachy in hyperthyroidism

Assess the behavior of the patient (agitated in toxic, lethargic in hypothyroidism)

Ask the patient to rise from squatting position without using hands for support (proximal myopathy in

hyperthyroidism)

Test the biceps reflex and look for slow relaxing reflex suggestive of hypothyroidism.

Final checklist for clinical examination of thyroid

1. Look for signs of toxicity

2. Look for signs of malignancy

3. Look for signs of retrosternal extension

4. Look for position of carotid artery

5. Look for position of trachea

6. Look for cervical lymph nodes

7. Look for bony swellings especially in the scalp.

Toxic Goiter

23

Clinical Surgery Pearls

24

Q 1. Why is the swelling a goiter?

The points in favor of goiter are:

1. The plane of the swelling is deep to deep fascia

and deep to sternocleidomastoid (the deep

fascia of the neck is stretched by extending the

neck and sees whetherthe swelling is becoming

less prominent or not, similarly contracts the

sternomastoid muscles.

2. The swelling moves up and down with deglutition.

3. It occupies the normal position of thyroid.

4. It is having the shape of thyroid.

Q 2. Why does the thyroid gland move up and

down with deglutition?

The inferior constrictor muscle has two parts namely,

thyropharyngeus and cricopharyngeus and they are

attached respectively to thyroid cartilage and cricoid

cartilage. Therefore, when the patient swallows this

muscle will contract and the thyroid and cricoid

cartilage will move up. The thyroid gland is attached

to the cricoid by means of the suspensory ligament

of Berry that is nothing but a condensation of

pretracheal fascia. Therefore, the thyroid gland will

move up and down with deglutition.

Q 3. What is goiter?

Any enlargement of thyroid gland is called goiter.

Even though for neoplasms we call it malignancy

and for inflammation we call it thyroiditis.

Q 4. Can the normal thyroid be felt on palpation?

Inareasonablyslenderpersonitcanbefeltasasmooth

firm structure that moves upwards during deglutition.

Q 5. How do you grade a goiter?

WHO grading (1994) of goiter

Grade 0 : No palpable/visible goiter

Grade 1 : Athyroidthatispalpable but not visible

when the neck is in normal position

Grade 2 : An enlarged thyroid that is visible with

the neck in normal position.

Q 6. What are the signs of retrosternal extension?

1. Cannot‘get below’the swelling.

2. Pemberton’s test positive (arm raising test)—

when both arms are elevated so as to touch

the sides of the face, after a few moments there

will be congestion of face, some cyanosis and

distress. This is due to the narrowing of the

thoracic inlet and when the arms are elevated

this results in obstruction of great veins of the

neck.

3. On percussion over the manubrium sterni there

will be dullness (normally this is resonant).

4. Radiological assessment.

Q 7. How will you assess the position of trachea?

The position of trachea can be assessed by:

1. Palpation oftrachea by three fingertest(this will

be difficult in case of large goiter).

2. Auscultation to detect the position of trachea.

3. Radiological.

Q 8. In which position you normally palpate a

patient with thyroid?

The examiner stands behind the patient and will

do the palpation.

Q 9. What is Kocher’s test?

Slight compression on the lateral lobes of thyroid

producesstridor. Ifthistestispositive itsignifiesthat

the patient has an obstructed trachea.

Q 10. What are the conditions in which you get

narrowing of the trachea?

Narrowing of trachea is found in:

1. Carcinoma of the thyroid

2. Retrosternal goiters

3. “Scabbard” trachea of long standing multinodular

goiter

4. Riedel’s thyroiditis.

Toxic Goiter

25

Q 11. What is plunging goiter?

In this condition,the whole ofthe enlarged thyroid

lies in the superior mediastinum and there is no

palpable thyroid gland in the neck. When the

intrathoracic pressure rises as in coughing, the

goiterwillbe seen intheneck,thisis calledplunging

goiter.

Q 12. What is Berry’s sign?

In goiter, the carotid artery may be pushed

posteriorly by the enlarging thyroid and this is

called displacement. When there is infiltration of

the carotid by tumor the carotid pulse on that side

will be absent. This absent carotid pulse is called

Berry’s sign.

Q 13. What are the signs of malignancy in a goiter?

Signs of malignancy in a goiter

1. Rapidly enlarging thyroid

2. Hard consistency (unripe apple)

3. Fixity of the thyroid (the lateral mobility becomes

restricted before there is noticeable movement on

deglutition

4. Regional lymph nodes (the first lymph node to be

involved in carcinoma is calledDelphic lymph node

which is nothing but prelaryngeal lymph node). This

is also called the Delphian lymph node

5. Berry’s sign (absent carotid pulse)

6. Horner’s syndrome

7. Hoarseness of voice

8. Stridor due to tracheal obstruction

9. Distant metastases (pulsatile, bony swelling from

the scalp).

Q 14. Where will you auscultate for thyroid bruit?

The usual position to look for thyroid bruit is at the

upper pole where the superior thyroid artery enters

the thyroid gland.

Q 15. What are your points in favor of toxicity in

this patient?

The toxicity is diagnosed on the basis of symptoms

andsignsinthispatient.Thesymptomsoftoxicityare:

Symptoms of toxicity

System Symptoms

Nervous system, Nervousnessagitation,irritability,

 insomnia, nervous instability,

 tremor of the hands and tongue

Cardiovascular Palpitation, dyspnea on system

system exertion, chest pain, etc.

Metabolic and Increase in appetite and loss

alimentary of weight, change in bowel habit,

system usually diarrhea; preference for

 cold weather; excessive sweating;

 intolerance to hot weather

Menstrual changes Usually amenorrhea or oligo-

 menorrhea

Musculoskeletal Generalizedweightloss;wasting

and weakness of small muscles

 of the hand, shoulder and face.

Skin Pretibial myxedema

Nail Onycholysis—Plummer’s nail.

Signs of toxicity

1. Uniform, smooth, soft or firm enlargement of

thyroid in Graves’ disease (primary) bosselated

swelling or solitary nodule in case of secondary

2. The gland is vascular as evidenced by bruit

3. Tremor of the outstretched hands (fine, fast) and

tongue

4. Warm and moist hands

5. Tachycardia

6. Extra systoles, atrial fibrillation, and cardiac failure

7. Eye signs

8. Myopathy — weakness of the proximal limb muscle

is commonly found. Severe muscular weakness

resembling myasthenia gravis occurs occasionally.

Inability togetupfromchairis calledPlummer’s sign.

Clinical Surgery Pearls

26

Q 16. What are the eye signs of thyrotoxicosis?

Eye signs

1. Lid retraction—this sign is caused by over activity

of involuntary smooth muscle part of the levator

palpebrae superior is muscle. If the upper eye lid is

higher than normal and the lower lid is in correct

position, the patient has lid retraction (this is not

exophthalmos)

2. Lid lag (Von Graefe’s sign)—when the upper lid does

not keep pace with the eyeball asit follows a finger

moving from above downwards, it is lid lag

3. Exophthalmos—here both the eyelids are moved

away from center with sclera visible below or all

around. Here the eyeball is pushed forwards by

increase in retro-orbital fat, edema, and cellular

infiltration (sclera should be always visible below

the lower edge of eyes in exophthalmos)

4. The other eye signs are:

a. Widening of the palpebral fissure (Stellwag’s

sign) this is due to lid retraction

b. Joffroy’s sign—absence of wrinkling of the

forehead when the head is bent down

c. Möbius’s sign—difficulty in convergence when

the patient is asked to look at near objects

5. Severe exophthalmos—Intraorbital edema is super

added to the increased deposition of intraorbital

fat. It comprises of:

a. Intraorbital congestion—watering of eyes,

dilated blood vessels in lateral conjunctiva

b. Increased intraocular tension

c. Muscle paralysis (Ophthalmoplegia)—evidenced by double vision, especially when eye is

moved upward end and outwards (muscles of

elevation and abduction namely, superior rectus

and inferior oblique muscles are affected)

d. Chemosis.

Q 17. What is pretibial myxedema?

This is a misnomer and it is seen in primary toxicosis

(In Graves’ disease with exophthalmos only). It is

usually symmetrical. The earliest stage is a shiny red

plaqueofthickenedskinwithcoarsehair,whichmay

be cyanoticwhencold. Insevere casesthe skinofthe

whole legbelowknee isinvolved,togetherwiththat

of foot and the ankle and there may be clubbing of

the fingers and toes (Thyroid acropachy).

Q 18. What are the three most important clinical

types of toxicity?

Clinical types of thyrotoxicosis

1. Primary thyrotoxicosis/Graves/diffuse toxic goiter

2. Secondary thyrotoxicosis/Plummer’s disease/toxic

nodular goiter

3. Toxic nodule/adenoma/autonomous nodule.

Q 19. What is the difference between thyrotoxicosis and hyperthyroidism?

Thyrotoxicosis refers to the biochemical and

physiological manifestations of excessive thyroid

hormone. Hyperthyroidism is a term reserved

for disorders that result in the over production

of hormone by the thyroid gland. Thyrotoxicosis

need not be due to hyperthyroidism. In short in

hyperthyroidism the pathology is in the thyroid

gland itself. The causes for hyperthyroidism and

toxicosis without hyperthyroidism are shown below.

Hyperthyroidism Toxicosis without hyperthyroidism

1. Graves’s disease 1. Subacute thyroiditis*

2. Toxic nodular 2. Ectopic functioning thyroid

 goiter tissue

3. Toxic adenoma 3. Silent thyroiditis

4. Jod-Basedow’s 4. Struma ovarii

 disease 5. Metastatic follicular

 carcinoma (functioning)

 6. Trophoblastic tumors

7. Postpartum thyroiditis

 8. Thyrotoxicosis factitia

Toxic Goiter

27

*Note: In thyroiditis, inflammation ofthyroid causes

release of already formed thyroid hormones into the

circulation,resulting in toxicosis. In other conditions

such as struma ovarii, trophoblastic tumors, etc.,

there is extrathyroid production of thyroxin from

these tissues.

Q 20. What is Graves’ disease?

The essential components of Graves’disease are:

Diffuse goiter

Thyrotoxicosis

Autoimmune manifestations like:

Infiltrative ophthalmopathy

Dermatopathy

– Myopathy.

Q 21. What is the essential etiology of Graves’

disease?

Graves’disease is an autoimmune disorder caused

by thyroid stimulating immunoglobulins (TSIs)

that have been produced against an antigen in the

thyroid. This is directed to the thyroid stimulating

hormone receptors (TSHR - Ab). This acts like TSH

agonist. TSH-Ab is found only in Graves’disease.

Q 22. What are the precipitating factors for

primary thyrotoxicosis?

Remember 3 - S

Sex (puberty, pregnancy)

Sepsis

Psyche (sudden emotional upset).

Q 23. What are the differences between primary

thyrotoxicosis and secondary thyrotoxicosi?

Primary Secondary

1. Etiology—Autoimmune Not autoimmune

2. Enlargement of goiter is diffuse, firm or soft Bosselated or nodular not uniform

3. Onset is abrupt Insidious

4. Hyperthyroidism is usually severe Hyperthyroidism usually mild

5. Cardiac failure is rare Cardiac failureormultiple extrasystole,paroxysmal atrialtachycardia,

paroxysmal atrial fibrillation, or persistent atrial fibrillation

6. Eye signs common Except lid lag and retraction other eye signs are not seen

7. No pre-existing goiter Pre-existing nodular goiter for a long duration

8. Usually younger women Usually middle aged or elderly

9. The entire gland is overactive Internodularthyroid tissue is overactive,rarely one or more nodules

also may be overactive

10. Presence of bruit Bruit need not be present

11.  It is due to abnormal thyroid stimulating

antibodies (TSAb)

No such antibodies (it is due to over activity of nodules)

12.  Can be managed by, drugs, radioiodine,

and surgery

Surgery is the treatment of choice after control of the toxicity

13. Manifestations not due to hyperthyroidism

pretibial myxedema may occur

Not seen

Clinical Surgery Pearls

28

Q 24. How will you confirm your diagnosis of

toxicity?

Confirmation by:

Thyroid Function Test – T3, T4 and TSH

(Immunochemiluminometric assay is the current

method).

Free T3

, T4

 are more significant and meaningful.

The T3

 and T4

 are raised and TSH is lowered in

hyperthyroidism.

Normal values are total Free T3 3.5 – 7.5 mol/L

 Free T4

10–30nmol/L

 TSH 0.3 – 3.3 mU/L

Note: The total T3

 and T4

 hormone level will vary

depending upon the amount of thyroid binding

globulin (TBG).

Q 25. What are the other investigations required?

Antithyroglobulin antibody: More than 1:100.

Thyroid peroxidase (TPO): > 25 units (TPO and

TSH antibodies are increased in autoimmune

thyroiditis).

TSHreceptor antibodies are difficultto estimate.

Radioisotope scintigraphy (radionuclide scan).

Q 26. What is the role of isotope scanning in

thyroid?

The only absolute indication in thyrotoxicosis

for isotope scanning is for the diagnosis of

Autonomous Toxic Nodules.

Toxicity with nodularity is an indication. It can

identify hypofunctioning nodule (cold). Cold

nodule in Graves’ is likely to be malignant.

Itisthe only method bywhich one can definitely

differentiate primary, secondary and toxic

nodules.

Isotope scan can also differentiate hyperthyroidism from toxicosis due to other causes.

(To differentiate hyperthyroid thyrotoxicosis

from non-hyperthyroid thyrotoxicosis). The

radioactive iodine uptake (RAIU) is increased

in hyperthyroidism whereas toxicosis because

of extrathyroidal causes the RAIU is decreased

(e.g. thyroiditis).

Other indications for isotope scan are:

To identify ectopic thyroid tissue.

To identify recurrence and metastasesin thyroid

carcinoma.

Q 27. What is the isotope of choice for diagnostic

scanning of the thyroid?

99mTc is the isotope of choice for diagnostic

purposes. It is cheap and the radiation islessthan

radioiodine. Twenty minutes after intravenous

injectionof 99mTc, scanning is done over the thyroid.

If radioactive iodine is used 123I is the isotope of

choice for diagnostic purposes.

Q 28. What is the half-life of the various radioisotopes used in thyroid?

Isotope Half life Route Rays Comment

123I 13 hours Oral Gamma rays Will not detect nodules < 1 cm size

131I 8 days Oral Gamma and beta rays Too much irradiation if used for diagnostic scanning

132I 2.3 hours Oral Gamma and beta rays Not used for clinical purposes

99Tc 6 hours IV Gamma rays Commonly used for diagnostic scanning of thyroid

Toxic Goiter

29

Q 29. What is the problem with technetium

scanning?

Carcinoma concentratestechnetium and therefore

a hot nodule need not necessarily be benign.

Q 30. What is discordant scan?

A nodule which is warm on technetium scanning

and cold on radioiodine scanning is called

discordant scan. This is suggestive of malignancy.

Q 31. Why technetium is preferred over radioiodine for diagnostic scanning?

It gives small amount of radiation and you get the

image within minutes.

Q 32. What will be the appearance in scintigraphy

in primary, secondary and toxic nodule?

Primary :Uniformdiffuseincreaseduptake(Fig.1.1).

Secondary: Heterogeneous pattern with some

focal areas of enhanced uptake (Fig. 1.2).

Toxic nodule: Increased uptake only in the

nodule, with no uptake in the surrounding

thyroid tissue (Fig. 1.3).

Q 33. What are the toxic situations where there

is decreased uptake of isotope in thyroid gland?

Low uptake is seen in:

Thyroiditis

Postpartum thyrotoxicosis

Struma ovarii

Factitious thyrotoxicosis

Jod-Basedow thyrotoxicosis.

Fig. 1.3: Toxic nodule

Fig. 1.1: Primary toxicosis

Fig. 1.2: Secondary toxicosis

Clinical Surgery Pearls

30

Q 34. What are the features of toxic adenoma?

Toxic Adenoma

These are: Benign, monoclonal thyroid tumors

of more than 3 cm size

Are autonomousratherthan responding toTSH

stimulation

Eye signs and other stigmata of Graves’ are

absent

Somatic mutation of TSH receptor gene or G

protein gene is present

T4 may be normal (hence check T3

 levels).

Q 35. What are the conditions in which Thyroid

Binding Globulins (TBG) are increased?

The concentrations of TBG are increased in

pregnancy, liver diseases and where there is

hyperestrogenism. The levels of free T3

 and T4

 in

these conditions are normal despite high TBG.

Q 36. What are the conditions in which the TBG

levels are decreased?

High androgens, severe hypoproteinemia, chronic

liver disease and acromegaly.

Q 37. What is the problem with the measurement

of free T3

 and T4

?

The method usually used is radioimmunoassay

and it is costly.

Q 38. What is the normal free T3

 and T4

 value?

Free T3

—3.5 to 7.5 pmol/L

Free T4

—10 – 30 nmol/L

It isto be noted that 0.3% of the total T3 and 0.03%

of the total T4

 are free and physiologically active.

Q 39. What is T3

 Thyrotoxicosis?

T3

 alone is raised and TSH is decreased in this condition.

Q 40. What is subclinical hyperthyroidism?

Seen in 1% of hyperthyroids

Serum TSH is low but the free T4

 is normal

Symptoms are absent and hence called ‘subclinical’.

Q 41. What is apathetic hyperthyroidism

(masked)? (PG)

Apathetic hyperthyroidism

Lack almost all of the clinical manifestations

Presents as behavior problems

May end up at the psychiatrist’s

Thyroid gland is not usually enlarged

Commonly seen in elderly patients

Decreased appetite and lethargy

Newonset of atrialfibrillationandincreasedangina.

Q 42. Is there any role for FNAC in thyrotoxicosis?

Yes. Sometimes the thyrotoxicosis may be associated

with a papillary carcinoma ofthe thyroid. Itis better

to do after controlling toxicosis because of the

increased vascularity of the gland.

Q 43. How you will manage thyrotoxicosis?

In primary thyrotoxicosis we have 3 options.

1. Antithyroid drugs

2. Radioiodine therapy

3. Surgery.

Q 44. What will be the choice of therapeutic agent

in thyrotoxicosis?

We have some broad guidelines. This must be

modified according to the facilities available and

wishes of the patient.

Age over 25 years Radioiodine therapy (when

 development is complete)

Under 25 years – Surgery for large goiter

 – Antithyroid drugs for the small

 goiter

Toxic nodular goiter usually will not respond

very well to radioiodine and antithyroid drugs.

Therefore, surgery is the treatment of choice.

Toxic Goiter

31

Q 45. What are the drugs available for the

treatment?

Drugs available for the treatment of thyrotoxicosis

a. Thionamides

Carbimazole (Neomercazole)

Dose 40 – 60 mg daily for first 3 weeks,

20 – 40 mg daily for 4 – 8 weeks,

Maintenance of 5 – 20 mg/daily for

 18 – 24 months (each tablet is 5 mg)

Propylthiouracil (PTU)

Methimazole 20 – 30 mg daily (single dose)

b. Beta blockers

c. Potassium per chlorate inhibits iodide transport

d. Lugol’s iodine

e. Iopanoic acid—500 mg bid

Severe cases unresponsive to conventional therapy

f. Lithium carbonate – 300 mg 6th hourly

g. Guanethidine 30 – 40 mg oral 6th hourly

h. Reserpine 2.5 – 5 mg IM 4th hourly

i. Glucocorticoids: dexamethasone

2 mg oral 6th hourly.

Q 46. What is the dose of Propylthiouracil?

About 100 to 300 mg 3 times daily orally initially

for 4 to 6 weeks followed by 100 mg 3 times daily.

Q 47. What is the action of Propylthiouracil?

a. PTU blocks conversion of T4

 to T3

 in periphery

(liver)

b. Inhibits iodine organification and coupling of

iodotyrosines

c. Immunomodulatory effectsthatreducesthyroid

stimulating antibodies.

Q 48. What are the advantages of PTU?

a. PTU may be given during pregnancy at reduced

doses.

b. Ifthyroidectomy isrequired in second trimester,

the patient can be prepared with PTU.

c. Useful for the treatment of thyroid storm

(multiple doses needed).

Q 49. What are the adverse effects of PTU? (PG)

Adverse effects of PTU

Hepatotoxicity which is not dose related

Mild transaminase elevation in 30%

Agranulocytosis

Minor side effects as seen in carbimazole therapy

Antineutrophilic cytoplasmic antibody (ANCA) in

20% especially with long-term treatment.

Q 50. What is the action of carbimazole?

Carbimazole acts by the following methods:

a. Blockage of organic binding and oxidation of

iodine

b. Immunosuppression (decreasesthyroidantigen,

prostaglandin and cytokine release)

c. Reduction of thyroid autoantibody titers.

Q 51. What are the side effects of carbimazole?

Side effects of carbimazole

Fever,rash,urticariaandarthralgia(minor side effects)

Liver dysfunction

Neuritis

Myalgia

Lymphadenopathy

Psychosis

Occasional agranulocytosis (< 1 in 200 cases).

Q 52. What is the clinical manifestation of agranulocytosis?

Agranulocytosis presents as sore throat, which

warrants immediate cessation of the drug.

Q 53. Can the thyroid be enlarged during medical

treatment?

Yes.Duringtreatmentin1/3rdtohalfofthepatients,

the thyroid willshrink. Enlargement usually occurs

Clinical Surgery Pearls

32

because of commencement of hypothyroidism,

which should be avoided.

Q 54. What is “block and replacement” regime?

The thyroid enlargement because of the development

of hypothyroidism during medical treatment

is prevented by supplementing low dose of

levothyroxine (0.1 mg) along with the antithyroid

drugs.

Q 55. In what percentage of patients is medical

treatment effective?

Permanent remission is possible only in a small

minority of adults and 20% of children.

Q 56. What is the dose of beta blocker?

Propranolol is the drug of choice for initial control

of adrenergic symptoms.

The dose is 20 – 80 mg every 6 – 8 hours orally.

1 – 2 mg IV propranolol for thyroid storm.

Q 57. What is the action of propranolol?

a. Peripheral conversion of T4

 – T3 is blocked

b. Adrenergic antagonistic action helps to alleviate

cardiac symptoms, tremor, etc.

Contraindications for propranolol

Bronchial asthma

COPD

Heart block

CCF.

Q 58. Is propranolol indicated in all patients with

toxicity?

No;

It is given for emergency surgical management

of toxicity

It is also used for control of the adrenergic

symptoms.

Q 59. If the patient was prepared using propranolol before thyroid surgery, how long should it

be continued postoperatively?

Propranololshouldbe given over a periodof 1week

and preferably tapered over a period of 2 weeks

after surgery.

Q 60. What are the drugs inhibiting peripheral

conversion of T4

-T3

? (PG)

Drugs inhibiting peripheral conversion of T4

-T3

Beta blockers

PTU

Glucocorticoids

Iopanoic acid.

Q 61. What is the minimum duration of medical

treatment required before surgery?

Thyroidectomy performed immediately after control

of thyrotoxicosis is associated with risk of thyroid

crisis and it is preferable to wait approximately two

months until after a patient is euthyroid.

Q 62. Is there any role for Dexamethasone in the

management of thyrotoxicosis? (PG)

Itis used forthe management ofthyrotoxic crisis

Dose is 2 mg every 6th hourly (injection)

The actions are:

a. Inhibits glandular secretion of hormone

b. Inhibits peripheral conversion of T4

 to T3

c. Immunosuppression.

Q 63. What is Lugol’s iodine and what is its dose?

Five percent iodine in 10% potassium iodide is

called Lugol’s iodine.

The dose is 10 drops in a glass of water 3 times

daily for 10 days.

Toxic Goiter

33

Q 64. What are the actions of Lugol’s iodine?

a. Decreases the vascularity of the gland

b. Makes the thyroid firm and less friable (helps in

surgical removal)

c. Prevents the release of hormone from the

gland—thyroid constipation.

Q 65. What will happen if Lugol’s iodine is given

for more than 10 days?

After 2 weeks the effect of Lugol’s iodine therapy

is lost due to the so called thyroid escape from

iodine control.

Q 66. What are the indications for radioiodine

therapy?

Radioiodine (

131I) is usually given to patients

above 45 years for primary thyrotoxicosis.

Isotope facility must be available.

Q 67. What are the problems of radioiodine

therapy? (PG)

Problems of radioiodine therapy

a. Indefinite follow-up is essential as the patient may

develop hypothyroidism (75%)

b. Chance of permanent thyroid failure – 90% (hypo

is more because of failure of cellular reproduction)

c. Theoreticalpossibility ofgeneticdamage, leukemia,

damage to fetus and carcinoma (no convincing

evidence)

d. Takes 2 – 3 months for control of symptoms

e. Worsening of ophthalmopathy (especially in

smokers) and dermatopathy

f. Mild anterior neck pain

g. Increased risk of benign tumors

h. Malignant transformation in young patients

i. May induced hyperparathyroidism.

Q 68. What are the contraindications of radioiodine therapy?

Contraindications of radioiodine therapy

Pregnancy

Lactating mothers

Women desiring pregnancy within 1 year

Children/adolescents (relative).

Q 69. What is the dose of radioiodine? (PG)

300 to 600 MBq, if there is no clinical improvement

after 12 weeks further dose is given. Two or more

doses are necessary in 20 to 30% of cases.

Q 70. What is the method of radioiodine treatment

for toxicity? (PG)

a. Make the patient euthyroid with drugs

b. Discontinue drugs for 5 days

c. Administer 131I 300–600 MBq (5–10 mCi)

d. Start antithyroiddrugs after 1week andcontinue

for 6 to 8 weeks

e. After 12 weeks, if there is no improvement, give

another dose of radioiodine

f. Twoormoredosesofradioiodinemayberequired.

Q 71. What are the indications for surgery in thyrotoxicosis? (PG)

a. Intolerance or non-compliance with antithyroid

drugs

b. Contraindications to radioiodine therapy

c. Graves’disease in children, adolescents and those

who are under the age of 25 years

d. In women who are potential mothers

e. Large goiter

f. Persistent thyromegaly

Contd...

Clinical Surgery Pearls

34

g. If antithyroid medication isrequired for more than

2 years

h. Graves’with nodules

i. Ophthalmopathy

j. Pressure symptoms

k. Toxic MNG

l. Substernal goiter

m. Amiodarone induced thyrotoxicosis.

Q 72. What are the drugs used for preparation of

a patient for “urgent thyroidectomy”? (PG)

Combination of oral iopanoic acid 500 mg bid and

dexamethasone 1 mg bid and PTUor MMI and beta

blockers for 5 to 7 days.

Q 73. What are the advantages of surgery? (PG)

Advantages of surgery

a. Surgery is effective in achieving euthyroid status in

95–97% of patients

b. Controls hyperthyroidism immediately

c. Hazards associated with radioiodine therapy are

avoided

d. Surgery will provide tissue for histology

e. Surgery will remove occult foci of malignancy

f. Childbearing is immediately possible

g. Coexisting parathyroid carcinoma can be removed

h. Isabettertreatmentfortoxicity with ophthalmopathy

i. No need for follow up because nodules are not left

behind.

Q 74. What is the surgical treatment of primary

thyrotoxicosis?

Near total thyroidectomy is now recommended as

the treatment of choice.

Q 75. What is the recommended treatment for

secondary thyrotoxicosis?

Surgery is preferred over radioiodine for secondary

thyrotoxicosis because:

It will not respond to radioiodine as most of the

nodules may not take up radioiodine.

Large and repeated doses ofradioiodine may be

required.

Q 76. What is the recommended treatment of

toxic nodule?

Once the patient is made euthyroid, surgery in the

form of Hemithyroidectomy will give permanent

relief.

Q 77. What is subtotal (bilateral) thyroidectomy?

Two grams ofthyroid remnantis kept on both sides

and the rest of thyroid gland is removed in subtotal

thyroidectomy.

Q 78. What is Hartley-Dunhill procedure?

Total lobectomy and isthmectomy on the affected

side and 4 g remnant left on the contralateral

(normal) side. This form of surgery is recommended

by some authorities for the surgical management

of toxic goiter.

Operation Part of thyroid removed Indications

Lobectomy Removal of one lobe of thyroid Solitary thyroid nodule

Hemithyroidectomy Removal of one lobe and isthmus • STN

• Toxic nodule

• Follicular neoplasm

Contd...

Contd...

Toxic Goiter

35

Bilateral subtotal thyroidectomy (Fig.

1.4)

2 g ofthyroid remnantis kept onboth

sides and the rest of the thyroid gland

is removed

• Toxic goiter

• Toxic nodular goiter

• Multi nodular goiter

(Near total is the preferred

treatment for these conditions now)

Hartley-Dunhill procedure (Fig. 1.5) Total lobectomy and isthmectomy on

the affected side and 4 g remnant left

on the contralateral side

Toxic goiter

Near total thyroidectomy 1 to 2 g remnant is left on the

contralateral side of the lesion and

the rest of the thyroid is removed

• Toxic goiter

• MNG

• Papillary carcinoma

• Follicular carcinoma

• Medullary carcinoma, etc.

Total thyroidectomy isthmusectomy/

isthmectomy

Entire gland is removed

Removal of isthmus alone

Thyroid malignancy

•  Emergency decompression of

trachea for anaplastic carcinoma

• Biopsy for anaplastic carcinoma

Contd...

Fig. 1.4: Subtotal thyroidectomy Fig. 1.5: Hartley-Dunhill procedure

Clinical Surgery Pearls

36

Q 79. What is the management of intrathoracic

goiter with toxicity? (PG)

Antithyroid drugs will increase the size of

the retrosternal and intrathoracic goiter and

therefore dosage should be carefully adjusted.

Su rge ry is the treatment of choice for

intrathoracic goiter with toxicity.

Q 80. What is the management of intrathoracic

toxic goiter in pregnancy? (PG)

Antithyroid drugs are given in small doses in

first trimester to prevent fetal goiter and airway

obstruction

Antithyroid drugs may be combined with

propranolol and surgery is done in the second

trimester.

Q 81. What is the treatment for recurrent toxicity?

 (PG)

Radioiodine/medical treatment.

Q 82. What are the preoperative preparations for

thyroid surgery?

Thyroidectomy is done only after making the

patient euthyroid

Lugol’s iodine is given for a period of 10 days for

reducing the vascularity and making the gland firm

(for toxic cases only)

Always send the patient for preoperative indirect

laryngoscopy (or video laryngoscopy) to rule out

occult vocal cord palsy and document for medicolegal purposes

Arrange cross matched blood for vascular thyroids

Assessment of the cardiac status by ECG

Assessment of the chest by an X-ray chest and

X-ray neck AP view (for detecting tracheal

displacement) and lateral view (for detecting

luminal narrowing). X-ray neck will also reveal soft

tissue shadows and calcifications. X-ray chest may

reveal the presence of retrosternal extension

Rule out hypertension and diabetes mellitus.

Q 83. What are the preliminary steps of thyroid

surgery?

Surgery is done under general anesthesia using

endotracheal intubation (preoperative X-ray

neck to rule out displacement ofthe trachea and

luminal narrowing).

Position of the patient: Supine with sand bag

behindthe shoulders andheadringfor adequate

neck extension.

Skin is painted with antiseptics and proper

draping with sterile towels and head cover.

Kocher’s collar incision is used for incising the

skin (1 to 2 cm above the manubrium sternum).

Incision is deepened up to the deep fascia by

incising the platysma.

Anterior jugular veins seen on the surface of the

investing layer, may be ligated (communicating

vein seen connecting the two anterior jugulars

requires ligation).

Investing layer is opened vertically in the midline.

The strap muscles are retracted laterally (in big

thyroids, they may be divided either on one side

or bilaterally in the upper part to save the Ansa

hypoglossi nerves entering the strap muscles in

the lower part)

ThePretracheal fascia is incised vertically and the

thyroid gland is mobilized by ligating the middle

thyroid vein (this is the first vessel to be ligated).

Q 84. What are the essential steps of thyroidectomy?

A. Identification of parathyroid glands:

Identify the parathyroids before ligating the

vessels.

Parathyroids have the color of peanut butter,

each of 6 × 3 × 3 mm size and lessthan 40 mg

weight.

The recurrent laryngeal nerve (RLN)—

Inferior thyroid artery junction is critical in

Toxic Goiter

37

identifying the parathyroids (this is a critical

area of RLN injury as well).

The superior parathyroid glands are above

and behind this junction.

The inferior parathyroid glands are below

and anterior to this junction having variable

positions. The inferior glands may be situated

on the inferior pole of the thyroid, thyrothymic

ligament, in the thymus or in the perithymic fat.

B. Identification of the recurrent laryngeal nerve:

Itis preferable to identify the entire course of

the nerve in thyroid surgery.

There is no role for the old axiom “nerve seen

is nerve injured”, which is called the ‘ostrich

philosophy’.

The first identification is at the so-called

Riddle’s triangle, which is bound by inferior

thyroid artery above, the carotid artery

laterally and the trachea medially.

From there, the nerve is traced upwards to

the point of its entry into the larynx at the

greater cornu of the thyroid cartilage.

Before entry, the nerve may divide into two

or more extralaryngeal branches.

When there isdifficulty in identifyingthe RLN,

the entry point of the nerve can be located

by identifying Zuckerkandl’s tubercle.

The nerve may cross the inferior thyroid

artery usually deep to the artery, sometimes

superficial or may even passthrough the fork

of the branches of the artery.

C. Ligation of vessels:

The branches ofthe superior thyroid vessels

are individually skeletonized and ligated

as close to the superior pole as possible

after identifying the external branch of the

superior laryngeal nerve.

A medial approach to the superior pole via

the avascular space between the cricothyroid

muscle and the upper pole of the gland is ideal.

“Mass Ligation” of the superior pole is to be

avoided.

Capsular ligation of the inferior thyroid artery:

a. Inferior thyroid artery is an end artery to

the parathyroids and hence ligation of the

trunk of the inferior thyroid artery is not

recommended.

b. The small branches entering the capsule

of the gland alone are ligated, thereby

preserving the blood supply to the

parathyroids.

c. These small branches are therefore situated

between the parathyroid and the thyroid

gland

Finally,the lower pole veins are ligated.

Blind“mass ligation”of the lower pole

may injure the RLN in this situation.

D. Removal of the gland:

The gland is nowfree forremoval.Depending

on the type of surgery, the extent of removal

and the amount of remainder may be

decided (see box on various operations of

thyroid gland in page 38).

Beware ofthe suspensory ligament of Berry,

which is the last attachment of the gland to

the cricoid cartilage.

The RLN may pass through the substance,

superficial or deep to the Berry’s ligament.

Take care of the nerve before the final

removal of the gland.

Q 85. What are the critical areas of recurrent laryngeal nerve injury? (PG)

There are 3 critical areas of RLN injury:

At the site of where the inferior thyroid artery

crosses the RLN.

Clinical Surgery Pearls

38

Atthe region ofthe suspensory ligament of Berry.

At the lower pole of the gland during ‘mass

ligation’ of the vessels of the inferior pole,

especially on the right side (Fig. 1.6).

Q 86. What is Zuckerkandl’s tubercle? PG)

It is the posterior extension of the lateral lobes

of the thyroid gland near the ligament of Berry.

It is found in 14–55% of cases.

Fig. 1.6: Recurrent laryngeal nerve injury—critical sites

Toxic Goiter

39

The RLN runs cranially in a fissure between the

Zuckerkandl’stubercle and the main body ofthe

gland or the tracheal surface.

The RLN may have a sharp angle beneath the

tubercle.

Thenerve issoconstantly relatedtothistubercle,

that it is often called “an arrow pointing to the

RLN.”

Q 87. What is non-recurrent laryngeal nerve?

Thisisseen on the rightside as a result of failure

of development of the 4th aortic arch.

The RLN here may arise as a branch from the

vagus at the level of the inferior horn of the

thyroid cartilage (instead of going down, curving

around the subclavian artery and coming up).

The incidence is 0.2 – 0.5%.

Q 88. What is suspensory ligament of Berry?

Itisnothingbutacondensationofthepretracheal

fascia attaching the thyroid gland to the cricoid.

The two ligaments on either side form a sling

anchoring the gland to the larynx.

It must be severed before the gland can be

removed.

The RLN is in immediate contact with the back

of the ligament.

Q 89. What are the complications of thyroidectomy?

Complications of thyroidectomy can be classified as

A. Metabolic

1. Hypoparathyroidism

Temporary Hypoparathyroidism

Temporary hypocalcemia without hypoparathyroidism (‘hungry bone’ syndrome)

Permanent hypoparathyroidism (ischemia

and removal of the gland )

Spurioushypoparathyroidism(total calcium

is decreased but ionized calcium is normal)

2. Thyroid storm (thyrotoxic crisis)

3. Hypothyroidism (20 – 40%)

B. Non metabolic

1. Neural:

RLN injuries

External laryngeal nerve injuries

2. Nonneural:

Hemorrhage

Hematoma

Stridor and airway obstruction

Skin flap necrosis

Compromise of tracheoesophageal blood

supply

Horner’ssyndrome(sympatheticnerveinjury)

Chylous fistula (extremely rare).

Q 90. What are the two most important complications of thyroidectomy?

1. Hypoparathyroidism

2. Recurrent laryngeal nerve injury.

Note: The incidence of both should be lessthan 1%

for an experienced thyroid surgeon.

Q 91. What are the clinical manifestations of

hypoparathyroidism?

Clinical manifestations of hypoparathyroidism

Acral paresthesia

Circumoral tingling numbness and paresthesia

Anxiety

Carpopedal spasm

Laryngeal stridor

Spasm of respiratory muscles: dread ofsuffocation

Convulsions in later stages

Blurring of vision due to spasm of intraocular

muscles

Prolongedunrectifiedtetanycangiverisetocataracts.

Note: These clinical manifestations can occur on

Contd... the same day of surgery or may be noticed later on.

Contd...

Clinical Surgery Pearls

40

Q 92. What will be the biochemical finding in

hypoparathyroidism?

Decrease in calcium and increase in phosphorus

is the feature of hypoparathyroidism.

Decreaseincalciumwithadecreaseinphosphorus

is suggestive of “hungry bone” syndrome.

Q 93. What is Chvostek sign?

With a knee hammer, gently tap the facial nerve as it

courses in front of the external auditory meatus. When

tetany (as a result of hyperparathyroidism) exists, the

tappingofthehyperexcitablenerveprovokes abrisk

muscular twitch on the same side of the face.

Q 94. What is Trousseau’s sign?

A sphygmomanometer cuff is placed around the

arm and the pressure is raised to 200 mm of Hg. If

tetany is present, in 5 minutes, typical contractions

of the hand are seen: the fingers are extended

except at the metacarpophalangeal joints and the

thumb is strongly adducted, the combined effect

of which is to produce the so-called obstetrician’s

hand (accoucheur’s hand).

Note: The migratory superficial thrombophlebitis

is also called Trousseau’s sign (seen in visceral

malignancies and TAO).

Q 95. What is carpopedal spasm?

Strong adduction of the thumb is always present

in tetany and this when coupled with extension of

the feet is called “Carpopedal spasm”.

Q 96. What is spurious hypocalcemia? (PG)

Decrease in total calcium, albumin and

hematocrit are seen in first two postoperative

days following any surgery including nonthyroid.

This is as a result of antidiuretic hormone

release from general stress of the surgery and

the consequentwater retention by kidneys and

hemodilution.

The albumin-bound total calcium is decreased

as a result of this.

Thenonprotein bound free (ionized) calcium will

be normal in this situation and therefore estimation

of the free calcium is important to distinguish

spurious hypocalcemia from true hypocalcemia.

Q 97. How can you prevent hypoparathyroidism?

It is prevented by taking the following precautions

during surgery:

Identification of parathyroids preoperatively

Protection ofblood supply toparathyroid glands

Capsular ligation of inferior thyroid vessels

Autotransplantation of parathyroidsin an event

of inadvertent removal.

Q 98. What is the treatment of hypoparathyroidism? (PG)

A. Emergency: 20 mL of 10% calcium gluconate IV

in 100 mL of D5W over 10 – 15 minutes

B. Chronic Patient:

Calcium alone is not enough for the

management of hypoparathyroidism.

Calcium carbonate 500 mg oral tablets (1.5

to 2.5 g per day).

Vitamin D (ergocalciferol) 50 – 100,000 units

per day or dihydrotachysterol (DHT) 250 –

1000 micrograms daily.

Magnesiumparenteral IV/IM0.5g(4mEq) and

2 to 4 mEq per Kg body weight over 3–5 days.

Magnesium gluconate oral 500 mg tablets.

Note: If true hypocalcemia is identified, it is better

to treat it pre-emptively rather than to wait for the

symptoms.

Q 99. Can the patient get recurrent thyrotoxicosis

after surgery? (PG)

Yes, in 5% of cases. Cure is possible if the thyroid

tissue can be reduced below a critical mass. This will

Toxic Goiter

41

result in reduction of TSAb. When the mass of

thyroid tissue is small it can produce only limited

hypertrophy and hyperplasia even if the circulating

TSAb is high.

Q 100. What are the causes for stridor postoperatively?

Causes of post-thyroidectomy stridor

a. Hematoma (rule out hematoma first by change of

dressing)

b. Laryngeal edema—three causes

1. Edema because of intubation

2. Edema because of hematoma

3. Edema because of hypothyroidism as a result of

aggressive antithyroid drug therapy

c. Recurrent laryngeal nerve injury.

Q 101. What are the clinical manifestations of

RLN injury?

The most common manifestation is hoarseness.

The other manifestations are:

Dysphonia

Paralytic aphonia

Periodic aspiration

Ineffective cough.

Note:

Unilateral RLN palsy is well-compensated

normally

Normal voice does not mean that the nerve is

intact

All hoarseness are not because of nerve injury

either.

Q 102. What is the indirect laryngoscopy finding

in unilateral RLN palsy? (PG)

Paramedian position of the paralyzed cord

Hyperadduction of the normal cord during

phonation as compensation.

Q 103. Is there any role for routine post-operative

indirect laryngoscopy? (PG)

Early routine indirect laryngoscopy is done on

fourth or fifth day.

Cord paralysisis more common than supposed.

Asymptomatic bilateral vocal cord paralysis is

possible.

Q 104. What is the treatment of unilateral RLN

palsy? (PG)

Symptomaticimprovementisseenwithin6weeks.

Perioperative steroids are given to reduce the

incidence oftemporary RLNpalsy resulting from

edema or contusion.

Steroids should be started within 7 days of

surgery.

Prednisolone 15 mg tid for 10 days is given.

Gradually taper the dose to zero over the next

10 days.

If there is not any recovery within 6 months,

degeneration is to be suspected (recovery may

be delayed for 6 – 12 months; no regeneration

after 18 months).

Speech therapy is instituted if there is no recovery.

Medialization of the cord by Teflon injection or

some other technique.

Q 105. What are the clinical presentations and

treatment of bilateral RLN palsy? (PG)

This will cause paralysis in adduction

Clinicallythismaygoundetectedforlongperiods.

Patientmaytolerateminimalairwayformanyyears.

May present as inspiratory stridor, dyspnea, or

minimal dysphonia.

Emergency endotracheal intubation may be

required.

It is better to do tracheostomy and wait for 1

year (valved tracheostomy tubes are available).

Arytenoidopexy, cordectomy or endoscopic

laser treatment is done after 18 months.

Clinical Surgery Pearls

42

Q 106. What is the innervation of external

laryngeal nerve (a branch of superior laryngeal

nerve)?

The nerve supplies the cricothyroid muscle, which

is a tensor of the vocal cord.

Q 107. What is the clinical presentation of

unilateral external laryngeal nerve injury?

Clinical presentation:

Loss of high pitch for the voice

Voice fatigue

Breathy voice

Frequent throat clearing.

Indirect laryngoscopy will reveal:

Shorter and hyperemic vocal cord

The affected vocal cord will be at a lower level

The glottic chink is oblique (rotation of the

posterior commissure to the paralyzed side).

Q 108. What is thyrotoxic crisis (thyroid storm)?

It is a sudden life-threatening exacerbation

of thyrotoxicosis seen in 1 to 2% of patients.

This is a syndrome manifested by high grade

fever, sweating, tachypnea, hyperventilation,

tachycardia, palpitation, restlessness, tremor,

psychosis, delirium, diarrhea, dehydration, nausea,

vomiting, hypotension and end-stage coma. The

causes for crisis are:

a. Inadequate preparation prior to surgery

b. Infection in thyrotoxicosis

c. Trauma in thyrotoxicosis

d. Pre-eclampsia

e. Diabetic ketoacidosis

f. Surgical emergency

g. Emotional stress

h. Vigorous palpation of the gland.

Q 109. What is Bayley’s symptom complex?

Bayley’s symptom complex of thyroid storm

a. Insomnia

b. Anorexia

c. Vomiting

d. Diarrhea

e. Diaphoresis

f. Emotional instability

g. Temperature > 38°C

h. Tachycardia

i. Accentuated symptoms and signs of toxicosis

j. System dysfunction.

Q 110. What is the treatment of thyroid storm?

Treatment of thyroid storm

1. To control fever:

Acetaminophen is used

Aspirin is never used as it can elevate the free

thyroid hormones

Tepid sponging

Cooling blankets

2. To correct dehydration and electrolyte imbalance:

IV fluids

3. To control the heart rate:

a. Propranalol 1–2 mg IV 6th hourly (40–80 mg

QID) orally)

b. Esmolol 250 – 500 microgram per Kg body

weight loading and 50 microgram/Kg/minute

maintenance

4. To inhibit hormone release:

a. Logols’ iodine 10 drops 3 times

b. Sodium iodide 1gm IV over 24 hours

c. Super saturated potassium iodide (SSKI) 10

drops twice daily

5. To inhibit new hormone synthesis:

a. Propylthiouracil (50–200 mg)

b. Carbimazole (20 mg every 4 hours)

Contd...

Toxic Goiter

43

In case of adverse reaction to PTU or carbimazole, use

lithium carbonate 300 mg every 6th hours

6. To reduce systemic symptoms:

a. Hydrocortisone 100mg IV 6th hourly

b. Dexamethasone injection 6–8 mg IV or 2 mg

orally 6th hourly

7. Treatment of CCF

8. Antibiotic coverage for infection

9. Sedation

10. Dialysis if required.

Q 111. What are the methods available to remove

T3

 and T4

 from serum? (PG)

a. Oral cholestyramine

b. Peritoneal dialysis

c. Hemoperfusion.

Q 112. What is the treatment of thyrotoxicosis in

pregnancy? (PG)

Radioiodine is absolutely contraindicated

because of the risk to the fetus.

The antithyroid drugs and TSH cross the

placenta and therefore the baby is born goitrous

and hypothyroid.

Low dose antithyroid drugs, preferably PTU

is the ideal treatment (to keep the free T4

 of

pregnant women in the high normal.

Avoid methimazole: Associated with cutis

aplasia, and esophageal and choanal atresia.

The danger of surgery is miscarriage.

Surgery can be carried out in the second

trimester.

Q 113. What is the treatment of hyperthyroidism

during lactation? (PG)

Thionamides are secreted in breast milk and this

was once considered a contraindication.

PTU at a dose of 750 mg is safe.

Q 114. What is postpartum hyperthyroidism?(PG)

Pregnancy will lead onto exacerbation of autoimmune diseases. This may occur with previously

diagnosed or undiagnosed hyperthyroidism. There

is a strong association with HLA-DR3 and HLA-DR5

haplotypes.

Q 115. What is thyrotoxicosis factitia?

Thisis usually seen in health cranks as a result of oral

intake ofthyroxine usually taken to reduce weight).

Q 116. What is Jod-Basedow thyrotoxicosis?

Large doses of iodide given to hyperplastic

endemic goiter which is iodine avid may produce

temporary hyperthyroidism. This is Jod-Basedow

thyrotoxicosis.

Q 117. What is neonatal thyrotoxicosis?

This is seen in babies born to hyperthyroid

mothers. TSAb can cross the placental barrier. The

hyperthyroidismgraduallysubsidesafter3to4weeks.

Q 118. What is thyrocardiac?

Severe cardiac damage wholly or partly due

to hyperthyroidism. This is usually because of

secondary thyrotoxicosis and is mild. This must

be rapidly controlled with propranolol to prevent

further cardiac damage.

Q 119. What is struma ovarii?

Teratoma of the ovaries may differentiate into

thyroid tissue. This thyroid tissue becomes

hyperactive resulting in mild thyrotoxicosis. T3

 and

T4 are raised with suppressed TSH. Radioactive

iodine uptake (RAIU) in neck is suppressed and

higher intake is seen in the pelvis.

Q 120. What is Hashitoxicosis? (PG)

This is because of painless thyroiditis.

Thisis an early stage of autoimmune thyroiditis.

FNAC picture is that of Hashimoto’s thyroiditis.

Contd...

Clinical Surgery Pearls

44

Thyrotoxicosis in this situation is mild.

Glandularenlargementisseenonlyin60%ofcases.

The inflamed gland releases the already

formed thyroid hormones into the bloodstream

resulting in toxicosis.

Q 121. What is trophoblastic thyrotoxicosis?(PG)

The hCGfrom hydatidiform mole, choriocarcinoma

and metastatic embryonal carcinoma exhibit cross

specificity to thyroid stimulating hormone receptor

(TSHR). This results in thyroid overactivity.

Q 122. What is the histopathological appearance

of hyperthyroidism?

a. There is hyperplasia of the acini lined by high

columnar epithelium.

b. The acini are empty and some of them contain

vacuolated colloid (Scalloping).

c. Pseudopapillary formation is seen.

Q 123. What will happen to exophthalmos after

surgery or radioiodine therapy?

Both will worsen ophthalmopathy. Thionamides

will alleviate the eye problem through immunosuppression.

Q 124. What is the management of exophthalmos

(Thyroid Associated Ophthalmopathy)?

Symptom control:

a. Sleeping with head end elevation.

b. 1% methylcellulose eyedropstoprevent corneal

ulceration.

c. HighdoseprednisoloneorallyorhydrocortisoneIV.

d. Collimated super voltage radiation to retro

orbital space (needs expertise).

Note: Radioiodine is avoided in ophthalmopathy.

Q 125. Is there any role for surgery in exophthalmos?

Surgical removal of lateral wall or roof of orbit is done

for decompression when optic nerve is in danger.

For PG’s—What is New?

1. Thyrotoxic periodic paralysis

It is common in Asian population with thyrotoxicosis. It is seen in 1.9% of hyperthyroidism. It is usually seen in

the third decade. It is a condition where there is weakness of the proximal musclesin the form of mild weakness

to generalized flaccid paralysis with loss of deep tendon reflexes. It is precipitated by trauma, exposure to cold

and after excessive ingestion of carbohydrate. Attacks are usually seen in night after a carbohydrate rich food.

During attack the serum potassium will be in the range of 2.2 to 3.2 mEq. It is a metabolic abnormality of the cell

membrane in hyperthyroid state with resultant shift of potassium to the intracellular position. There is associated

decreased serum phosphate and magnesium.The treatmentis by administering potassium. Propranolol is given

topreventperiodicparalysis. Patientisgivenlowcarbohydratediet. Ifpatientrequiresdiuretics,potassiumsparing

diuretics are given. The EMG will show myopathic pattern during attack. The ECG will show sinus tachycardia,

increased PR interval and prolonged QTu interval.

2. Severe thyrotoxicosis—T4 more than 21 mu/dL.

2 Solitary Thyroid

Nodule (STN-Nontoxic)

Case

Case Capsule

A 40-year old male patient presents with a swelling

in the lower part of right side of the neck. There

are no symptoms of toxicity or hypothyroidism.

On examination, the patient has a pulse rate of

70/minute. There is no tremor. The swelling is

seen on the right side of the neck and moves

with deglutition but without any movement

with protrusion of the tongue. The swelling is

of about 4 × 3 cm size, firm in consistency. The

surface of the swelling is smooth. There is no fixity

of the swelling. The lower limit of the swelling

is visible and palpable. The trachea is shifted to

the left side. The left lobe of the thyroid is not

palpable. The carotids are normally felt. There are

no regional nodes. There is no clinical evidence of

toxicity. Examination of the oral cavity is normal.

Examination of the skull is normal.

Clinical Surgery Pearls

46

Q 1. What are your points in favor of solitary

thyroid nodule?

Points in favor of Solitary Thyroid Nodule (STN):

a. There is only a single palpable thyroid nodule.

b. The rest of the gland is impalpable.

Q 2. What is the difference between a dominant

nodule and solitary thyroid nodule?

A discrete swelling in a gland with clinical

evidence of abnormality in the form of palpable

contralateral lobe or generalized mild nodularity

is called dominant nodule. As mentioned earlier,

a discrete swelling in an otherwise impalpable

gland is called isolated or STN. About 70% of the

discrete thyroid swellings are STN and 30% are

dominant nodules.

Q 3. What percentage of the so-called STN will

ultimately become a dominant nodule of nodular

goiter?

The true incidence of STN after USG/surgical

exploration comes down to only 50%.

Q 4. What is the importance of STN?

The importance of STN lies in the risk of malignancy

compared with other thyroid swellings.

Q 5. What percentages of STN prove to be malignant?

Twenty percent of STN prove to be malignant.

Q 6. What are the differential diagnoses of STN?

Differential diagnoses of STN

a. Nodular goiter with dominant nodule – 50%

b. Adenoma – 20%

c. Cancer – 20%

d. Thyroiditis – 5%

e. Cyst – 5%

Q 7. What is the most important single investigation for diagnosis in STN?

Because of the risk of neoplasia, the single most

important investigation of choice is fine needle

aspiration cytology (FNAC).

Q 8. Can you differentiate a follicular carcinoma

from follicular adenoma in FNAC?

FNAC cannot differentiate follicular adenoma from

carcinoma. The distinction is dependent not on

cytology but on histological criteria namely capsular

and vascular invasion. The capsular and vascular

invasion can only be identified on histology, which

requires the thyroid tissue taken as paraffin block.

Q 9. What are the new techniques available for

differentiating follicular carcinoma from adenoma

in FNAC? (PG)

In future, FNAC differentiation may be possible by

the following techniques:

a. Ploidy study of the DNA material: Polyploidy for

benign and aneuploidy for carcinoma

b. Benign tumors are monoclonal and malignant

tumors are polyclonal (monoclonal antibody

MOAB 47)

c. Magnetic resonance spectroscopy

d. Thyroimmunoperoxidase estimation.

Q 10. What is the definition of ‘adequate smear’ in

FNAC? (PG)

An adequate smear should have at least six clusters

of cells each containing about 20 cells.

Q 11. What are the possible FNAC reports in thyroid?

Possible FNAC reports in thyroid

a. Benign—abundant colloid and typical follicular

cells.

b. Malignant

c. Indeterminate—little colloid and many follicular

cells or Hurthle cells (Follicular neoplasm/

suspicious)

d. Inadequate—cystic lesions, degenerating adenomas.

Solitary Thyroid Nodule (STN-Nontoxic)

47

Q 12. What is the classification of fine needle

aspiration cytology?

Classification of fine needle aspiration

cytology of thyroid

Thy 1 Non-diagnostic

Thy 2 Non-neoplastic

Thy 3 Follicular

Thy 4 Suspicious of malignancy

Thy 5 Malignant

Q 13. What is the overall diagnostic accuracy of

FNAC?

The overall diagnostic accuracy is about 95%

The diagnostic sensitivity of 83% and specificity

of 92%.

Q 14. What are the conditions in which FNAC will

give a definite diagnosis?

Colloid nodule

Thyroiditis

Papillary carcinoma

Medullary carcinoma

Anaplastic carcinoma

Lymphoma.

Q 15. Which institution was responsible for

pioneering the technique of FNAC?

Karolinska Hospital, Sweden (they decide the Nobel

prize for Medicine).

Q 16. What is the minimum number of needle

passes required in FNAC?

Minimum 6 passes.

Q 17. What is the approach if the STN is cystic?

The FNAC is less reliable in cystic swellings. After

aspirating cyst fluid, a further sample should be

taken from the cyst wall for cytology.

Q 18. What will be the course of action if the cyst

is recurring after aspiration?

A recurrent cyst should be removed surgically.

Q 19. What is the malignancy rate in cystic lesion?

Cystic lesions are less likely to be malignant. Malignancy

rate in complex cysts is 72% and simple cyst is 7%.

Q 20. What are the indications for true cut (core)

biopsy in thyroid?

It is useful in:

a. Anaplastic carcinoma

b. Lymphoma.

In these two situations, true cut biopsy can avoid

an operation.

Q 21. What are the problems of a true cut (core)

biopsy in thyroid?

Poor patient compliance

Pain

Bleeding

Tracheal injury

Recurrent laryngeal nerve damage.

Q 22. What is the role of isotope scan in STN?

Routine isotope scanning has been abandoned

in STN. When toxicity is associated with nodularity,

isotope scanning is done to localize the area of

hyperfunction.

Q 23. What are hot nodules, warm nodules and

cold nodules?

A hot nodule 5% (of which 5% of these are malignant)

is one that takes up isotope while the surrounding

thyroid tissue does not. Here the surrounding thyroid

tissue is inactive due to TSH suppression as a result of

excess thyroid hormone (Fig. 2.1).

A warm nodule 10% (of which 10% are

malignant) takes up isotopes along with normal

thyroid tissue (Fig. 2.2).

A cold nodule80%(of which 20% are malignant)

is one where there is no isotope uptake (Fig. 2.3).

Clinical Surgery Pearls

48

Differential diagnoses of cold nodule

1. Cyst

2. Hemorrhage

3. Benign adenomas

4. Malignancy

5. Thyroiditis.

Q 25. What is the role of ultrasonography in thyroid?

Differentiate benign from malignant nodule

The ultrasonography can demonstrate subclinical nodularity and identify deep nonpalpable thyroid nodules

Size of the nodule can be measured

It can also differentiate solid from cystic

swellings

Sono guided FNAC can be done

Identify cervical lymph nodes

Identify multicentricity.

Q 26. Can you differentiate benign condition from

malignancy in ultrasonography? (PG)

Yes, the differences between benign and malignant

conditions in USG are given here in the table.

Fig. 2.1: Hot nodule Fig. 2.3: Cold nodule

Fig. 2.2: Warm nodule

Q 24. What is the significance of a cold nodule

and what are the differential diagnoses of cold

nodule?

A cold nodule is more likely to be malignant. About

80%ofthediscretenodules are coldbutonly 20%of

the cold nodules are malignant.That means 80%of

cold nodules are benign and therefore cold nodule

as such is not an indication for surgery.

Solitary Thyroid Nodule (STN-Nontoxic)

49

Q 27. What are the differential diagnoses of

thyroid cysts?

a. Colloid degeneration—50%

b. Involution of follicular adenoma

c. Malignancy—10–15%

d. Papillary carcinoma.

Q 28. What are the indications for surgery in STN?

Indications for surgery in STN

a. FNAC positive for malignancy

b. Follicular neoplasm

c. Clinical suspicion—hard texture, fixity, hoarseness,

lymph node, male sex, etc.

d. Recurrence of a cyst after aspiration

e. Toxic adenoma

f. Pressure symptoms

g. Cosmesis

Q 29.When the FNAC report comes as follicular

neoplasm. How will you proceed?

Follicular neoplasm may be follicular adenoma or

follicular carcinoma and distinction is possible only

on histology evidenced by capsular and vascular

invasion. Majority (about 80%) of the follicular

neoplasms are benign.

Q 30. What are the clinical situations in favor of

malignancy?

Suspect malignancy in the following

clinical situations

a. Discrete swelling in a male (more likely to be

malignant)

b. Child with a thyroid nodule (50% malignant)

c. Extremes of age with discrete swelling (above 50

years and < 20 years teenagers)

d. Hard irregular swelling

e. Fixity

f. Recurrent laryngeal nerve paralysis

g. Lymph node

h. Size > 4 cm

i. History of head and neck irradiation

j. Association with other endocrine neoplasia (e.g.

Multiple Endocrine Neoplasia—MEN).

Q 31. What are the causes for a hard thyroid

nodule?

Causes of a hard thyroid nodule

a. Malignancy

b. Calcification—dystrophic

c. Thyroiditis

d. Hemorrhage into nodules.

Benign Malignant

1. Hyperechoic nodule 1. Hypoechoic

2. Significant cystic component 2. Need not be there

3. Peripheral egg shell like calcification 3. Microcalcifications inside

4. A sonolucent rim (halo) around the nodule 4. No halo

5. Well-defined nodule margin 5. Poorly defined margin

6. Taller than wide lesion

7. Increased central vascularity

Clinical Surgery Pearls

50

Q 32. What is the meaning of follicular neoplasia?

Follicularneoplasiamay be benign or malignant.

80% of follicular neoplasia are benign (20% are

malignant) and therefore total thyroidectomy as

a treatment is recommended only after getting

the histopathology report.

The pathologist can make a cytological

diagnosis of a carcinoma only in the following

situations:

1. Papillary thyroid carcinoma (PTC): Orphan

Annie eyed nucleus.

2. Medullary thyroid carcinoma (MTC): Cell

balls and amyloid stroma.

3. Anaplastic carcinoma (difficulty in differentiating it from lymphoma) .

Follicular carcinoma cannot be diagnosed by

cytology.

Q 33. What is the minimum surgery for a case with

FNAC report of follicular neoplasia when the STN

is confined to one lobe of thyroid?

It is preferable to do a hemithyroidectomy on the

side of the STN, i.e. removal of the affected lobe

along with the isthmus. The minimum surgery

should be a lobectomy.

Q 34. After hemithyroidectomy histopathology

is reported as carcinoma (papillary carcinoma /

follicular carcinoma), how will you proceed?

In this situation, re-exploration and completion

total thyroidectomy is recommended in all patients

so that further postoperative follow-up is possible.

Q 35. Why is re-exploration and completion thyroidectomy recommended in carcinoma thyroid?

a. In follicular carcinoma, the main mode of

spread is by bloodstream. When a patient

develops metastases, the treatment of choice is

radioiodine therapy. If the remaining thyroid is

not removed the iodine will be taken up by the

remaining thyroid and metastases will not take

up radioiodine. Therefore, it cannot be located

and treated.

b. The tumor marker for differentiated thyroid

cancer (Papillary and follicular carcinomas)

is thyroglobulin. The thyroglobulin level

as a tumor marker is significant only after

total thyroidectomy. The thyroglobulin levels

are elevated in tumor bed recurrence and

metastases anywhere in the body.

c. Multicentricity and intrathyroid spread which

are seen in papillary thyroid cancer can be

tackled with re-exploration and completion

thyroidectomy.

Q 36. What is the timing for completion

thyroidectomy? (PG)

It is better to do it as early as possible.

The ideal timing would be 2 to 3 days after the

initial surgery.

If this is not possible, do it after 12 weeks of

the initial surgery (it takes 12 weeks for the

inflammatory response to settle).

Q 37. What is the treatment if the histopathology

returns as papillary thyroid carcinoma (PTC)?

In this particular situation, patient may be categorized into low risk and high risk. In high risk group

and in tumors of more than 1 cm, re-exploration

and completion thyroidectomy is carried out.

Tumor of less than 1cm is called microcarcinoma

and follow-up is enough for such patients. Some

surgeons routinely practice total thyroidectomy

for such cases. (Please read the risk categorization

in the topic carcinoma thyroid).

Solitary Thyroid Nodule (STN-Nontoxic)

51

Flow chart for the management of solitary thyroid nodule depending on the four possible cytology reports

For PG’s—What is new?

US guided FNAC isthe mostimportantinvestigation

ForindeterminateFNACmolecularmarkersareusedtoidentifyRASandBRAFmutations,soalsoRET/PTCrearrangement

More than 70% of papillary thyroid cancer have BRAF and RAS mutations

Follicular lesion of undetermined significance (FLUS)—the risk of malignancy 10%.

FLUS positive for BRAF or RAS—total thyroidectomy is indicated

The immunohistochemical stains available for thyroid are:

Leukocyte common antigen (LCA)

– HBME - 1

– Galectin – III

– Cytokeratin

– Calcitonin—for medullary thyroid cancer

The rate of cancer for 4 cm size nodule is 19%

FDG PET avid thyroid nodule found incidentally deserves thorough work-up

FNAC

– Straw colored fluid—benign cyst

– Clear watery fluid—parathyroid cyst

– Hemorrhagic fluid—high risk of malignancy.

3 Papillary Carcinoma Thyroid with

Lymph Node Metastases

Case

Case Capsule

A 45-year-old female patient presents with a

swelling on the left side of the front of the neck of

5 years duration. For the last 1 year there is rapid

increase in size. There is no history of radiation to the

neck or exposure to radiation in childhood. There

is no family history of goiter or breast carcinoma.

The patient is coming from an endemic area. There

is no history of hypertension, or diarrhea. Patient

complains of hoarseness of voice for the last 6

months. On examination there is a hard irregular

swelling of about 8 × 5 cm size with distinct edges. It

moves with deglutition, but not with protrusion of

the tongue. There is restriction of the lateral mobility

of the swelling. The carotid arteries are displaced

backwards on the left side of the neck. There are

multiple lymph nodes on the left side of the neck

in the posterior triangle (Level 5). The lymph nodes

move more easily in the transverse than vertical

plane and do not move with swallowing.

Papillary Carcinoma Thyroid with Lymph Node Metastases

53

Clinical Surgery Pearls

54

Read the Preliminary Portion and Checklist of

Case No: 1

Q 1. How will you differentiate thyroid swelling

from lymph nodes of the neck?

The thyroid swelling will move up and down with

deglutition whereas the lymph node swelling will

not do so. For all practical purposes any swelling,

which is not moving up and down with deglutition

is non-thyroidal, e.g. may be lymph nodes, may be

neurofibroma or other solid and cystic swellings.

Q 2. What are the diagnostic points favoring

carcinoma thyroid in this case?

The diagnostic points are:

a. Hard thyroid nodule

b. The suspicious lymph nodes in the neck

c. Hoarseness of voice

d. Restriction of lateral mobility

e. Carotid displaced backwards on left side (Berry’s

Sign).

Q 3. What are the differential diagnoses of a hard

thyroid nodule?

Differential diagnoses of a hard thyroid nodule are:

a. Calcification of a nodule of a nodular goiter

b. Carcinoma

c. Thyroiditis.

Q 4. How will you identify calcification of a thyroid

nodule?

Simple plain X-ray of the neck will reveal calcification

corresponding to the nodule.

Q 5. What type of calcification do you get in such

cases?

The type of calcification is dystrophic.

Q 6. What is Delphic lymph node?

The Delphic Lymph Node is the first lymph node

involved in carcinoma thyroid. It is nothing but the

enlarged prelaryngeal lymph node.

Q 7. What is the importance of examining the scalp

in carcinoma thyroid?

The main mode of spread of follicular carcinoma is

by bloodstream and the most common metastases

are bony metastases that are usually seen in the

skull bone as a pulsatile bony swelling.

Q 8. What are the differential diagnoses of

pulsatile bony swelling in the scalp?

The differential diagnoses are:

a. Primary malignancy—solitary plasmacytoma,

telengiectatic variety of osteogenic sarcoma.

b. Metastases—from papillary carcinoma, renal cell

carcinoma, etc.

Q 9. What are the types of carcinoma thyroid in

which you get lymph node metastases?

1. Papillary carcinoma thyroid

2. Medullary carcinoma thyroid.

Q 10. What are the malignancies associated with

autoimmune thyroiditis?

a. Lymphoma of thyroid

b. Papillary carcinoma thyroid.

Q 11. What is the most important investigation in

this patient to confirm the diagnosis of carcinoma

thyroid?

Fine needle aspiration cytology (FNAC) from thyroid

nodule and FNAC or biopsy of the lymph node.

Q 12. Why FNAC or biopsy of lymph node?

The lymph node may harbor another pathology

like tuberculosis, which can be proved or disproved

by FNAC or lymph node biopsy. If the lymph

node biopsy report is coming as metastases from

papillary carcinoma, even if the thyroid is apparently

normal one should carefully palpate the thyroid and

exclude a suspicious nodule.

Q 13. What is the lymphatic drainage of thyroid?

The gland is drained by 2 sets of lymph vessels,

ascending and descending. Each consists of medial

and lateral channels (Fig. 3.1).

Papillary Carcinoma Thyroid with Lymph Node Metastases

55

Fig. 3.1: Thyroid lymphatic drainage with arrows

Clinical Surgery Pearls

56

Ascending vessels medial: Leave the upper

border of the isthmus and go to the nodes in

the cricothyroid membrane, i.e. the prelaryngeal

node.

Ascending vessels lateral: Leave the upper pole

of the gland along with the superior thyroid

artery to the deep cervical lymph nodes.

Descending medial: Pass from lower border of

the isthmus to the pretracheal group of lymph

nodes.

Descending lateral: Pass from the deep surface

of the thyroid to small nodes of the recurrent

laryngeal chain.

Q 14. What is the classification of neoplasms of

thyroid?

Classification of thyroid neoplasm is shown in Flow

chart 3.1.

Flow chart 3.1: Classification of thyroid neoplasm

Papillary Carcinoma Thyroid with Lymph Node Metastases

57

Q 15. What is the relative incidence of the various

malignant tumors of thyroid gland?

Carcinoma of the thyroid forms less than 1% of

human malignant tumors. It is the most common

endocrine malignancy.

About 90% are well differentiated (Papillary,

follicular, Hurthle cell)

About 10%are poorly differentiated (Anaplastic,

medullary and Lymphoma).

Incidence of malignant tumors of thyroid

1. Papillary carcinoma 60%

2. Follicular carcinoma – 17%

3. Anaplastic carcinoma – 13%

4. Medullary carcinoma – 6%

5. Malignant lymphoma – 4%

Q 16. What are the differences between the two

types of differentiated carcinoma namely papillary

carcinoma and follicular carcinoma?

Papillary carcinoma Follicular carcinoma

Multiple foci in the same Unifocal

lobe or on both lobes

(21–46%)

Main mode of spread Mainmodeofspread

– lymphatic bloodstream

Blood borne metastases Common

unusual

Intrathyroid lymphatic Absent

spread present

Papillary structure Papillary structure

present absent

Orphan Annie eyed Absent

nuclei

Capsular and vascular Diagnostic

invasion absent

Lymph node involve- Uncommon

ment common

Prognosis good Prognosis bad

Q 17. What are the indications for tru-cut biopsy

in thyroid (core biopsy)?

a. Anaplastic carcinoma

b. Lymphoma.

Q 18. What are the complications of tru-cut biopsy?

a. Pain

b. Bleeding

c. Tracheal injury

d. Recurrent laryngeal nerve damage.

Q 19. What is the main mode of spread of various

primary malignant tumors of thyroid?

The main mode of spread of papillary carcinoma

is lymphatic.

The main mode of spread of follicular carcinoma

is bloodstream.

The main mode of spread of anaplastic carcinoma

is both bloodstream and lymphatic.

Q 20. Can you make a diagnosis of follicular

carcinoma in FNAC?

No. The diagnosis of follicular carcinoma is by

capsular and vascular invasion which is by histology

(paraffin block).

Q 21. What is the FNAC appearance of papillary

carcinoma?

In FNAC the cells will be having Orphan Annie Eyed

Nuclei (pale empty nuclei).

Q 22. What is lateral aberrant thyroid?

This is also called occult carcinoma. The term lateral

aberrant thyroid is a misnomer because what we

mean by lateral aberrant thyroid is a lymph node

metastasis from an impalpable papillary tumor. The

primary tumor may be a few millimeter in size only.

The term occult carcinoma is applied to all papillary

carcinomas less than 1.5 cm diameter.

Clinical Surgery Pearls

58

Q 23. What is papillary microcarcinoma?

Papillary cancer less than 1cm is called Microcarcinoma. This is also called microscopic cancer

or laboratory cancer.

This is unexpectedly detected after a lobectomy

or hemithyroidectomy for a benign thyroid

condition.

Since recurrence and cancer-specific mortality

rates are near zero, no more surgery or 131I is

required in this situation.

Thyroid suppression therapy with thyroxine is

instituted.

Q 24. What are the etiological factors for carcinoma

thyroid?

1. Ionizing radiation: Accidental exposure to

radiation in less than 10 years of age (Chernobyl

Nuclear disaster). May produce papillary

carcinoma and follicular variant of papillary

carcinoma.

2. Iodine deficiency and raised TSH: Increased TSH

stimulation (Follicular carcinoma in seen in

iodine deficient areas and papillary thyroid

cancer in iodine rich areas).

3. Radiotherapy to head and neck for lymphoma

and thymoma in childhood will also lead on to

carcinoma thyroid in later years.

4. Autoimmune thyroiditis: Hashimoto is associated

with a 70-fold increase in lymphoma.

5. Genetic: Papillary carcinoma as a result of RET

proto-oncogene rearrangement, Kindred’s of PTC,

Cowden syndrome (DTC and Breast carcinoma

and Hamartomas), APC gene mutation, (associated

with papillary carcinoma) and BRAF and RAS

mutations are seen in papillary thyroid cancer. RET

gene for familial medullary thyroid carcinoma.

6. Oncogenes: C-myc, C-erb, Ras Oncogene, etc.

(associated with MEN).

Q 25. What is Cowden syndrome?

The association of DTC with carcinoma breast

and multiple hamartomas.

The defectis because of a germ line mutation of

PTENtumorsuppressorgene inchromosome 10.

Q 26. What is the pathology of papillary thyroid

cancer (PTC)?

Demonstration of true papillae

Orphan Annie nuclei

Nuclear pseudo inclusions

Nuclear grooves

Psammoma bodies in 50% of cases.

Q 27. What are the scoring systems available for

categorization of patients into low risk and high

risk in differentiated thyroid cancer?

Many scoring systems are available

1. AGES—Age, Grade, Extent and Size (Mayo

clinic—Hay et al)

2. AMES—Age, Metastases, Extent and Size (Lahey

clinic—Cady et al)

3. MACIS—Metastases, Age, Completeness of surgery,

Invasion of extra- thyroidal tissue, Size (Hay et al)

4. Sloan—Kettering

In first 3 systems, patients are categorized into

low risk and high risk. In Sloan - Kettering, patients

are categorized into 3 groups namely low risk,

intermediate and high risk.

Low risk Less than 45 years of age

 Less than 4 cm size

 Favorable tumor factors

Intermediate risk Low risk patients with high

 risk tumors

 High risk patients with low

 risk tumors

High risk More than 45 years of age

 More than 4 cm size

Unfavorable tumor factors.

Papillary Carcinoma Thyroid with Lymph Node Metastases

59

Q 28. What is logic behind considering papillary

and follicular carcinoma together for management

purposes?

They are considered together because, both of them

are Differentiated Thyroid Cancer Derived from

follicular cells. Differentiated Thyroid Cancer (DTC) is a

spectrum of disease rather than a single disease entity.

It is not a life-threatening disease and you can have

near normal life-expectancy. It is a tumor with indolent

biological behavior with variable aggressiveness.

Q 29. What do you mean by well-differentiated

thyroid cancer?

They constitute 90% of the thyroid cancers:

Papillary thyroid cancer

Follicular cancer

Hurthle cell tumors.

Note: 10% are poorly differentiated, comprising

of anaplastic, medullary cancers and lymphomas.

Q 30. What is Hurthle cell neoplasm?

It is a variant of follicular neoplasm in which

more than 75% of the follicles have oxyphil

(Asknazy or Hurthle) cells or oncocytic features.

The cells aregranularwithacidophilic cytoplasm.

Immunostaining for thyroglobulin is positive.

Mean age of the disease is 60 years.

It is multicentric in 33%.

More likely to produce lymph node mets.

It will not take up radio iodine.

Distant metastases are seen in 18%.

Q 31. What are the types of follicular carcinoma ?

They are classified as minimally invasive and widely

invasive.

Q 32. What are the peculiarities of follicular

carcinoma?

They do not invade the lymphatics and hence

lack nodal mets.

Immunostaining for thyroglobulin is positive.

Seen predominantly in women.

Q 33. What is ‘encapsulated variant’ of papillary

carcinoma? (PG)

Thistumor has a capsule-like adenoma but with

local invasion.

It may be associated with nodal metastases.

There is problem in distinguishing this lesion

from a hyperplastic nodule.

The overall prognosis is excellent.

Q 34. What is ‘diffuse sclerosing variant’? (PG)

This is seen primarily in children.

It is highly aggressive (often misdiagnosed as

Hashimoto’s).

Prominent lymphocytic infiltration is present.

Incidence of lymph node mets is 100%.

Prognosis is poor.

Q 35. What is ‘Lindsay tumor’? (PG)

It is a combination of the encapsulated variant

and the follicular variant of papillary carcinoma.

It behaves in a very indolent fashion and has good

prognosis.

Q 36. What is extrathyroidal spread?

The term extrathyroidal indicates that the primary

tumor has infiltrated through the capsule of the

gland.

Q 37. What is the AJCC staging (postoperative) of

differentiated thyroid carcinoma?

TNM classification for

differentiated thyroid carcinoma

T0 No primary

T1 Tumor diameter 2 cm or smaller

T1a Tumor < 1 cm

T1b Tumor > 1 cm, < 2 cm

Contd...

Clinical Surgery Pearls

60

T2 Tumor diameter > 2 to 4 cm

T3 Tumor diameter > 4 cm limited to the thyroid with

minimal extrathyroidal extension (extension to

sternothyroid muscle or perithyroid soft tissue)

T4a Moderately advanced disease; Tumor of any size

extending beyond the thyroid capsule to invade

subcutaneous soft tissue, larynx, trachea,

esophagus or recurrent laryngeal nerve

T4b Very advanced disease; Tumor invades prevertebral fascia or encases the carotid artery or

mediastinal vessels

Tx Primary tumor size unknown, but without extrathyroidal invasion

N0 No metastatic nodes

N1a Metastases to level VI (pretracheal, paratracheal,

prelaryngeal/Delphian nodes)

N1b Metastases to unilateral, bilateral, contralateral

cervical or superior mediastinal node metastases

Nx Nodes not assessed at surgery

M0 No distant metastases

M1 Distant metastases

Mx Distant metastases not assessed

Q 38. What is the importance of age in differentiating thyroid cancer (DTC)?

This is the only human cancer where age is included

in staging.

Q 39. What is the AJCC staging (postoperative) of

differentiated thyroid carcinoma? (PG)

AJCC staging (postoperative) of

differentiated thyroid carcinoma

Age less than 45 years (only 2 stages)

Stage I : any T, any N and M0

Stage II : any T, any N, M1

Age more than 45 years (4 stages)

Stage I : T1, N0 and M0

Stage II : T2, N0 and M0

Stage III : T3, N0, and M0

: T1, N1a, M0

: T2, N1a, M0

: T3, N1a, M0

Stage IV A :T4a,N0,M0(T4a/Lateral node is stage IVA)

: T4a, N1a, M0

: T1, N1b, M0

: T2, N1b, M0

: T3, N1b, M0

: T4a, N1b, M0

Stage IV B : T4b, any N, M0 (T4b is Stage IV B)

Stage IV C : any T, any N, M1 (metastasis is Stage IV C)

*Note: Stage III constitutes minimal extrathyroid

extension or level VI nodes.

All anaplastic carcinomas are considered stage IV.

The AJCC staging is for predicting the risk of death.

For assessing the risk of recurrence, a three level

stratification is recommended by the American

Thyroid Association, which is as follows: (PG)

After initial surgery and remnant ablation:

1. Low risk:

No local/distant metastases.

All microscopic tumors resected

No tumor invasion of locoregional structures

No aggressive histology

No vascular invasion

No 131I

 uptake outside thyroid bed.

2. Intermediate risk:

Microscopic invasion oftumorto perithyroid soft

tissue

Tumor with aggressive histology

Vascular invasion

Cervical lymph node metastases

131I uptake outside the thyroid bed after remnant

ablation

3. High risk:

Macroscopic tumor invasion

Incomplete tumor removal

Distant metastases.

Contd... Contd...

Contd...

Papillary Carcinoma Thyroid with Lymph Node Metastases

61

Q 40. What is minimal extrathyroid extension? (PG)

Extension to sternothyroid muscle or perithyroid

soft tissue is called minimal extrathyroid invasion.

Q 41. What is the prognosis of differentiated

thyroid cancer (DTC)?

The 25 yearmortality rateof a low-risk DTC is 2%.

That means 98% of the patients will survive 25

years.

• For the high-risk group the 25 year mortality

is 46%.

80 to 90% ofthe patients come underthe lowrisk group

In short 80% of the patients require lobectomy

alone

15% require aggressive treatment

5% die regardless of a treatment.

Q 42. What is the surgical treatment of choice for a

preoperatively proven case of papillary carcinoma?

If the FNAC is diagnostic of papillary thyroid

cancer, the treatment of choice is near total or total

thyroidectomy (TT) with central compartment

dissection. Lymph nodes in the jugular chain

should be carefully looked for during surgery.

Q 43. What is the surgery for a non-diagnostic

biopsy?

The surgery may be hemithyroidectomy or total

thyroidectomy. Total thyroidectomy is indicated in

the following situations.

> 4 cm tumor size

FNAC suspicious of PTC

Family history of PTC

History of radiation.

Q 44. What is the difference between near total

and total thyroidectomy?

In near total thyroidectomy 1 to 2 g of thyroid

tissue is preserved on the contralateral side, which

preserves blood supply to one or both parathyroids.

Q 45. What is central compartment neck dissection?

Itextendsfromhyoid bone above to innominate

vein below and carotids laterally.

Thyroid and the thymus are removed en-bloc

along with the paratracheal, tracheoesophageal,

pretracheal and prelaryngeal nodes.

Q 46. What is the rationale for central compartment

neck dissection and what are the problems

associated? (PG)

Fifty percent node positivity is seen in routine

central compartment dissection.

Reoperation and nodal dissection in the central

compartment area are difficult.

Thymus and thyroid are removed en-bloc in this

dissection.

Central compartment neck dissection is done as

long as low incidence of hypoparathyroidism

can be achieved.

Lower parathyroids are a greaterrisk for damage

in a central compartment neck dissection.

Q 47. What is the indication for central compartment

dissection in DTC?

Patients with clinically involved nodes therapeutic central compartment dissection done.

Prophylactic central compartment dissection is

recommended in DTC only for T3 and T4 tumors.

Q 48. What are the indications for completion

thyroidectomy when there is histological surprise

of carcinoma after a hemithyroidectomy?

Indications for completion thyroidectomy

1. History of radiation

2. High-risk factors

3. More than 1cm size

4. Contralateral thyroid nodule

5. Regional or distant metastases

Contd...

Clinical Surgery Pearls

62

Seen in elderly

Represents 10% of papillary tumors

Extracapsular and vascular invasion in 30%

Five-year survival rate is < 30%.

Q 53. What are the features of columnar cell

variety of papillary cancer? (PG)

This is seen only in men

All patients will die within 5 years of diagnosis.

Q 54. What are the indications for total thyroidectomy in carcinoma?

Indications for total thyroidectomy in carcinoma

1. Primary thyroid cancer > 1cm

2. Contralateral thyroid nodule

3. Patients with regional or distant metastases

4. History of radiation

5. History of DTC in first degree relatives

6. Age > 45 years

7. High-risk category.

Q 55. What is the treatment of lymph node

metastases in papillary thyroid cancer?

If lymph nodes are clinically present, a functional

neck dissection (preserving the sternomastoid

muscle, accessory nerve and internal jugular vein)

is carried out along with total thyroidectomy.

Oncologically carcinoma thyroid is the only

indication for a functional neck node dissection. If

clinically and peroperatively there are not any nodes,

only central compartment dissection is done.

Q 56. What is the postoperative follow-up after

total thyroidectomy?

All patients, after total thyroidectomy should

receive the following:

a. T4

 suppression (300 microgram eltroxine daily)

after scintigraphy for remnant thyroid tissue.

b. Look for thyroid remnant by scintigraphy at 6

weeks (1 mCi radioiodine for131I cervical scanning).

Contd...

6. Family history of DTC in first degree relatives

7. Age > 45 years

8. Multifocal tumor

9. Extrathyroid spread

10. Major vascular invasion

11. Major capsular invasion.

Q 49. Is there any role for lobectomy alone as a

treatment for DTC?

Lobectomy is acceptable in Low-risk patients with

Small intrathyroid tumor of < 1cm

Node-negative

Unifocal

Intrathyroid.

Q 50. What are the indications for lateral neck

node dissection (functional neck dissection)?

Indications for functional neck dissection

Lymphadenopathy detected clinically

Node identified by imaging

Biopsy-proven metastatic nodes

Frozen-section node positivity during surgery.

Q 51. What are the bad histological subtypes of

DTC?

Bad histological subtypes of DTC

Tall cell variety

Columnar cell

Trabecular

Scirrhous

Solid

Oxyphilic subtype of follicular thyroid cancer

Insular type of follicular thyroid cancer.

Q 52. What are the features of tall cell variety of

papillary cancer? (PG)

This is an aggressive variant

Papillary Carcinoma Thyroid with Lymph Node Metastases

63

For optimal scanning, the serum TSH level should

be at least 30 mIU/L. If the patient is on eltroxine,

two injections of recombinant human TSH will

give the desired effect.

c. If remnant is detected, Radio Remnant Ablation

at a dose of 30 mCi (RRA) is given (131I).

d. At 3 to 6 months, whole body scan with 3 mCi

radioactive iodine for metastases and ultrasound

examination is carried out.

e. If negative scanning is obtained, follow-up

with Thyroglobulin (Tg) level. Upon detection

of increased thyroglobulin level, further whole

body scan is indicated.

Q 57. What is the rationale of RRA? (PG)

a. RRA will destroy occult microscopic carcinoma.

b. Later detection of persistent or recurrent disease is

possible, after destruction of the thyroid remnant.

c. Serum thyroglobulin as a tumor marker is useful

only after complete removal and destruction

of the thyroid. (Therefore, there is no role for

thyroglobulin assay in the preoperative period).

Q 58. What are the indications for radio remnant

ablation?

Tumor size > 4 cm

Gross extrathyroid extension of tumor

Distant metastases

Lessthan 4 cm with lymph node metastases and

high risk group.

Q 59. What is the upper limit of normal level of

thyroglobulin (Tg)? (PG)

When on suppressive therapy: Above 2 ng/mL.

When the patientis hypothyroid: more than 5 ng/

mL.

Increasing values are important and TG antibody

should be quantitatively assessed.

These are indications of imaging for persistent,

recurrent or metastatic disease.

Q 60. If thyroglobulin (Tg) measurement is high

and total body scanning reveals metastases, what

will be the course of action? (PG)

For metastases, 100 to 200 mCi radioiodine 131I is

given (RAI).

Q 61. What are the precautions to be taken before

RAI therapy? (PG)

Precautions to be taken before RAI therapy

a. Low iodine diet for 10 days

b. Isolation of the patient (when dose is more than

30 mCi)

c. Oral fluid intake to increase urine outputso that

bladder injury is reduced

d. Lemon sucking to avoid sialadenitis

e. Avoid pregnancy for 6 months

f. Treat constipation with cathartics to reduce

gonadal and colonic irradiation

g. Sperm count reduction is noticed for several

months.

Note: Post-treatment whole body scan should be done

4 to10 days after RAI, which may detect new lesions

that need further treatment.

Q 62. What is the dose for skeletal metastases?

 (PG)

250 to 300 mCi (milli Curie) of 131I

Q 63. What is the role of WBS (Whole Body

Scanning) and ultrasound in follow-up?

Low risk patients with negative TG and negative

cervical ultrasound no need for WBS

Intermediate and high risk—6 to 12 months

after ablation do diagnostic WBS

CervicalUltrasoundat 6 and12 months andthen

annually for at least 3 to 5 years.

Q 64. What is the further follow?

a. Serum Tg level every 6 months along with TG

antibody.

Clinical Surgery Pearls

64

b. Lifelong suppressive therapy with thyroxin (the

TSH should be monitored every 6 months and

dose of thyroxin adjusted).

Q 65. What is the dosage of radio iodine?

Dose of radioiodine (131I) in differentiated

carcinoma thyroid

Cervical scan 1 mCi

Total body scan 3 mCi

Remnant ablation 30 mCi

Treatment of metastases 100 – 250 mCi

Q 66. What is the treatment of a solitary metastasis

in manubrium sterni from DTC?

Solitary bone metastasisisideally resected (the

dosage of radio iodine required for treating bone

metastases is very high and surgery gives lasting

relief)

Multiple bone metastases may be treated with

radio iodine.

Flow chart for management of DTC

After total thyroidectomy and TSH suppression

T4 withdrawal for 3 weeks OR two IM injections of 0.9

mg of recombinant TSH

Pre therapy low dose 131I – scanning (1 to 3 mCi) before

ablation

Radio remnant ablation (30–100 mCi) if indicated

Postablation whole body scan 5 to 8 days after RRA

If low-risk, negative thyroglobulin and normal neck

USG, then no whole body scan

If intermediate-risk and high-risk with persistent

disease, then 6 to 12 months after RRA do diagnostic

whole body scan

Further follow-up

Thyroglobulin: 6 to 12 months

CervicalUS: 6 and 12 months, annually for 3 – 5 years

Q 67. What are the peculiarities of medullary

thyroid carcinoma (MTC)?

a. They constitute 5 to 10% of all thyroid cancers

b. Are derived from parafollicular cells (C-cells)

developed from neural crests.

c. There is a characteristic amyloid stroma.

d. High levels of serum calcitonin are produced

(more than 0.08 ng/mL); itis a tumor marker for

medullary carcinoma

e. Diarrhea is a feature in 30% of cases

f. Some tumors are familial (~ 20%)

g. It may form part of multiple endocrine neoplasia

(MEN) syndromes: IIa and IIb

h. Clinical course is more aggressive than differentiated thyroid cancers

i. Do not take up radioactive iodine

j. Higher recurrence rate and mortality

k. Radiation and chemotherapy are ineffective

l. This is perhaps the only situation where a surgery

based on genetic testing is routinely done

m. Once the diagnosis of medullary thyroid cancer

is suspected, all patients should be screened for

mutation of the RET proto-oncogene to exclude

familial disease

n. When a genetic defect is found, all family

members should be screened.

Q. 68. What are the four clinical settings of MTC?

 (PG)

a. Sporadic medullary thyroid carcinoma (80%)

b. Familial MEN IIa

c. Familial MEN IIb

d. Familial non-MEN medullary thyroid carcinoma

Contd... (FMTC).

Contd...

Papillary Carcinoma Thyroid with Lymph Node Metastases

65

Q 69. What are the features multiple endocrine

neoplasia IIa ?

MEN IIa (Sipple’s syndrome)

Pheochromocytoma: frequently bilateral (may be

extra-adrenal)

Hyperparathyroidism

Medullary thyroid cancer

Seen in late childhood and teenage

Amyloid deposits in the skin of upper back

(Cutaneous Lichen Amyloidosis)

May be associated with Hirschsprung’s disease

Autosomal dominant.

Q 70. What are the features of multiple endocrine

neoplasia IIb?

Multiple endocrine neoplasia IIb

Familial medullary thyroid cancer

Mucosal neuromas of lips, tongue, inner eyelid

Marfanoid habitus

Pheochromocytomas are common (40 – 50%)

Severe gastrointestinal symptoms of alternating

diarrhea and constipation (because of the increased

number of ganglion cells)

Toxic megacolon and pseudo-obstruction are seen

Note: MEN I (Wermer’s syndrome) involves the

parathyroids, pituitary and the pancreas.

Q 71. What is the hallmark of multiple endocrine

neoplasia II?

The hallmark of MEN II is medullary thyroid

carcinoma.

MTC in MEN II is bilateral, multifocal, and affects

younger age group.

Q 72. What are the differences between MEN

associated MTC and non-MEN MTC? (PG)

The MTC associated with MEN are preceded by

hyperplasia of C-cells. It is possible to identify

relatives of such patients at the stage of hyperplasia

and operate before malignancy.

a. MTC with MEN are more aggressive

b. MTC with MEN is multicentric, (sporadic/nonMEN familial MTC is unicentric.

c. MTC with MEN occurs at a younger age.

Q 73. What are the features of familial non-MEN

medullary thyroid carcinoma?

Features of familial non-MEN medullary

thyroid carcinoma

They are autosomal dominant (germ line mutation

of RET gene)

Indolent course

Least malignant with very good prognosis

Mean age is 40 years

Occasion never manifests clinically

Extracellular/intracellular cysteine codon is seen.

Q 74. What are the features of medullary thyroid

carcinoma in MEN IIb?

Medullary thyroid carcinoma in MEN IIb

Most aggressive form of MTC is seen in this setting

Rarely curable

Affects very young age group: infancy and early

childhood

Autosomaldominant:germlinemutationofRETgene

Typical phenotype: Marfanoid.

Q 75. What is the screening program for the family

members of familial MTC? (PG)

Basal calcitonin may be normal at the stage

of hyperplasia but stimulation by calcium or

pentagastrin will give high value.

Screening every 6 months from the age of 5

years up to 50 years is useful.

Ultrasound of the neck is also useful for

identifying nodal metastases.

Clinical Surgery Pearls

66

Q 76. Is there any role for prophylactic thyroidectomy in family members? If so, at what age?

Yes. Prophylactic thyroidectomy can prevent

tumor occurrence at the stage of C-cell hyperplasia.

Surgery by 3 years for MEN IIa.

Surgery by 1 year for MEN IIb.

Q 77. What are the clinical presentations of

medullary thyroid carcinoma?

Medullary thyroid carcinoma is a great imitator

Lump in the neck

Nodal metastases

Paraneoplasticsyndromes:Cushing and carcinoid

Hoarseness, stridor and upper airway obstruction

Diarrhea and flushing.

Q 78. How do you confirm the diagnosis of

medullary thyroid carcinoma?

FNAC

Calcitonin:unstimulated serum calcitonin >100

pg/mL is suggestive of MTC

CEA: 50% of medullary thyroid cancer

USG/CT scan/MRI

Screening for pheochromocytoma by 24 hours

urinary catecholamines in all cases.

Note:

1. In all suspected MTCs, do USG abdomen and

24 hours urinary catecholamines to rule out

pheochromocytoma.

2. Rule out MTC in all thyroid cases with diarrhea.

3. Rule out MTC in all thyroid cases with hypertension.

Q 79. What is the treatment of medullary thyroid

carcinoma?

Since they do not take up RAI, surgery is the

only curative treatment.

No role of anything less than total thyroidectomy.

Routine dissection of central compartment is

a must.

Sampling of the jugular nodes are done (if

positive, Modified Neck dissection is done).

Autotransplantation of parathyroid is required

(for completeness of thyroid resection):

transplanted to the non-dominant forearm.

Some centers do a thymectomy with these

regularly (for fear of metastases in thymus).

Postoperative stimulated calcitonin assay is

done for assessment of the adequacy of surgical

resection.

Q 80. What is the treatment of anaplastic carcinoma of the thyroid?

It is very difficult to differentiate anaplastic

carcinoma from lymphoma of thyroid.

They are extremely lethal tumors

The treatment of choice is not surgery but

radiotherapy.

The indication for surgery is to relieve tracheal

obstruction by isthmusectomy. This will give

tissue for histology.

Curative resection is attempted when there is

no infiltration through the thyroid capsule.

Q 81. What is the management of lymphoma?

There is no role for surgery.

Radiotherapy and chemotherapy are the treatment of choice.

For low grade B cell MALT lymphoma, radiotherapy alone is enough.

Tracheal obstruction requires urgent chemotherapy after intubation.

Thetemptationforatracheostomymustberesisted.

Q 82. What is incidentaloma?

When performing imaging for other head and

neck problems, small nodule in the thyroid less

than 1 cm is identified by serendipity. It is called

incidentaloma. They are invariably benign and what

is required is an ultrasound guided FNAC.

Papillary Carcinoma Thyroid with Lymph Node Metastases

67

For PG’s—What is New?

1. Insular Carcinoma—It is otherwise called as Poorly Differentiated Thyroid Cancer (PDTC). It is having an

intermediate position between differentiated thyroid cancer and anaplastic cancer. Histologically solid clusters

of tumor cells are seen with variable number of small follicles. Capsular and vascular invasion are also seen.

Sometimes peritheliomatous pattern is seen. Calcification and bone are seen in stroma. It is an example of

De-differentiation theory of differentiated thyroid cancer. Metastasis are seen in bone and lungs. The 10-year

survival is 42%. The differences between insular and anaplastic are given below.

Insular Anaplastic

Younger – 45 Older – 70’s

P 53 and P 21 – negative P 53 and P 21–positive

Bone and lung metastasis Metastasis in variety of organs

5-year survival – 46%, 10-year 42% 5-year 15%, 10-year 3%

 P53 P53

Well DTC → Insular → anaplastic

2. Sonological findings of lymph node metastasis

Loss of fatty hilus

Round shape of lymph node

Hypoechogenic

Cystic changes

Calcification

Peripheral vascularity

3. Treatment of isolated bone metastasis from DTC

Wherever excision is possible e.g. Clavicle or sternum, surgery is preferred over radio iodine for therapy since a

very high dose of radio iodine in the range of 200 to 300 mCi isrequired for bone metastasis with itsside effects.

The excisional surgery is more curative than radio iodine.

4 Multinodular Goiter

Case

Case Capsule

A 30-year-old female patient comes from an

endemic area for goiter and presents with painless

swelling in front of the neck of 5-year duration. She

complains of nocturnal dyspnea and discomfort.

In the recumbent position the patient gets

dyspnea when she is lying on the left side. There

are no symptoms of toxicity. On examination

her pulse rate is 72/min. There is no tremor of

the outstretched hands. The jugular veins are

distended. A few dilated veins are seen over the

swelling and upper chest. On Pemberton’s test

there is congestion of the face and distress. There

is asymmetrical nodular swelling in the lower

half of the neck with up and down movements on

swallowing. The swelling has irregular shape. The

lower border of the swelling is not visible and

palpable. The nodules in the thyroid are having

varying consistency, some are firm, some feel

hard and some are soft in consistency. The nodules

in the central part (isthmus) are more prominent.

There is no fluctuation or transillumination. The

trachea is deviated to the right. There is no bruit

over the upper pole on auscultation. The cervical

lymph nodes are not enlarged. There is no clinical

evidence of toxicity or malignancy.

Multinodular Goiter

69

Read the Checklist and Preliminary Part of

Clinical Examination in Case No: 1

Q 1. What are your points in favor of nodular goiter?

a. The swellings are arising from thyroid gland.

b. Both lobes and isthmus are studded with

nodules of varying sizes.

c. There is no clinical evidence of malignancy or

toxicity.

Q 2. What is the pathology in nodular goiter?

Multinodular goiter is defined as a thyroid

enlargement with follicles that are morphologically

and functionally altered (there is structural and

functional autonomy). It is a discordant growth

of heterogeneous cell cohorts.

The etiopathogenesis is enumerated below:

1. Increased TSH stimulation

2. Iodine deficiency

3. Other environmental factors

4. Goitrogens

5. Heredity

6. Dyshormonogenesis

70

Clinical Surgery Pearls

7. Circulating growth factors like growth stimulating autoantibodies, thyroid stimulating

peptides, immunoglobulin stimulating growth.

Q 3. What is the natural history of a nodule formation?

Increased TSH secretion acts as a goitrogen.

a. TSH stimulation will lead on to diffuse

hyperplasia composed of active follicles. This is

called diffuse hyperplastic goiter. At this stage

it is reversible.

b. Later as a result of fluctuating TSH stimulation,

mixed patterns of active and inactive lobules

develop.

c. Active lobules become more vascular,

hyperplastic followed by hemorrhage and

central necrosis. The necrotic area is surrounded

by rim of active follicles.

d. The necrotic lobules coalesce to form a nodule

filled with either iodine-free colloid or inactive

follicles.

e. Repetition of this process will result in a nodular

goiter. Most nodules are inactive and active

follicles are present only in the internodular tissue.

Q 4. Do you expect increased TSH in nodular

goiter?

No. A plausible explanation for the growth

promoting effects of TSH is the presence of a

subset of thyroid follicular cells with an increased

sensitivity to TSH.

Q 5. What are the investigations required in this

case?

Most important investigations are FNAC and thyroid

function test (TFT).

Q 6. Which nodule will you select for FNAC?

The most dominant nodule

The most suspicious nodule

FNAC from more than one nodule may be required.

Q 7. What are the other investigations required?

Investigations for nodular goiter (In addition to

FNAC and TFT).

X-ray of the neck AP and lateral views. This is

to rule out tracheal displacement and tracheal

compression. Calcification of thyroid also can be

identified.

X-ray of the chest is also taken to rule out

retrosternal extension. If the soft tissue shadow of

the thyroid is coming down beyond the clavicle,

it is radiologically suggestive of retrosternal

extension.

Ultrasonography: High resolution USG can

identify clinically impalpable nodules. Expert

sonologists can differentiate benign from

malignant nodule. One can identify nodules as

small as 0.3 cm size in USG.

USG guided FNAC can also be done.

Indirect laryngoscopy: In all cases of nodular

goiter, it is important to have an indirect

laryngoscopy done to rule out occult recurrent

laryngeal nerve palsy, preoperatively.

Serum calcium: The preoperative calcium

assessment is important to rule out parathyroid

pathology and it will act as a baseline value in

the postoperative period.

Q 8. Is there any role for CT scan in a case of multinodular goiter?

Yes. The only indication for CT scan in nodular

goiter is when you suspect retrosternal extension.

MRI may also be useful in this situation.

Q 9. Is there any role for pulmonary function test

(PFT) in MNG?

Clinically whenever there is large/long-standing

goiter and tracheal deviation or compression, it

is better to do a pulmonary function test so that

Multinodular Goiter

71

one can identify a patient who is likely to develop

tracheomalacia/scabbard trachea.

Q 10. How will you identify and tackle tracheomalacia (weakness of tracheal rings)?

After thyroidectomy, the surgeon should palpate

for the tracheal rings for its strength. In case of

doubt, the anesthesiologist may withdraw the

endotracheal tube partially to check for weakness

of tracheal rings. When there is suspicion of

weakness of tracheal rings it is better to keep the

tube as tracheal stent postoperatively for 24 hours.

After 24 hours, extubate and see. If the patient is

going for trouble, do a tracheostomy.

Q 11. Is there any role for thyroid scintigraphy?

It is not required routinely. It is useful in a hyperthyroid patient with a dominant nodule as it

defines the area of hyperactivity, thereby choosing

proper surgical therapy.

Q 12. What are the complications of nodular goiter?

Complications of nodular goiter

1. Toxicity

2. Malignancy

3. Retrosternal extension

4. Pressure effects

5. Calcification.

Q 13. What type of toxicity do you get in MNG?

1. When the patient develops hypertrophy of

the nodule he/she gets Plummer’s disease,

(secondary).

2. When the internodular tissue is hypertrophied,

patient will get primary thyrotoxicosis.

Q 14. What are the indications for surgery in MNG?

The indications for surgery are:

1. Complications (already mentioned)

2. Cosmesis.

Q 15. What is the role of suppressive levothyroxine

therapy?

The results of T4

 suppression therapy are inconsistent

and marginal. Only small goiter would respond and

that too partially. Goiters that respond do so within

a period of 6 months.

Q 16. What are the disadvantages of suppression

therapy?

1. One should consistently suppress the TSH level

to < 0.5 mIU/L.

2. Periodic monitoring of TSH is required to rule

out hyperthyroidism.

3. Large amounts of thyroid tissue are likely to be

hormone insensitive in MNG.

4. Variable TSH dependency by various thyrocytes

resulting in inconsistent response.

5. Indefinite treatment is required and most of the

goiter recurs after cessation of therapy.

Q 17. Is there any role for radioiodine therapy in

MNG?

Radioiodine therapy is of no value in large MNG

with poorly functioning nodules. The efficacy of

RAI depends on the presence of reasonable gland

activity all over the thyroid that is not seen in large

MNG. Radiation induced autoimmune thyroiditis

and hypothyroidisms are other problems. It is of

limited value and useful only in two groups of

patient. Patients with small goiter with reasonable

gland function and patients with substantially

increased operative risk.

Q 18. What is the surgical option in a case of

nodular goiter?

The conventional surgery is subtotal thyroidectomy

that is not adequate in a case of nodular goiter that

is involving both lobes of the gland. The ideal option

will be to do a total thyroidectomy, with immediate

and lifelong replacement of thyroid hormone.

72

Clinical Surgery Pearls

When one lobe is more involved than the other

lobe, total lobectomy on the more affected side

with subtotal resection on the less affected side

will be an ideal option. It is, therefore, ideal to have

removal of the entire diseased gland, the so-called

adequate thyroidectomy.

Q 19. What is the real problem if you leave behind

diseased nodular portions?

1. It will produce enlargement and recurrence after

10–15 years.

2. Reoperation for recurrent nodular goiter is

more difficult and hazardous because of altered

anatomy and fibrosis. Therefore, it is better to

favor total thyroidectomy in younger patients.

Q 20. What is the timing of recurrence?

It is seen usually after 10 to 15 years.

Q 21. Is there any role for TSH suppression in

preventing recurrence?

If diseased nodules are left behind, TSH suppression

will not help in preventing recurrence. At

present its role is uncertain. However, once total

thyroidectomy is done, patient needs replacement

therapy to prevent hypothyroidism.

For PG’s—What is New?

Retrosternal Goiter

Definition: Goiter extending down to the level of the transverse process of the fourth thoracic vertebrae on chest

radiograph or goiter extending down to the level of arch of aorta (Goldenberg) or more than 50% of the mass lying

distal to the thoracic inlet (Katlic). 7 to 10% are bilateral, 7 to 10% are seen in posterior mediastinum.

Indications for surgery are:

1. Clinical symptoms (Stridor/dysphagia)—hoarseness, dyspnea, cough, etc.

2. Radiological evidence of tracheal narrowing

3. Esophageal compression

4. Superior venocaval syndrome

5. Malignancy

6. Toxicity.

Indications for sternotomy/thoracotomy

About 90% of the retrosternal goiters can be removed by cervical approach. Sternotomy is required in 2 – 8%.

The indications are:

The lower border below the level of aortic arch

More than 70% of the mass below the thoracic inlet

Posterior mediastinal mass (transthoracic approach through fourth space)

Malignancy technical tips for removal of retrosternal goiter

1. Division of strap muscles

2. Surgery on contralateral side first

3. Control of middle thyroid vein first

4. Ligation of superior thyroid pedicle initially

Contd...

Multinodular Goiter

73

5. Identify RLN and stay anterior to the nerve

6. Pushing out rather than pulling out

7. In posterior mediastinal goiter, the nerve may be ventral to that:

Grading of intrathoracic goiter

Grade I 0 – 25% in chest

Grade II 26 – 50% in chest

Grade III 51 – 75% in chest

Grade IV> 75% in chest

Since evaluation of retrosternal goiter is not possible with ultrasound and FNAC, surgery is recommended for all.

Contd...

5 Early Breast Cancer

Case

Case Capsule

A 40-year-old female patient presents with a

painless lump in the right breast of 6 months

duration, which she noticed while washing. There

is no history of backache, dyspnea, pleuritic pain or

jaundice. There is no family history of carcinoma

breast. The patient attained menarche at the age of

13 years. Her menstrual cycles are regular. Her first

child birth was at the age of 30-years. She has two

children and both were breastfed. On raising her

hands above her head, there is visible asymmetry

of the breast (right breast is more prominent and

distorted). There is retraction of the nipple on right

side and it is at a higher level. Areola is normal. There

is no discharge from the nipple on right side and it

is at a higher level. The skin overlying the breast is

normal. There is no peau d’ orange appearance or

ulceration or edema. On palpation the lump is 4 ×

3 cm size, stony hard in consistency in the upper

outer quadrant of right breast. The lump is fixed to

the breast, but there is no fixity to the skin or pectoral

muscles. There is a mobile, firm pectoral node

palpable in the right axilla. Supraclavicular fossa

and right arm are normal. Contralateral breast, axilla,

Early Breast Cancer

75

and supraclavicular fossa are normal. There is no

evidence of hepatomegaly or ascites on examining

the abdomen. There is no evidence of pleural effusion

or consolidation on examining the chest. Skull and

spine are normal. Pelvic examination is normal.

Checklist for Clinical Examination

1. Examine the patient in supine (45° semirecumbent position is ideal), sitting and arms

by the side, arms raised, hands on the hip and

leaning forward positions (Figs 5.1A to C).

2. Inspect both breast simultaneously for

asymmetry, look for visible lumps, and inspect

the nipple areolar complex.

3. In nipple look for 6 Ds:

Discharge

Destruction

Depression (retraction)

Discoloration

Displacement

Deviation.

4. Look for peau d’ orange, skin tethering, skin

fixity.

5. Check whether the lump is freely mobile

within the breast (breast mouse), whether it is

fixed to the breast, whether there is fixity to

pectoral muscles or to chest wall.

6. Examine the axilla for lymph nodes (remember

the various groups of lymph nodes). Examine

the supraclavicular area for lymph nodes.

7. Always examine the contralateral breast, axilla

and supraclavicular area.

Figs 5.1A to C: Examination of the patient with arms by the side, arms raised and arms on the hip

76

Clinical Surgery Pearls

8. Examine the abdomen for liver metastases

and ovarian metastases (Krukenberg’s tumor–

ovarian metastases in premenopausal ovaries.

9. Always examine the chest for signs of

metastases—pleural effusion, consolidation.

10. Look for bony tenderness—any evidence of

bony metastases (examination of spine, long

bones and skull).

Concepts in Breast Cancer

a. Halstedian concept of spread of carcinoma

breast: The spread of carcinoma breast as per

the Halstedian concept was that the tumor

will remain localized to the breast for some

time before spreading to the axillary nodes.

The tumor will remain in the axillary nodes for

some time before producing systemic spread

and metastases. William Halstead believed that

aggressively attacking the cancer when it is

localized to the breast and lymph nodes can cure

the cancer, i.e. the so-called locoregional attack.

He, therefore, advocated radical operations in

the form of removal of the entire breast, the

pectoral muscles (both major and minor) and

axillary nodes. Later it was found that this form

of aggressive treatment will not change the final

outcome and survival of the patient.

b. Concept of systemic disease introduced by

Fischer: According to Fischer, carcinoma breast

is a systemic disease from the very beginning

and micrometastases may be present even

when the tumor is very small and localized to

the breast. The staged progression of spread

described by Halstead is no longer true.

Aggressive locoregional attack is not going

tackle the micrometastases, which is possible

in carcinoma breast. Therefore, now we know

that the surgical treatment alone is not enough

for carcinoma breast. The micrometastases

must be managed by chemotherapy after the

surgical removal of the cancer. Thus, the concept

of multimodality management came into

existence.

c. Helman spectrum theory

d. Importance of axillary nodal status and dissection:

The most important single prognostic factorfor

carcinoma breast is the nodal status (whether the

axillary nodes are involved or not). Four or more

axillary node involvement is a bad prognostic

factor. Therefore, axillary dissection has become

a part of any form of surgical treatment including

the conservative breast surgery.

e. The concept of breast conservation: This concept

was initially introduced by Veronessi of Milan

(Italy). He introduced the QUART regimen

consisting of Quadrantectomy, Axillary

dissection and RadioTherapy for carcinoma

breast and found that there is no difference

in survival when this form of treatment is

compared with radical operations. Finally the

breast conservation therapy - BCT (consisting

of wide local excision, axillary dissection and

radiotherapy) is accepted as the treatment of

choice for early breast cancer, world over.

Note: Always put elliptical for wide excision. If the

lesion is nearer to the skin take more ellipse. When

the growth is superficial there is more chance for

nodal involvement.

Q 1. What are your points in favor of diagnosis of

carcinoma breast?

1. There is a hard lump, which is fixed to the breast.

2. Hard mobile axillary lymph node.

3. The nipple is elevated and retracted.

Early Breast Cancer

77

Q 2. What are your differential diagnosis in this

case?

1. Traumatic fat necrosis (It can present very

similar to carcinoma breast, hence, it is the first

differential diagnosis).

2. Antibioma.

3. Fibroadenosis (ANDI).

4. Fibroadenoma.

Note: Aberration in normal development and

involution (ANDI).

Q 3. What is the order of palpation of axillary

lymph nodes?

The examiner stands in front of the patient for

examining the following axillary nodes:

1. Central group

2. Apical group

3. Brachial group (lateral)—near the insertion of

pectoralis major muscle

4. Pectoral or anterior group.

Examiner stands behind the patient for palpation

of the following group of nodes:

1. Subscapular or posterior group (anteroinferior

to the latissimus dorsi muscle)

2. Supraclavicular nodes

3. Infraclavicular nodes.

Q 4. What is the incidence of carcinoma breast?

In India—one in 100 to 120 women are affected.

In Kerala—one in 50–60 women are affected.

Q 5. What are the predisposing factors for

carcinoma breast (risk factors)?

Note: Normally 12% risk.

Two times risk means 25%.

1. The number of menstrual cycles between

menarche and first child birth is the most

important predisposing factor for carcinoma

breast: If the number of cycles are more, there

is more chance for carcinoma of the breast.

When there is early menarche, which is seen

in developed countries because of the good

nutrition and the first pregnancy is delayed as

is the usual practice there, there is more chance

for carcinoma of the breast.

2. When the first full-term pregnancy is after

35 years of age, there is more chance for

carcinoma of the breast.

3. Increased hormonal exposure as is seen in

early menarche and late menopause (early

menarche when the menarche occurs before

12 years and late menopause is when the

menopause occurs after 50 years).

4. Patients who have had cancer in one breast

are at increased risk of development of cancer

in the other breast at the rate of 1 to 2% per

year.

5. Family history: Carcinoma breast is more

common in women with a family history of

breast cancer than in the general population

(they account for < 5% of all breast cancer).

If there is a history of a first degree relative

(mother, daughter and sister) with carcinoma

breast, there is more chance for carcinoma in

others. Tamoxifen for 5 years appears to reduce

the risk of carcinoma by 30 to 50% in this group.

6. Genetic:

 BRCA1 gene is seen in chromosome 17q—it

is associated with ovarian, colorectal and

prostate cancer. If the gene is positive there is

50 to 80% risk of developing carcinoma breast

(many will opt for prophylactic mastectomy).

 BRCA2 gene is seen in chromosome 13q—is

associated with familial male breast cancer.

 p53 is a tumor suppressor gene for carcinoma

breast.

78

Clinical Surgery Pearls

7. A breast which is denied of its function

(breastfeeding): There is more chance for

carcinoma.

8. Nulliparity is associated with increased chance

of cancer of the breast.

9. Women with carcinoma of the endometrium is

associated with increased risk of development

of carcinoma of the breast.

10. History of exposure to ionizing radiation: History

of Mantle radiotherapy for Hodgkin’s disease

in childhood is associated with increased

incidence of carcinoma, especially when the

breast is developing. But this will take at least

10 years.

11. Postmenopausal obesity: This is due to increased

conversion of steroid hormones to estradiol in

body fat.

12. Diet: Diets low in phytoestrogens and fatty food

is associated with increased risk of developing

breast carcinoma.

13. High intake of alcohol.

14. Age: Extremely rare below 20 years but there after

the incidence steadily rises so that by the age of

90 years nearly 20% of women are affected.

15. Geographical: Common in the Western world,

less common in developing countries and

Japan.

16. History of benign proliferative breast disease

is associated with increased chance so also a

breast with previous scars.

17. Environmental: Pesticides, etc.

18. Hormone replacement therapy: Combination of

estrogen and progesterone taken for a very

long duration is associated with increased risk for

carcinoma breast. Estrogen alone is associated

with slightly increased risk for carcinoma.

19. Sex: Female breast is more prone for carcinoma

than the male breast.

20. Oral contraceptive pills: Do not appear to

increase the risk of carcinoma breast.

Note: The two protective factors are:

1. Breastfeeding for more than 2 years

2. First pregnancy less than 21 years of age.

Q 6. How will you proceed to investigate such a

patient?

Triple assessment consisting of clinical examination, imaging (mammogram/ultrasound) and

FNAC/Core biopsy. FNAC/Core biopsy is done first,

after clinical examination.

Q 7. What is the first investigation of choice?

Imaging first - Mammogram/USG followed by

FNAC/core biopsy. FNAC will give a cytological

diagnosis of malignancy or benign nature. It is also

cheap, simple and cost-effective.

Q 8. What are the different grades of cytological

reporting?

Grade Result

AC0 (Grade 0) No epithelial cells present

AC1 (Grade 1) Scanty benign cells

AC2 (Grade 2) Benign cells

AC3 (Grade 3) Atypical cells present—may need a

biopsy if clinically or radiologically

suspicious

AC4 (Grade 4) Highly suspicious of malignancy

AC5 (Grade 5) Definitely malignant

Q 9. What are the stains used for staining FNAC

smears?

a. Giemsa

b. Papanicolaou

c. Hematoxylin and eosin.

Early Breast Cancer

79

Q 10. How many needle passes are required for

FNAC and what is the needle size?

Usually 23G needle is used.

Minimum 6 needle passes are required.

Q 11. If the FNAC is inconclusive can you repeat

the procedure?

Yes, you can repeat FNAC up to 3 times.

Q 12. Still FNAC is negative, how will you proceed?

Core biopsy is the investigation of choice in such

a patient. Usually core biopsy is not recommended

for lesions less than 2 cm because the small lesions

can be easily excised under local anesthesia.

Q 13. How is core biopsy done?

After infiltrating 1% lignocaine anesthetic into the

skin and down to around the lesion, a small incision

is placed and the lesion is approached with the

specific core biopsy needle at approximately 45o

angle to the breast so that it will not damage the

chest wall.

Q 14. What are the advantages of core biopsy

(Tru-cut) over FNAC?

1. The FNAC cannot differentiate in situ carcinoma

from infiltrating carcinoma.

2. Receptor assay (Estrogen receptor and

progesterone receptor) can be done with core

biopsy specimen.

Note: Therefore, core biopsy is the preferred method

for diagnosis of suspicious breast lump.

Q 15. What are the indications for excision biopsy

when FNAC is negative?

If a lesion is reported as malignant on cytology and

is clinically and mammographically malignant, the

patient does not require further confirmation.

Indications for excision biopsy

a. In lesions < 2 cm where core biopsy is not possible

without imaging.

b. In lesions between 2 and 4 cm where core biopsy

is negative

c. In cystic lesions when the cyst recurs or increases

in size or there is residual mass after aspiration and

FNAC is inconclusive twice

d. A discrete breast lesion even if it is considered

benign by all investigation needs excision biopsy.

Q 16. Is there any role for incision biopsy?

For lesions over 4 cm in size where FNAC and core

biopsy are negative, but the lesion is suspicious

of malignancy on clinical examination and

mammography, incision biopsy is done (excision

biopsy of such a big lump will produce distortion

of the breast).

Q 17. If the FNAC is inconclusive, what is the order

of further investigations?

Lessthan 2 cm size swelling—wide local excision

2–4 cm size swelling—core biopsy.

If core biopsy fails:

Size of tumor < 4 cm—wide local excision

Size of tumor > 4 cm—incisional biopsy.

Q 18. What is the incision planning for breast

biopsy?

Plan the incision in such a way that if the tumor

turns out to be carcinoma, the biopsy scar can be

included along with the nipple areolar complex

within the future transverse elliptical incision of

mastectomy.

For lesions within 5 cm of areolar margin,

cosmesis is better with circum areolar incision.

There is no role for radial incision in breast

except at 3 and 9 o’clock position.

80

Clinical Surgery Pearls

Prefer curvi linear incisions.

Followthe lines ofresting skin tension/Langer’s

lines.

Q 19. What are the modalities of imaging available

for breast?

a. Mammography

b. Ultrasound

c. MRI—useful for differentiating scar tissue from

tumor recurrence

Always use breast coil for MRI of breast

Always use contrast—type III enhancement

is suggestive of malignancy

d. 18 FDG PET scan (not routinely done).

The most frequently used modalities are

mammography and ultrasound. Above 35 years of

age, mammography is preferred. In young patients

less than 35 years of age ultrasound is preferred.

Note:

1. Screening mammogram is ideally done 5 to 7

days postmenopausal.

2. Now a days digital mammography is favored.

Q 20. Is there any role for mammography in this

patient if it is found to be positive for malignancy

in FNAC?

Yes. Mammography is indicated for the same breast

to rule out multicentricity. It is also indicated for

the contralateral breast to rule out a non palpable

lesion there.

Q 21. What are the mammography findings in a

case of carcinoma breast?

Mammographic findings in carcinoma breast are:

Primary signs

a. Mass lesion with clustered pleomorphic microcalcification (dense calcification is suggestive of

benign disease).

b. Speckled mass lesion with ill defined margin.

c. Architectural distortion of the breast parenchyma

(Stellate lesion).

d. Density of the lesion will be more than the rest

of the breast.

e. Malignant lesions are taller than wider.

Secondary signs

a. Skin changes—retraction, thickening, dimpling.

b. Nipple changes—flattening, retraction, etc.

c. Increased vascularity.

d. Axillary lymphadenopathy.

Q 22. What is the commonest malignancy of the

breast?

Infiltrating duct carcinoma.

Q 23. If the FNAC is reported as infiltrating duct

carcinoma, what will be the further management?

One should proceed to staging work up in such a

patient. Since this case clinically is an early breast

cancer, the minimum investigations required in

this patient are:

Staging work up in carcinoma breast

a. Mammography of the same breast and contralateral

breast

b. Ultrasound of the abdomen to rule out metastasis

in the liver and for Krukenberg’s tumor

c. X-ray of the chest to rule out pulmonary metastasis

d. Liver function test (LFT).

Q 24. What is the staging of carcinoma breast?

It is divided into 4 stages by the AJCC 6th edition.

Stage I and II are called early breast cancer

(EBC).

Stage III a, b, c are called locally advanced

breast cancer (LABC).

Stage IV is metastatic cancer.

Early Breast Cancer

81

Q 25. What is the TNM staging?

Primary tumor (T)

Tx—primary tumor cannot be assessed*

T 0—no evidence of primary

Tis—carcinoma in situ (Tis DCIS, Tis LCIS, Tis Paget’s)

Tis (DCIS)—Ductal carcinoma in situ

Tis (LCIS)—Lobular carcinoma in situ

Tis (Paget’s)—Paget’s disease of the nipple with no

tumor

T1— tumor less than 2 cm

T1 mi—Microinvasion 1 mm or less in greatest

dimension

T1a—Tumor more than 1 mm but not more than 5

mm in greatest dimension

T1b—Tumor more than 5 mm but not more than 10

mm in greatest dimension

T1c—Tumor more than 10 mm but not more than 20

mm in greatest dimension

T2—Tumor more than 20 mm but not more than 50

mm in greatest dimension

T3—Tumor more than 50 mm in greatest dimension

T4—Tumor of any size with direct extension to the

chest wall and or to the skin (ulceration or skin nodule

and peau d’ orange). Invasion of dermis alone does

not qualify as T4

T4a—Extension to chest wall, not including

pectoralis muscle adherence/invasion

T4b—Ulceration and or ipsilateral satellite nodules

and or edema (including peau d’ orange) of the

skin which do not meet the criteria for inflammatory

carcinoma

T4c—Both T4a and T4b

T4d—Inflammatory carcinoma

Note: TX-lump already excised, treated elsewhere

without documentation of the size, or treated with

chemotherapy.

The chest wall includes ribs, intercostal muscles

and serratus anterior muscle, but not the pectoral

muscles.

Regional nodes

Nx Regional lymph nodes cannot be assessed (e.g.

previously removed)

N0 No regional node metastases

N1 Metastases to movable ipsilateral level 1 and 2

axillary lymph nodes

N2 Metastases in ipsilateral level 1 and 2 axillary

lymph nodes that are clinically fixed or matted

or in clinically detected ipsilateral internal

mammary nodes in the absence of clinically

evident axillary lymph node metastases

N2a Metastases in ipsilateral level 1 and 2 axillary

lymph nodes fixed to one another (matted) or

to other structures

N2b Metastases only in clinically detected ipsilateral

internal mammary nodes and in the absence

of clinically evident level 1 and 2 axillary lymph

node metastases

N3 Metastases in ipsilateral infraclavicular (level 3

axillary) lymph nodes with or without level 1, 2

axillary lymph node involvement or in clinically

detected ipsilateral internal mammary lymph

nodes with clinically evident level 1 and 2 axillary

lymph node metastases or metastases in Ipsilateral

supraclavicular lymph nodes with or without axillary

or internal mammary lymph node involvement

N3a Metastases in ipsilateral infraclavicular lymph

nodes

N3b Metastases in ipsilateral internal mammary

lymph node(s) and axillary lymph node(s)

N3c Metastases in ipsilateral supraclavicular lymph

node(s)

Note: Clinically detected is defined as detected by

imaging studies (excluding lymphoscintigraphy) or by

clinical examination and having characteristics highly

suspicious for malignancy or a presumed pathologic

micrometastases based on fine needle aspiration biopsy

with cytologic examination. Confirmation of clinically

detected metastatic disease by fine needle aspiration

without excision biopsy is designated with f suffix

82

Clinical Surgery Pearls

Distant metastases

Mx Distant metastases cannot be assessed

M0 No distant metastases

M1 Distant metastases

Stage Grouping

Stage 0 Tis N0 M0

Stage IA T1 N0 M0

Stage IB T0 N1mi M0

T1 N1mi M0

Stage IIA T0 N1 M0

T1 N1 M0

T2 N0 M0

Stage IIB T2 N1 M0

T3 N0 M0

Stage IIIA Up to T3 Up to N2 M0

Stage IIIB Any T4 N0/N1/N2 M0

Stage IIIC Any T N3 M0

Stage IV Any T Any N M0

Note:

T1 includes T1mi

T0 andT1 tumors with nodal micrometastases only

are excluded from Stage II A and classified as Stage

IB.

If a patient presents with MI prior to neoadjuvant

systemic therapy, the stage is considered stage

IV and remains Stage IV regardless of response to

neoadjuvant therapy.

Post-neoadjuvant therapy is designated with “yc”

or “yp” prefix of note, no stage group is assigned

if there is a complete pathologic response (CR) to

neoadjuvant therapy.

Summary of Staging

Early breast cancer Stage I and II

 – T1N1, T2N1 and T3N0

Locally advanced breast Stage IIIA and IIIB

Cancer (LABC)

Metastatic breast cancer Stage IV

(MBC)

Q 26. What is the staging in this case?

Since the size of the tumor is T2 and patient is

having a single clinically positive axillary lymph

node the stage is IIB that is (T2, N1, M0).

Q 27. What is the origin of carcinoma breast?

The carcinoma breast originates from TDLU

Terminal duct lobular unit (Fig. 5.2).

Fig. 5.2: Breast and terminal duct labular unit

Q 28. What is the new pathological classification?

It is classified into ductal carcinoma and lobular

carcinoma depending on the origin from TDLU. Both

may be in situ cancer or invasive cancer.

Early Breast Cancer

83

In situ Invasive (infiltrating)

Lobular LCIS (lobular Infiltrating

carcinoma carcinoma in situ) lobular carcinoma

Ductal DCIS (ductal Infiltrating ductal

carcinoma carcinoma in situ) carcinoma

The invasive carcinoma is classified into:

Special types (ST)

No special type (NST)/not otherwise specified

(NOS).

Note:

The special types are having good prognosis

but forms only 15% of the total. The examples of

special types are tubular, mucinous, cribriform,

medullary, etc.

NOS or NST is having bad prognosis (85%).

Q 29. What is the commonest pathological type

of carcinoma breast?

Infiltrating carcinoma (NOS type is the commonest

pathology).

Q 30. What is Gene array analysis?

There are two types:

a. Oncotype Dx—21 gene analysis

b. Mammoprint—17 gene

Five subtypes of carcinoma breast are identified

1. Luminal A (40–55%) – ER positive and HER

2 negative–. They are well or moderately

differentiated. It occurs in postmenopausal

women and is slow growing. They respond well

to hormone treatment.

2. Luminal B (15–20%)—(Triple positive – ER,

PR and HER2 positive) – they respond to

chemotherapy.

3. Triple negative – ER, PR and HER2 negative (Basal

like) (13–25%) They are high grade tumors with

aggressive course and poor prognosis. Examples

are medullary carcinoma and metaplastic

carcinoma. There is high incidence of brain and

lung metastases. It has got poor prognosis. It

will affect younger patients often black races are

affected. There is a chemosensitive subgroup of

15–20%. BRCA1 mutation is seen in this group.

4. HER2 rich (7–12%) They are ER negative and

poorly differentiated with high proliferation

rate and high frequency of brain metastases.

Treatment of choice is chemotherapy and

Trastuzamab/Herceptin. However Trastuzamab/

Herceptin will not cross blood brain barrier in

case of metastases. Lapatinab is the drug of

choice when there is metastasis.

5. Normal breast like – (6–10%) ER positive and

HER2 negative.

Q 31. What is the tumor doubling time for

carcinoma breast?

One doubling time takes 2–9 months.

For reaching 1cm size of the tumor 30 doubling

times are required.

The minimum size required for clinical palpation

of a tumor is 1cm.

On an average, it takes five years for a tumor to

become clinically palpable in the breast.

Q 32. What is the grading of the tumor?

The Nottingham combined histological grade

(Elston – Ellis modification of Scarff BloomRichardson grading system) which is a modified

Bloom- Richardson grading is used for grading. In

this the nuclear pleomorphism, mitotic count and

tubule formation are taken into account giving

scores of 1 (favorable) 2–3 (unfavorable). A score

of 3–5 is grade I, 6–7 is grade II, 8–9 is grade III.

84

Clinical Surgery Pearls

Q 33. What is the surgical treatment of choice in

this patient?

Now the gold standard surgical treatment option for

stage I and II patients are BCT (Breast conservation

therapy). Modified radical mastectomy is another

option (Flow chart 5.1).

Flow chart 5.1: Management of early breast cancer

Early Breast Cancer

85

Q 34. What is breast conservation therapy?

Breast conservation consists of wide excision with 1

cm tumor clearance without excising the overlying

skin, axillary node dissection through a separate

incision in the axilla, followed by radiotherapy to

the rest of the breast.

Components – of BCT

Wide excision

Axillary node dissection

Radiotherapy to the remaining breast.

Note: Bilateral breast conservation surgery is not

recommended because of the irradiation risk

(bilateral).

Q 35. What are the precautions to be taken during

wide excision in breast conservative surgery?

Important steps of wide excision for carcinoma breast

a. The incision should be curvilinear placed along the

Langer’s lines (skin tension lines)

b. Removal of the skin overlying the lesion is

unnecessary

c. 1 cm tumor clearance is important (macroscopic)

d. The skin flaps should not be undermined.

e. Pectoral fascia is not excised unless involved

f. If it is involving the muscle, a fillet of muscle should

be removed

g. Specimen should be oriented with sutures or clips

h. Meticulous hemostasis is secured with diathermy

i. The cavity is lavaged with dilute chlorhexidine

solution and kept for 2 minutes

j. The cavity is not obliterated

k. No drain is put.

Q 36. What are the contraindication for breast

conservation?

Contraindication for breast conservation

1. T3, T4 tumors, N2 or M1

2. Large or central tumors in small breast

3. Multifocal/multicentric disease

4. Collagen vascular disease

5. Extensive in situ component.

Consider MRM (Modified radical mastectomy) in

these cases.

Q 37. What is the rationale for axillary dissection?

The single most important prognostic factor

for carcinoma breast is nodal status (4 or

more pathologically positive axillary node is

prognostically bad). Therefore, axillary dissection

will form part of the surgical management of all

types procedures including breast conservation.

Minimum of 12 lymph nodes must be removed

in axillary dissection.

Q 38. What is the average number of axillary

lymph nodes and internal mammary nodes?

Average number of axillary nodes is 53

Internal mammary nodes – 3 to 4.

Q 39. What is the rationale for radiotherapy in

early breast cancer surgery?

Radiotherapy is given to tackle the multicentricity

in the remaining breast.

The patient should be willing to undergo

radiotherapy 5 days a week for 20 days and

radiotherapy facility should be available.

The indications for radiotherapy in early breast

cancer are:

1. Four or more positive axillary nodes

2. Lymphovascular invasion (1–3)—High grade

tumors

86

Clinical Surgery Pearls

3. Resection margin positive

4. Skin and chest wall involvement

5. Size more than 5 cm.

Q 40. What is the rationale for mastectomy in early

breast cancer?

Mastectomy is recommended to remove the

multicentricity seen in carcinoma breast.

Q 4 1 . Wh at i s t h e d i f fe re n ce b e t we e n

multicentricity and multifocality?

When tumor foci are found in a different

quadrant it is called multicentricity.

When tumorfoci are found in the same quadrant

it is called multifocal.

Q 42. What is the t ype of mastec tomy

recommended for early breast cancer?

Modified radical mastectomy of Auchincloss is

preferred (Case No 6).

Q 43. What is QUART Regimen of VERONESI

(Contemporary Italian Surgeon Umbreto Veronesi

from Milan)?

Veronesi of Milan is responsible for bringing a

paradigm shift in surgical treatment of carcinoma

breast from mastectomy to conservation. Now quart

regimen is not followed. It consists of quadrantectomy,

axillary dissection and radio therapy).

Q 44. What is extensive in situ component (EIC)?

The presence of EIC in an invasive cancer is defined

as the presence of more than 25% DCIS component

in the main tumor or if DCIS is present elsewhere in

the surrounding tissue.

Q 45. What is inflammatory carcinoma?

Features of inflammatory carcinoma

It is a rare and most malignant form of breast

cancer, constituting less than 3% of cases.

It is a clinico pathological entity characterized by

diffuse erythema and edema (peau d’ orange) of

the breast, often without underlying palpable

mass involving 1/3rd or more of the skin of the

breast. Histologic presence of invasive carcinoma

invading dermal lymphatic is supportive of

diagnosis, it is not required nor is dermal lymphatic

invasion without typical clinical findings sufficient

for a diagnosis of inflammatory breast cancer.

The clinical findingsshould involve the majority of

the skin of the breast.

The term should not be applied to a patient with

neglected locally advanced cancer of the breast.

The skin changes are due to lymphoedema caused

by tumor emboli within dermal lymphatics

On imaging there may be detectable mass and

thickening of the skin over the breast.

The clinical presentation is due to tumor emboli

within dermal lymphatics.

Thisis usually not associated with a palpable lump.

It has a very bad prognosis and it is included under

stage III (classified as T4d).

Q 46. Is there any role for chemotherapy in early

breast cancer surgery?

Yes. The indications for chemotherapy are:

1. Node positive cases

2. HER2 positive cases

3. Triple negative

4. More than 0.5 cm size and less than 70 years of

age

6 Cycles of CMF (Cyclophosphamide, Methotrexate,

5 FU) or CAF (Adriamycin-based regimen) (Flow

chart 5.1). It is important to remember that CMF

regimen is not an adequate adjuvant therapy. The

Contd... new regimens are:

Contd...

Early Breast Cancer

87

a. Adriamycin, Cyclophosphamide (AC) with

Taxanes

b. AC followed by taxanes – 4 cycles of AC followed

by 4 cycles of taxanes

Q 47. What is the rationale for chemotherapy?

The new concept about carcinoma breast is that it

is a systemic disease from the very beginning. This

is in contrast to the old Halstedian concept where a

staged progression of the tumor from the breast to

the axilla and from their to systemic dissemination

was proposed. Therefore from the very beginning

patient can have micrometastases, which cannot

be detected. In order to tackle the micrometastases

systemic chemotherapy is given.

Q 48. Is there any role for hormone therapy in

this patient?

If the tumor is ER and PR (Estrogen receptor and

progesterone receptor) positive the patient is given

the antiestrogen called Tamoxifen 20 mg daily for

five years (Flow chart 5.1).

Q 49. What are the actions of Tamoxifen?

Actions of Tamoxifen

a. It has anti estrogenic action on the breast and

estrogenic action on other tissues

b. Tumoricidal

c. Prevent recurrence in the contralateral breast.

Note: Tamoxifen is a selective estrogen receptor

modulator (SERM) that has antagonistic action

on estrogen receptors in the breast and agonistic

action on ER receptor elsewhere.

The half-life of the drug is 7 days and it takes

4 weeks for action. The other beneficial effects of

tamoxifen are:

a. Preservation of bone density in postmenopausal

women

b. Decrease in cholesterol

c. Reduction in cardiovascular morbidity (These

are due to the estrogenic effects of Tamoxifen

on extra mammary tissues).

Q 50. What are the side effects of tamoxifen?

Side effects of tamoxifen

a. Hot flushes

b. Thromboembolic complications (deep vein

thrombosis and pulmonary embolism) and stroke

c. Vaginal dryness and atrophy in premenopausal

women

d. Vaginal discharge in postmenopausal women

e. Increase risk of endometrial cancer in postmenopausal women

f. Bone loss in premenopausal women

g. Weight gain.

Side effects in postmenopausal women

Side effects in premenopausal women

• Endometrial cancer • Bone loss

• Pulmonary embolism •  Decreased vaginal

secretion

• Stroke •  Atrophy of vaginal

epithelium

• Vaginal discharge

Q 51. What are the other hormones used in the

treatment of carcinoma breast?

Aromatase inhibitors—the production of estrogen

requires the activity of aromatase enzyme. Estrogen

production in postmenopausal women is by

peripheral aromatization of androgens produced

from the adrenal gland. Aromatase inhibitors block

estrogen production in postmenopausal women

and therefore they form adjuvant therapy for

postmenopausal women. A number of aromatase

inhibitors are available like:

88

Clinical Surgery Pearls

a. Aminoglutethimide—this also blocks cortisol.

b. Newer aromatase inhibitors, e.g. Anastrozole

1 mg daily, Letrozole, Exemestane

c. Raloxifene 60 mg daily (newer molecule of

Tamoxifen

d. LHRH agonists—Goserelin, Buserelin,

e. Other LHRH agonists—Leuprolide

f. Antiestrogen—Fulvestrant

g. Progestational agents—Magestrol

h. Androgens—Fluoxymesterone.

Q 52. What is the source of estrogen in postmenopausal women?

The circulating estrogen in postmenopausal

women is about 10% of the level in premenopausal.

They are synthesized peripherally principally in

fat including breastfat, skin, muscle and liver.

Adrenal gland.

Q 53. What is the timing of hormone therapy?

Hormone therapy is started after the completion of

chemotherapy to reduce the side effects.

Q 54. What is the role of surgical oophorectomy?

Oophorectomy was the standard hormone treatment

in the pre-tamoxifen era. Currently when patient

compliant regarding tamoxifen intake is doubtful

some centers are recommending oophorectomy. The

other alternatives for oophorectomy are:

1. Medical oophorectomy with aminoglutethimide—suppress adrenal

2. High dose estrogen—diethyl stilbestrol

3. Radiation oophorectomy

4. Chemotherapy as such will produce temporary

oophorectomy effect.

Q 55. Can you avoid axillary dissection in

carcinoma breast?

Yes. Axillary dissection can be avoided by sentinel

node biopsy.

Q 56. What is sentinel lymph node biopsy?

A sentinel node is a lymph node that primarily

drains the tumor area. Sentinel node biopsy

is done for early breast cancer with clinically

negative axillary nodes. Injection of radiolabeled

(technetium) sulfur colloid and or a blue dye

(methylene blue is acceptable) into the tumor

or peri tumoral area or the skin overlying before

excision of the primary allows identification of the

sentinel node in the axilla either by intraoperative

visual inspection or with the help of gamma probe.

The blue dye is injected intradermally during surgery

and then wait for 10 minutes. The dye will spread

along the lymphatics. 0.5 mL of the dye is used. The

sulphur colloid is injected the day before surgery at

3 pm at the areolar region (peri tumoral region also

possible). Dissect out the axilla starting from axillary

tail. Always remove the blue node and hot node

(when radio labelled sulphur colloid is used). Do

not remove nodes which are not blue and not hot.

For patients with breast cancer one or two nodes

are usually identified. The nodes are send for frozen

section (– 21o

C). 4 microns thick sections are taken.

The concept is that if the identified sentinel

nodes are negative for malignancy in frozen section

examination, the chances for a higher level node

to be affected by skip metastases is remote and

therefore an axillary clearance can be avoided

with the consequent morbidity. If the sentinel

node is positive the patient will go in for formal

axillary clearance. Node replaced completely by

metastasis will not take up the dye.

Note: Routinely as per NCCN guidelines level II

dissection is adequate.

There are two methods level III lymph node

dissection—going between the sternal and

clavicular fibres of pectoralis major from

Early Breast Cancer

89

the breast side or through the axilla via the

interpectoral route.

Q 57. What is axillary node sampling?

Removal of the nodes upto the intercostobrachial

nerve is called axillary node sampling. Minimum 4

nodes are removed.

Q 58. What is the management of DCIS (Ductal

carcinoma in situ)?

The DCIS can be classified into widespread type (i.e.

more than 4 cm size) and localized type. For widespread type, the treatment of choice is mastectomy.

The 3 surgical options are:

1. Simple mastectomy

2. Breast conservation surgery (Lumpectomy) and

Radiotherapy to the breast and Tamoxifen for 5

years.

3. Lumpectomy alone (without radiotherapy).

There is no need for axillary dissection in

DCIS because of the fact that chance for node

involvement is negligible (4%).

There is no role for chemotherapy in DCIS.

Q 59. What are the indications for mastectomy

in DCIS?

1. Women with multicentric DCIS

2. Extensive or diffused ductal carcinoma

3. Positive margins

4. Acceptable cosmesis cannot be achieved

5. Radiotherapy is contraindicated (Collagen

vascular disease)

6. Pregnancy

7. Diffuse malignant appearing microcalcification

8. Patient preference.

Q 60. What is oncoplastic surgery?

Here immediate reconstruction is done after

mastectomy of wide excision.

Q 61. What are the essential steps of lumpectomy?

Essential steps of lumpectomy

Complete surgical excision

Careful orientation of the specimen

Negative margin of at least 1 mm width

There should be no tumor at the margin (optimal

margin width not known)

Margin statusistaken before closure of the wound

Radiography to confirm excision of microcalcification

Postexcision breast imaging to confirm removal of

suspicious areas

Re-excision as necessary to obtain negative margin.

Note: Lumpectomy is followed by radiotherapy in

breast conservation treatment for DCIS.

Q 62. Is there any role for hormone therapy in

DCIS?

Yes. If they are ER positive (Estrogen Receptor

Positive), Tamoxifen is given for 5 years.

Q 63. What are the indications for lumpectomy

alone

Small lesions < 0.5 cm size

Unicentric lesion

Low grade lesion.

Q 64. What is the follow-up of DCIS after surgery?

Physical examination every 6 monthsfor 5 years

Mammogram every 12 months.

Q 65. What is LCIS (Lobular carcinoma in situ)?

The present consensus regarding LCIS is that it is

a marker of subsequent development of invasive

cancer rather than a preinvasive cancerous lesion.

(It is not a precursor but a predictor for carcinoma.

Hence it is called a bystander lesion).

LCIS is a rare form of non-invasive breast cancer.

It does not typically form micro- calcifications

and therefore not easily detectable on X-rays

and is usually found incidentally on biopsy.

90

Clinical Surgery Pearls

Invasiveductal cancerdevelopsinapproximately

25% of patients, which may be delayed for 10–20

years

– 45% develop ipsilateral carcinoma

– 35% develop contralateral carcinoma

– 15% develop bilateral carcinoma.

The riskofdevelopmentofinvasiveductal cancer

is equal for both breasts.

LCIS tends to be multifocal in both breasts.

The contralateral breast is affected in 30% of

cases and in the residual breast in upto 70% of

cases. The incidence of nodal involvement is

very low in this cancer.

Q 66. What is the management of LCIS?

Management of LCIS

History and physical examination every (6–12

months)

Diagnostic bilateral mammogram (every 12

months)

Pathology review

Observation for the development of invasive cancer

(ipsilateral and contralateral breast)

Risk reduction with Tamoxifen for pre-menopausal

women

Risk reduction with Raloxifen for postmenopausal

women

Bilateral mastectomy and reconstruction in special

circumstances.

Q 67. What are the indications for prophylactic

mastectomy?

1. LCIS (Lobular carcinoma in situ)

2. BRCA1 and 2.

Q 68. What are the features of lobular carcinoma

breast?

Incidence is 15%

Multifocal

Bilateral

The marker for lobular carcinoma is e-cadherin

antibody.

Q 69. What are the bad prognostic factors for

carcinoma breast?

Bad prognostic factors for carcinoma breast

1. Axillary nodal status is the most important single

prognostic factor

2. Age < 35 years—bad prognosis

3. Grade of tumor (Bloom-Richardson grade)—higher

the grade, bad the prognosis

4. Extensive in situ component – bad prognosis

5. Receptor status – ER negative and PR negative —

bad prognosis

6. P53 status (Tumor suppressor gene called ‘Guardian

of Genome’) positive—bad prognosis

7. Presence of HER—2 (Tyrosine kinase receptor) —

bad prognosis.

Q 70. What is Paget’s disease of the breast?

It is nothing but a ductal carcinoma with associated

nipple destruction, seen in 1–2% of the patients. In

50% there will be an underlying mass lesion. 90%

of these patients will have invasive carcinoma. For

the patients without mass lesion 30% will be later

found to have an invasive carcinoma. It should be

differentiated from eczema. Mammography is

indicated in such patients and also imprint cytology

of the lesion. Incisional biopsy is finally required.

If a mass lesion is present, treatment is MRM

(Modified Radical Mastectomy).

Otherwise a wide local excision of the nipple

areolar region, axillary dissection followed by

radiotherapy and other adjuvant treatment is

required.

Early Breast Cancer

91

Q 71. What are the difference between Paget’s

disease and eczema?

Difference between Paget’s disease and

eczema of the nipple

Paget’s disease Eczema

1. Unilateral Bilateral

2. Itching absent Itching present

3. Absence of oozing Presence of oozing

4. Scales and vesicles

absent

Scales and vesicles

present

5. Nipple destroyed Nipple is intact

6. Underlying lump may

be present

No underlying lump

7. Edges are distinct Edges are indistinct

8. Will not respond to

treatment

Will respond to treatment

9. Occurs at menopause

(Old age)

Seen in lactating women

(Young age)

Q 72. What are the causes for nipple discharge?

Causes for nipple discharge

1. Bright red

blood

Duct papilloma

(commonest)

Carcinoma in

lactating breast

Duct carcinoma

2. Dark altered

blood

Papilloma

obstructing a

duct (Discharge of

blood is delayed)

3. Blood stained fluid with

sizeable cystic swelling

Intracystic

papilliferous

carcinoma

(Disease of reclus)

4. Clear serous

fluid with

a lumpy

breast

(Yellow)

ANDI

In women taking

OCP

5. Green/Black

discharge

Duct ectasia

6. Milky

discharge

Insufficient

suppression of

lactation after

weaning

Secreting

prolactinoma of

the Pituitary gland

Hypothyroidism

7. Purulent Infection

Pregnancy and Carcinoma Breast

Q 73. What is the management of carcinoma

breast in pregnancy?

It behaves in similar way to breast cancer in

nonpregnant woman and treated accordingly.

Breast cancer during pregnancy and lactation

tends to be at a later stage because the

symptoms are masked.

Radiotherapy is avoided during pregnancy

and therefore mastectomy is preferred over

breast conservation, (causes death of embryo,

intrauterine growth retardation, neurological

anomalies, etc.).

Chemotherapy is avoided during the first

trimester but it is safe afterwards (CAF regimen

is better. Breastfeeding is avoided while on

chemotherapy).

Mosttumors are receptor negative andtherefore

hormone treatment is not required.

Contd...

Contd...

92

Clinical Surgery Pearls

Hormone treatment during pregnancy is

potentially teratogenic.

Lactation needs to be suppressed with bromocriptine if chemotherapy is instituted after

delivery.

Q 74. What is the breast imaging of choice in

pregnancy?

MRI is the choice

Ultrasound and mammogram are likely to yield

more false-positive results because of increased

parenchymal density, increased vascularity,

water content and cellularity.

Q 75. What is the staging work up during

pregnancy?

There is no contraindication for X-ray chest. It

can be done with abdominal shielding.

Alkaline phosphatase is not dependable during

pregnancy (It is elevated in pregnancy).

Bone scan is delayed until after delivery (MRI is

safe for metastasis).

Q 76. Can a patient become pregnant after

treatment of carcinoma breast?

Women are usually advised to wait for two years

for subsequent pregnancy since the maximum

recurrences are seen during the first 2 years.

Q 77. What is the management of carcinoma

breast during various trimesters of pregnancy?

First trimester

Modified radical mastectomy and axillary

dissection for the local disease

If found to be node positive, terminate the

pregnancy and give chemotherapy

If itislocally advanced terminate the pregnancy

and treat.

Second trimester

The decision is based on the stage of the

disease and the wish of the patient to continue

pregnancy

All stages surgery is recommended.

Third trimester

Do surgery

Waittilldeliveryandgiveadjuvantchemotherapy

after delivery.

Q 78. What is the role of therapeutic abortion in

carcinoma breast?

Itisindicated ifradiotherapy and chemotherapy

cannot be postponed due to the stage of the

disease

There is no advantage as such for therapeutic

abortion, but the treatment is made easier.

Q 79. What is bilateral breast cancer?

Itmeans a separateprimary cancerineachbreast

It can be synchronous or metachronous.

Q 80. How to differentiate whether the second

tumor is primary or metastatic?

The Choudary Millis criteria used for calling the

second tumor a primary are:

1. Demonstration of in situ change in the

contralateral tumor.

2. If it is histologically different from the cancer in

the first breast.

3. If the degree of histologic differentiation is

distinctly greater than the lesion in the first

breast.

4. In the absence of histologic difference, the

contralateral is considered to be primary,

provided there is no evidence of local, regional

or distant metastases from the cancer in the

ipsilateral breast.

Early Breast Cancer

93

Q 81. What are the risk factors for the development

of a second primary?

A woman who has had breast cancer has a five

fold increase in risk for second breast cancer

Multifocal cancer in the original breast

Lobular carcinoma in situ

Family history of breast cancer

Diagnosis of original cancer at an early age.

Q 82. What is the treatment of second primary?

Give treatment appropriate for tumor depending

on the stage of the disease.

Q 83. What is the mammographic findings in

metastatic cancer of the breast?

Mammographic findings in metastatic

cancer of the breast

Metastatic cancer islessinfiltrative (primary tumor

is more infiltrative)

More diffuse

Fewer fine calcifications

Presence of secondary edema.

Elegy for a Poor Breast

For years and yearsthey told me, be careful of your breasts. Do not eversqueeze or bruise them. And give them

monthly tests

So I heeded all their warnings, an protected them by law. Guarded them very carefully, and I always wore my

bra

After 30 years of astute care, my doctor found a lump

She ordered up a mammogram

To look inside that bump

”Stand up very close”she said. As she got my boob in line, ”And tell me when it hurts”she said. ”Ah yes! There,

that’s fine

She stepped upon a pedal. I could not believe my eyes! A plastic plate pressed down and down, my boob was

in a vise!

My skin wasstretched and stretched, from way up under my chin. My poor boob was being squashed,to Swedish

pancake thin

Excruciating pain I felt, within it’s vice—like grip. A prisoner in this viscous thing. My poor defense less tit!

‘Take a deep breath’she said to me, who does she think she’s kidding? My chest is mashed in her machine, and

woozy I am getting

‘There, that was good, ”I heard her say as the room was slowly swaying. ”Now, let’s have a go at the other one,”

Lord have mercy I was praying

It squeezed me from up and down. It squeezed me from both sides. I’ll bet she’s never had this done not to her

tender little hide! If I had no problem when I came in, I surely have one now

If there had been a cyst in there, it would have popped,“ker-pow!”

This machine was created by a man, of this, I have no doubt. I’d like to stick his balls in there, and see how they

come out!

94

Clinical Surgery Pearls

For PG’s—What is new?

Paget’s disease

The MRI findings are type II or III contrast kinetics and nipple areolar complex flattening

It is cytokeratin 7 positive

The surgical treatment is central wide excision and breast conservation

MRI in breast cancer

Indications are problem solving cases, equivocal cases and lobular lesions

Silicone prosthesis will give snow storm appearance in mammogram

PET in breast cancer

It is used for loco regional and distant metastasis

PEM – PET Emission Mammography

Targeted therapy in breast cancer

The oldest targeted therapy is ER and PR positive with tamoxifen

50% are ER positive and 20% HER2 new positive which is a human epidermal growth factor type IV

Trastuzumab (Herceptin) for HER2 new positive (15 to 20%)—it will not cross blood brain barrier and therefore

cannot be used for brain metastasis. This drug is used for adjuvant therapy concurrently with Taxane, as

neoadjuvant therapy in LABC and in metastatic cancer. It is usually given weekly (9 weeks upfront) or three

weekly regimen for 1year

The complication of trastuzumab is heart failure

Lapatinib is an EGF receptor antagonist it is an oral preparation and is available as tablets. It is used for stage IV

disease in combination with traustuzumab—will cross blood brain barrier and used for HER2 positive metastasis

Bevacizumab—VEGF blockade

Screening mammogram for non palpable lesions (not visualized in USG)

The patient is kept in prone position (Swimmer’s position)

Stereo guided FNAC or core biopsy ofthe identified lesion is done and thisisfollowed by specimen mammogram

Another option is wire localization and excision (the localization is done by mammogram)

Gene array (also called genetic profile) (please read the text)

Two types are available: Oncotype Dx-21 genes (this is done in formalin fixed paraffin specimen

• Mammaprint - 17 gene analysis and is done in frozen section

6 Advanced Breast Cancer

Case

Case Capsule

A 55-year-old postmenopausal obese female

presents with a swelling in the right breast. Her elder

sister died of carcinoma breast at the age of 40 years.

There is no history of bony pain, no hemoptysis and

dyspnea. There is no history of jaundice, headache and

seizures. The patient attained menarche at the age of

13 years. She was married at the age of 25 years and

her first child birth was at the age of 28 years. She

has got three children and all were breastfed. She

gives history of recent retraction of the right nipple.

There is no history of discharge from the nipple. On

examination, the entire affected breast is at a higher

level than the left breast. The breast as a whole is

pulled up and contracted compared to the normal

side. Dilated veins are seen in the skin overlying. There

is retraction and elevation of the right nipple. There

is a visible lump of 6 × 7 cm size occupying the upper

and lower outer quadrants of right breast. There is

edema of the skin over the mass with peau d’ orange

appearance. A few satellite skin nodules are seen but

they are confined to the same breast. On palpation

there is local rise of temperature and fixity of the skin

overlying the mass. The lump is hard in consistency

and fixed to the breast. It is also fixed to the pectoral

muscles but there is no fixity to the chest wall. There is a

visible axillary swelling on the right side. On palpation,

there is matted hard axillary lymph nodes of 5 × 3 cm

size involving both pectoral and central group. There

are no infraclavicular nodes. The supraclavicular fossa

is empty on both sides and no palpable nodes. There

is no edema of the right arm. The contralateral breast

and axilla are normal. There is no cervical lymph node

enlargement. There is no hepatomegaly and ascites

on abdominal examination. On chest examination

there is no evidence of consolidation or pleural

effusion. The lumbar spine is normal.

96

Clinical Surgery Pearls

Advanced Breast Cancer

97

Read the Checklist of Case No. 5

Q 1. What is the probable diagnosis in this case?

The first diagnosis in this situation must be

carcinoma breast because of the following reasons.

a. A hard lump which is fixed to the breast

b. Peau d’ orange appearance

c. Nipple retraction and elevation

d. Presence of suspicious axillary nodes

e. Skin nodules

f. Skin fixity.

Q 2. What is the cause for peau d’ orange appearance?

It is due to cutaneous lymphedema as a result of

blockage of the lymphatics draining the skin. Here the

pits of the hair follicles appear to be retracted beneath

the skin. Wherever the hair follicles are anchored to

the subcutaneous tissue, it cannot swell. In between

the hair follicles the skin will swell like an orange peel.

Q 3. What is the cause for nipple retraction and

elevation in malignancy?

Only recent retraction of nipple is of significance. It

is due to the extension of growth along the ducts

and the periductal fibrosis. This is responsible for

nipple elevation also.

Q 4. What is the difference between skin tethering

and skin fixity?

The breast is anchored to the underlying pectoral

fascia by bands called ligaments of Cooper. The

malignant cells invade these ligaments and reach

the skin. In early stage, movement of the lump along

the axis of the ligament of Cooper will not produce

skin dimpling. The moment the axis is changed

there will be dimpling. This is called skin tethering.

In skin fixity there is extensive infiltration of the

skin and skin cannot be pinched and there is no

independent movement of lump.

Q 5. How is a skin nodule formed in carcinoma

breast?

In carcinoma breast discrete nodules are formed. This

is due to the fact that the lymphatic trunks from the

skin drain separate portions of the skin. And there

is no communication between adjacent territories.

When cancer cells grow along these vessels it will

result in small separate cutaneous nodules.

98

Clinical Surgery Pearls

Q 6. What will be the staging in this case?

This patient has got a T4B N1 lesion and therefore

it is stage III B. It is included in LABC (Locally

Advanced Breast Cancer).

Q 7. What is LABC?

Stage IIIa and IIIb is called LABC.

Q 8. How will you proceed in such a case?

a. Confirm the diagnosis

b. Staging work up and investigation to exclude

metastases

c. Other routine investigations and assessment of

systems when surgery is considered.

Q 9. How to confirm the diagnosis?

a. FNAC of the lump breast and axillary node

b. If FNAC is inconclusive do core biopsy

c. If core biopsy is not feasible do incision biopsy

(when the lump is more than 4 cm size it is

preferable to do incision biopsy rather than

excision biopsy).

Q 10. What are the investigations required to rule

out metastases?

All investigations in early breast cancer including

mammography:

a. FNAC of axillary lymph nodes

b. Ultrasound abdomen to rule out metastases liver

c. Bilateral mammography

d. A total body bone scan—If bone scan is positive

it will alter the stage and it will become a stage

IV disease (Metastatic breast cancer).

Q 11. Why mammography is required in LABC?

A bilateral mammography is a must. We can

assess the tumor size and multicentricity. Tumor

response to chemotherapy can be assessed if

initial mammographic size is available. So also

mammography of the contralateral breast is

required.

Q 12. What is the purpose of total body bone

scanning (if not available a skeletal survey)?

A routine X-ray will pick up metastatic lesion only

when 60% of the bone is demineralized. Bone

scanning will pick up metastases in about 20–30%

of cases of stage III breast cancer and if metastases

detected, it will become stage IV disease. A raised

alkaline phosphatase will give a clue in this regard.

Q 13. How does metastatic lesion reach vertebrae?

The malignant cells from the breast reaches

vertebrae via the intercostal veins.The intercostal

vein will join the vertebral plexus of the veins. This is

also called Batson’s plexus of veins. The peculiarity

of this plexus is that there are no valves and

therefore the malignant cells can freely reach all the

vertebrae from the base of the skull to the sacrum.

Q 14. What are the common sites of bone

metastases?

Vertebrae, ribs, sternum, upper end of humerus and

upper end of femur.

Q 15. How does a metastatic lesion reach the liver

from a carcinoma breast?

The liver is involved in 2 ways. The most important

route is bloodstream spread. The liver also can be

involved by lymphatic route. The lymphatics from

the lower inner quadrant of the breast traverse

the plexus in the rectus sheath and reach the

subperitoneal plexus. From their along the falciform

ligament it will reach the liver.

Q 16. What are the common types of chest

metastases?

a. Pulmonary metastases in the form of “cannon

balls”

b. Pleural effusion

c. Consolidation

d. Chest wall metastases.

Advanced Breast Cancer

99

Q 17. What are the clinical features of brain

metastases?

a. Headache

b. Vomiting

c. Signs of increased intracranial tension

d. Rarely seizures.

Q 18. What are the modes of spread of carcinoma

breast?

1. Lymphatic

2. Local spread is responsible for the fixity of the

lump to the breast tissue and it is also responsible

for the increase in size of the tumor. Direct spread

to the skin via the Cooper’s ligament will result

in skin tethering and fixity, direct extension to

the pectoral muscle and chest wall.

3. Bloodstream spread—bloodstream spread

along the draining intercostal veins will reach

the vertebral plexus (Batson’s plexus) which is a

valveless system and result in bone metastases

in the vertebrae. In order of frequency the

lumbar vertebrae, femur, thoracic vertebrae,

rib and skull are affected. Generally they are

osteolytic metastases. Bloodstream spread

is also responsible for liver, lung and brain

metastases. It can produce metastases in most

of the body sites including adrenal.

Q 19. What is the lymphatic drainage of the

breast?

The lymphatics of the breast drain into the axillary

and internal mammary nodes. The axillary nodes

receive 85% of the lymphatics and the axillary

nodes are arranged as the anterior (pectoral),

posterior (subscapular), central, interpectoral

(Rotters—between the pectoralis major and minor),

lateral (brachial) along the axillary vein and apical

nodes which lie above the level of pectoralis

minor. The apical nodes are in continuity with the

supraclavicular nodes. From the supraclavicular

nodes they drain to the thoracic duct or jugular

trunk.

There are 2 plexus in the breast and they

communicate freely.

a. Subareolar plexus of Sappey

b. A plexus over the pectoral sheath.

The subareolar plexus of Sappey and the

upper outer quadrant of the breast drain to the

pectoral, then to the central and to the apical

axillary nodes. The subscapular and the lateral

(brachial) are involved rarely and in a retrograde

way. Part of the upper quadrant will also drain to

the deltopectoral and apical nodes directly. From

the inner quadrants the lymph spread occurs to the

internal mammary group of nodes and also to the

contralateral breast. From the lower inner quadrant

the lymph will traverse through the plexus in rectus

sheath and communicate with the subperitoneal

plexus. Tumors in the posterior 1/3rd of the breast

generally drain to the internal mammary nodes.

Q 20. What are the levels of axillary lymph nodes?

The division is by the pectoralis minor muscle.

There are III levels of axillary lymph nodes.

Level I – The axillary lymph nodes below and lateral

to the lateral margin of the pectoralis

minor muscle.

Level II – The nodes situated behind the pectoralis

minor and Rotter’s nodes (Interpectoral)

Level III –The nodes above and medial to the medial

border of pectoralis minor and inferior to

the clavicle these are also known as apical

or infraclavicular nodes. Metastases to

these nodes portend a worse prognosis.

100

Clinical Surgery Pearls

Therefore, the infraclavicular designation

is to differentiate it from the remaining

level 1 and 2 nodes.

Note:

Supraclavicular lymph nodes—they are seen

in the supraclavicular fossa a triangle defined by

omohyoid muscle and tendon (lateral and superior

border), the internal jugular vein (medial border)

and the clavicle and subclavian vein (lower border).

Adjacent lymph nodes outside this triangle are

considered to be lower cervical nodes and hence

M1.

Internal mammary lymph nodes are seen in the

intercostal spaces along the edge of the sternum in

the endothoracic fascia in 2nd, 3rd and 4th spaces.

They lie along the internal mammary vessels.

Intramammary lymph nodes—the lymph nodes

within the breast. These are considered axillary

lymph nodes for purpose of N classification and

staging.

Q 21. How many lymph nodes are there in the

axilla?

There are about 20–25 lymph nodes in the axilla.

Level I – 15

Level II – 4–5

Level III – 2–3

Total – 25

Minimum of 10 nodes are necessary for adequate

staging.

Q 22. What is Krukenberg’s tumor?

This is nothing but the transcelomic spread of

malignant cells from the subperitoneal plexus to the

surface of the functioning ovaries in premenopausal

women. If the ovaries are not functioning as in the

case of postmenopausal group, the malignant cells

cannot get implanted because of the absence of raw

area on the surface of ovaries.

Q 23. What are the predisposing factors for

carcinoma breast?

Read early breast cancer.

Q 24. What are the bad prognostic factors for

carcinoma breast?

a. Nodal status—The most important single

prognostic factor is nodal status more than

3 histologically positive node is prognostically

bad.

b. Age of the patient—Less than 35 years has got

bad prognosis.

c. The size of tumor—Tumors less than 1cm size has

excellent prognosis.

d. Tumor grade—Grade III Bloom Richardson grade

is associated with bad prognosis.

e. Nottingham prognostic index (NPI)—NPI = [0.2

× size in cm] × grade + stage. The stage was

based on nodal status and was the combination

of triple node biopsy (the low axilla, high axilla

and internal mammary chain).

f. Histology—Certain special types like classical

lobular, tubular, cribriform, medullary, mucinous,

papillary and adenoid cystic are having better

prognosis than NST.

g. ER and PR status—Approximately 60% of the

tumours contain detectable estrogen receptor

(ER). ER and progesterone receptor (PR) positive

patients having better prognosis. It is a relatively

weak prognostic factor.

h. Ploidy—Ploidy is a measurement of the relative

proportion of DNA in each cell. Anuploidy is

associated with bad prognosis. Diploid tumors

have a lower risk of relapse. The cells in active

Advanced Breast Cancer

101

cell division (S-phase) can be determined by

floor cytometry. S-phase is better predictor of

relapse and survival than ploidy. Low S-phase

tumors have a more favorable prognosis. The use

of antibody Ki-67 allows an easier estimation of

proliferation.

i. c-erb B2 (HER 2/neu)—This is a cell membrane

receptor protein and is the product of neu

oncogene. Even though it should be done

routinely to predict the likelihood of their

response to Herceptin its prognostic significance

is doubtful.

j. p53—The abnormality of p53 expression is

seen in breast, ovarian and bowel cancer. The

abnormality is associated with bad prognosis.

k. Epidermal growth factor receptor (EGF)—has got

bad prognosis.

Q 25. If there is no evidence of metastases this

case will be LABC and what will be the line of

management?

The LABC may be operable or inoperable. T3

N1 is operable as per NCCN guidelines 2012.

Operable lesions are subjected for surgery first.

Multimodal treatment is the therapeutic option

for inoperable LABC. It is an indication for

anterior chemotherapy, i.e. chemotherapy given

prior to surgery (neoadjuvant). Start the patient

with CAF/CMF regimen. 2 cycles of chemotherapy

are given first (5FU-600 mg/m2, Adriamycin 50

mg/m2, Cyclophosphamide 600 mg/m2). After

2 cycles the patient is evaluated for response

(Flow chart 6.1).

Q 26. How to assess response?

The response is assessed by mammography and

clinical assessment. The patient is categorized into

the following:

a. Complete responder (CR)—no palpable tumor

after chemotherapy (absence of invasive

carcinoma in breast and axillary nodes).

b. Partial responder—any decrease in size of tumor

or node compared to pre-treatment T/N

c. Non responder—Less than 50% decrease in size

to up to 25% increase in size

d. Progressive disease

– More than 25% increase in size

– New lesions appear

– Depth of patient.

Q 27. Why chemotherapy is given first?

a. Down staging is possible with chemotherapy

b. Surgery is possible after cytoreduction

c. Local control is not the issue in LABC

d. LABC patients are likely to harbor micrometastases

e. The policy of hitting the micrometastases first

f. Systemic therapy is a must in all patients

g. Allows assessment of chemosensitivity.

Q 28. What are the options for non-responders/

progressive disease?

a. If operable, surgery is done

b. Otherwise, XRT (radiotherapy), hormone therapy

are the options.

– Small percentage will respond to the drug Taxol

(only 15% of the chemoresistant will respond.

– Radiotherapy is given to the chest, axilla and

supraclavicular region

 5000 cGy in fractions—200 cGy over 5 weeks.

Q 29. If the patient is responding what surgery

is done?

The surgeons who prefer breast conservation

therapy will do breast conservation surgery.

Some surgeons perform MRM (modified radical

mastectomy) in this setting.

102

Clinical Surgery Pearls

Q 30. After the breast conservation/mastectomy

what is the sequencing of therapy?

For all those who undergo MRM the remaining 4

cycles of chemotherapy is given first. Those who

undergo breast conservation and those with

positive resection margin, radiotherapy is given first,

followed by subsequent chemotherapy.

Flow chart 6.1: Locally advanced breast cancer management

Advanced Breast Cancer

103

Delaying chemotherapy beyond 12 weeks after

mastectomy is not good. Usually chemotherapy

is started within 6 weeks after mastectomy (for

early breast cancer)

Tamoxifen is given only after chemotherapy

Radiotherapy is started within 4 to 6 months.

Q 31. What are the side effects of chemotherapy?

a. Bone marrow depression

b. Gastrointestinal (nausea and vomiting)

c. Alopecia.

The patient is monitored by Hb, TLC before

therapy and after the cycle on day 21. If abnormal,

do platelet count also. Serum albumin, SGOT, SGPT

are done before and after therapy.

Q 32. Is there any use of cyto protective agents for

chemotherapy induced hematological toxicity?

 (PG)

The GMCF—Filgrastim granulocyte macrophage

colony stimulating factor, (GCSF) (Granulocyte

Colony Stimulating Factor) are useful for reducing

the severity and duration of neutropenia. They

are given either before or upto 24 hours after

chemotherapy. Dose: 30 million units. But it is costly.

Q 33. What is Trastuzamab/Herceptin?

It is a new biological agent used as a molecular

target acting against the growth receptor HER2

(c-erb B2). It costs about 1 Lakh for a vial. It is given

once in three weeks into 6 months to 1 year.

The other agents currently available include

Bevacizumab which is a vascular growth factor

receptor inhibitor and Lapitinab which is a

combined growth factor receptor inhibitor.

Q 34. What is the role of hormone therapy in LABC?

It should be started in all patients from day 1

irrespective of menopausal status and carried on

for 5 years.

Q 35. What is the cut off value of the ER for

considering it as positive?

There is no cut off value but more than 10 fmol/

mg is considered positive. About 2/3rd of postmenopausal and half of premenopausal are

positive. ER value more than 30 fmol/mg response

rate for hormone therapy is 75%. ER value 3 to 10

fmol/mg the response rate is 20%.

Q 36. What type of MRM is done in carcinoma

breast?

Three types of MRM are available.

a. The classical Patey’s mastectomy where the

pectoralis minor is removed and level III axillary

dissection is done (after putting a transverse

elliptical incision enclosing the nipple areolar

complex and the skin overlying the tumor, the

entire breast is removed initially). The pectoralis

major is preserved.

b. The second modification is by ‘Scanlon’ where

the pectoralis minor muscle is divided, not

removed and thereby a level III dissection is

done.

c. Auchincloss modification. This differs from the

Patey procedure by not removing or dividing the

pectoralis minor muscle but by simply retracting

the pectoralis minor. This modification limits a

level III dissection. But only 2% of the patients

will potentially benefit by removal of the highest

level of nodes and therefore it is justified.

Q 37. What is the difference between axillary

clearance and axillary sampling?

The axillary sampling is usually performed through

a separate axillary incision and should ideally be

undertaken immediately prior to the wide local

excision. Starting from the lower axilla at least 4

palpable nodes are excised and sent separately for

histological examination. If 4 nodes are not palpable

104

Clinical Surgery Pearls

in level I then higher palpable nodes from level II,

level III, interpectoral region nodes are excised.

Therefore, axillary sampling is not merely a level

I dissection but samples of palpable nodes from

any level of the axilla. This will allow detection of

the so-called skip metastases, i.e. level II and III

involvement without involvement of level I.

In axillary clearance the contents of axilla

below the axillary vein are cleared upto the apex

of the axilla preserving the long thoracic nerve,

thoracodorsal nerve and vessels and if possible the

intercostobrachial nerve.

Q 38. What is the important complication of level

III dissection?

Lymphedema. The incidence of lymphedema rises

with amount of axillary surgery. The combination

of lymph node dissection and radiotherapy gives

higher incidence.

Q 39. What are the two most important

common myocutaneous flaps available for the

reconstruction of breast after mastectomy?

a. TRAM FLAP (transverse rectus abdominis

myocutaneous flap)

b. Latissimus dorsi myocutaneous flap

The tram flap may be carried out as a rotational

flap based on the superior epigastric artery or as

free flap using microvascular anastomosis of the

inferior epigastric vessel to the subscapular or

thoracodorsal vessels. Both these flap procedures

can be carried out as an immediate procedure or

delayed procedure. Immediate reconstruction is

less time consuming for the patient.

Q 40. What is the other option for reconstruction

after mastectomy?

Immediate placement of a prosthesis in the

subpectoral position with the help of a tissue

expander and later replacement of the expander

with a prosthesis.

Q 41. What are the organs/tissues involved in

stage IV disease (metastatic breast cancer)?

Sites of Metastases in Breast Cancer

a. Bone

b. Lung with or without pleural effusion

c. Pericardial effusion

d. Brain

e. Lymph nodes

f. Skin, soft tissues of chest wall, and axilla

g. Metabolic complications like hypercalcemia.

Q 42. What is the median survival of MBC

(metastatic breast cancer)?

Two years.

Q 43. What are the treatment options in MBC?

Treatment options in MBC

H – Hormone therapy

E – Endocrine manipulation

R – Radiotherapy

O – Oestrogen blockers

I – Immunotherapy

C – Chemotherapy

(Source: Mnemonic by Dr Selvakumar)

Q 44. Which metastasis is prognostically better?

Soft tissue metastasis has better prognosis. Visceral

metastasis has worst prognosis. Response to treatment

decreases when the number of organs involved

increases. Generally receptor negative tumors are

aggressive and receptor positive are indolent.

Q 45. What is the basis for the selection of chemo/

endocrine therapy?

Endocrine therapy is preferred for the following

situations:

Advanced Breast Cancer

105

a. Slow growing soft tissue/bone metastases (only

30% will be hormone response)

b. Disease-free survival of more than two years

c. Age more than 35 years

d. Objective response to first line hormone therapy

Chemotherapy is preferred in following situations:

a. Rapidly growing visceral, skin metastases and

lymphangitis

b. Disease free survival less than two years

c. Negative response to first line hormones

d. Receptor negative.

Q 46. What is the choice of drugs in MBC?

The CAF regime provides better response than CMF.

Taxanes are used for anthracycline resistant cases.

Combination of taxanes and anthracycline regimes

are becoming gold standard.

Q 47. What are the indications for surgery in MBC?

Indications for surgery in MBC:

a. Locoregional control (Toilet mastectomy)

b. Spinal cord compression (Laminectomy)

c. CNS or choroidal metastases

d. Bone fractures (Internal fixation + radiotherapy)

e. Oophorectomy

f. Localized chest wall lesions.

Q 48. What are the indications for radiotherapy

in MBC?

a. Pain relief in skeletal metastases

b. Spinal cord compression

c. CNS diseases

d. Tumor recurrence in chest wall

e. Ovarian ablation.

Q 49. What is biological therapy?

The HER 2/neuoncogene amplification and over

expression is found in about 25–30% of patients with

CA breast. Recombinant humanized monoclonal

antibody against this oncogene is now available as

HERCEPTIN. The response rate is around 15%. The

drug is very costly. The combination of herceptin

and chemotherapy may improve the results. Breast

cancer vaccines, inhibitors of protein kinases and

antiangiogenesis factors are also included in this

category.

Q 50. What is the treatment of pleural effusion

in CA breast?

After draining pleural fluid through intercostal

drain, pleurodesis with tetracycline 1.5 g will give

symptomatic relief. Pleurodesis with Bleomycin

30–60 units is also being tried.

Q 51. What is the treatment of hypercalcemia due

to metastases?

This may sometimes be fatal. The treatment

is hydration and steroids. Bisphosphonates

(pamidronate, clodronate) in the dose of 90 mg

IV once a month may be useful which will arrest

demineralization and decrease the incidence of

pathological fracture and vertebral collapse. It will

also decrease the pain from bone metastases.

For PG’s—What is new?

Oligometastatic disease—Isolated single metastasis is called Oligometastatic

For a triple negative cases—Chemotherapy is given as first line treatment (for early breast cancer)

MRI is done before and after chemotherapy in LABC—(Titanium Clips are put in the periphery of the mass or

tattooing before chemotherapy)

Sentinel node biopsy is discouraged in LABC

Multifocal pattern of residual disease is a contraindication for breast conservation, so also positive axillary nodes

Full course of chemotherapy is given instead of sandwich surgery for complete pathological response

7 Epigastric Lump

Case

Case Capsule

A 60-year-old male patient presents with epigastric

pain, discomfort, distension of abdomen and

loss of appetite of 8 months duration. The patient

has lost 15 kg in the last two months. He vomits

large quantities of undigested food. His pain is

not relieved by vomiting or eating. On examination

there is gross wasting and extreme pallor visible in

face and hands. Inspection of the abdomen revealed

epigastric distension and visible peristalsis. The visible

peristalsis is seen starting in the left upper quadrant

and moving towards the right side. Succussion splash

is heard. There is a hard irregular massfelt in the

epigastrium extending to the right hypochondrium

beneath the costal margin of 10 × 7 cm size. The mass

moves with respiration and one cannot get above

it; however, fingers can be insinuated between the

costal margin and the lump. All the borders except

upper border are well made out. The lump is resonant

on percussion. There is no shifting dullness and no

ascites demonstrated. There is no other palpable lump.

The liver is not palpable. The left supraclavicular nodes

are not enlarged. Digital rectal examination revealed

absence of Blumer’s shelf. On chest examination, there

is no evidence of pleural effusion. There is no evidence

of superficial thrombophlebitis. (Fig. 7.2)

Checklist for Examination

1. Remember the 9 areas of the abdomen. (Fig.

7.1)

2. Remember that there is a large area called

retroperitoneum in addition to these 9 areas.

3. Pelvic organs can also come and occupy the

lower abdomen.

4. Look for visible lumps and visible peristalsis

(left to right upper abdominal visible peristalsis

is suggestive of gastric outlet obstruction and

right to left visible peristalsis is suggestive of

left sided colonic obstruction)

Epigastric Lump

107

5. Always assess the plane of the lump (Carnett’s

test, cough test, examination in knee elbow

position.

6. Always check for movement with respiration

and intrinsic mobility of the lump.

7. In upper abdominal lumps look for finger

insinuation between the lump and costal

margin.

8. Remember the causes for disappearing mass

— intussusception and Dietl’s crisis.

9. In suspected renal mass always look for

contralateral kidney.

10. Bimanual palpability will be positive for all

big lumps. Carefully palpate and decide with

which hand it is better felt.

11. Ballotability is a sign specific for kidney.

12. Percussion over the lump to see whether it is

dull or resonant and check whether the dullness

is continuous with liver dullness/splenic dullness.

13. Palpate for nodules in the umbilicus (Sister

Joseph’s nodule).

14. Examine the hernial orifices.

15. Always examine the genitalia in males.

16. Do a per rectal examination (Blumer’s shelf).

17. Examine the renal angle for fullness, tenderness

and dullness.

18. Look for supraclavicular lymph nodes

(especially on left side between the two heads

of sternomastoid) – Troisier’s sign.

Q 1. What is the most probable diagnosis in this

case?

A distal gastric cancer with gastric outlet obstruction.

Fig. 7.1: Nine areas of the abdomen

Fig. 7.2: Epigastric mass

108

Clinical Surgery Pearls

Q 2. What are the reasons for diagnosing a

stomach lump?

a. Symptoms of loss of appetite, loss of weight and

vomiting.

b. The type of visible peristalsis is from left to

right which is suggestive of a gastric outlet

obstruction by a tumor.

c. The lump is occupying the epigastrium and is

moving up and down with respiration.

Q 3. What is the significance of family history in

carcinoma stomach?

Gastric cancer can occur in families. A famous gastric

cancer patient in history is Napoleon Bonaparte. He

died of gastric cancer (?). His father and grandfather

died of gastric cancer. His three sisters and a brother

died of gastric cancer.

Q 4. What are the causes for gastric outlet

obstruction?

Causes for gastric outlet obstruction

1. Duodenal ulcer with pyloric stenosis

2. Distal gastric malignancy with outlet obstruction

3. Congenital hypertrophic pyloric stenosis

4. Adult type of hypertrophic pyloric stenosis

5. Acid poisoning with stenosis.

Q 5. What are the signs other than visible gastric

peristalsis (VGP) to diagnose gastric outlet

obstruction?

1. The dilated and distended stomach can be

palpated sometimes.

2. The succussion splash can be elicited.

Q 6. What is the objective evidence for gastric

outlet obstruction?

The objective evidence for gastric outlet obstruction

is saline load test. This is a simple means of

assessing the degree of pyloric obstruction and is

useful in following the patient’s progress during the

first few days of nasogastric suction.

After aspirating residual gastric juice through the

nasogastric tube, 700 mL of normal saline at room

temperature is infused over 3–5 minutes and the

tube is clamped. 30 minutes later the stomach is

aspirated and the residual volume of saline recorded.

Recovery of more than 350 mL indicates obstruction.

Q 7. What are the other abdominal clinical signs,

one should look for ascertaining that it is stomach?

a. May or may not be able to get above the lump

b. Percussion note will be resonant, may be some

times impaired.

Q 8. What are the clinical signs of inoperability in

such a patient?

Signs of inoperability in carcinoma stomach

a. The presence of ascites suggesting peritoneal

metastases

b. The presence of Blumer’s shelf which is nothing

but a peritoneal metastasis in the rectovesical/

rectouterine pouch

c. The presence of a metastatic liver

d. The presence of left supraclavicular lymph nodes

(Virchow’s, Troisier’s sign)

e. The presence of a cutaneous nodule in the

umbilicus (Sister Mary Joseph’s nodule)

f. Presence of jaundice

g. Fixity of mass

h. Irish nodule (Enlarged left axillary node)

i. Pleural effusion

j. Krukenberg’s tumor (enlarged ovaries in pelvic

examination).

Q 9. In the absence of these signs can you say that

it is an early gastric cancer?

No. By the time a lump is clinically palpable it will

be advanced gastric cancer.

Epigastric Lump

109

Q 10. What is the cause for jaundice in carcinoma

stomach?

Infiltration of the growth engulfing the CBD

(common bile duct)

Diffuse involvement of the hepatoduodenal

ligament.

Q 11. What is the pathway for umbilical nodule

formation (Sister Mary Joseph’s Nodule)?

Growth in lesser curvature

Involvement of lesser omentum

Involvement of ligamentum teres

Umbilicus

Note: Sister Mary Joseph was Nurse to Sir William

Mayo.

Q 12. What is early gastric cancer (EGC)?

Early gastric cancer is a cancer confined to the

mucosa and submucosa of the stomach irrespective

of the nodal status (T1 any N).

EGC with nodes surgery is recommended

EGC without nodes endoscopic surgery.

Q 13. What are the clinical manifestations of

carcinoma stomach?

The pneumonic stomach helps in memorizing

the clinical features.

Silent (No symptoms)

Tumor (Lump)

Obstruction

Melena

Achlorhydria

– Anemia

– Asthenia

Cachexia

Hemtemesis.

Q 14. How will you proceed to investigate such

a patient?

The most important investigation in this particular

situation will be an upper GI endoscopy.

Q 15. Why upper GI endoscopy is preferred over

a Barium meal examination?

ByUpper GI endoscopy, we can see the growth,

assess the type and extent of growth, and also

take biopsy for confirmation.

In Barium meal examination, we will only see

the shadow of the lesion in the form of persistent

irregular filling defect, persistent mucosal

irregularity and persistent loss of peristalsis in

a particular segment.

Q 16. In which situation Barium meal examination

is superior to upper GI endoscopy?

Barium meal examination is superior in the

following situations:

Barium meal is useful for the diagnosis of Linitis

Plastica (endoscopic biopsy confirmation may

be difficult because the mucosa is normal)

Lesions in the cardiac.

Lesions causing obstruction.

For early gastric cancer lesions.

Q 17. How many endoscopic biopsies will you take

from a suspected lesion in the stomach?

Minimum Six biopsies from different parts of the

identified lesion (Diagnostic accuracy reaches 100%

if 10 biopsy samples are taken).

Q 18. What are the types of biopsies available

endoscopically?

a. Biopsy with the help of a biopsy forceps

b. Punch biopsy

c. Brush biopsy

d. Gastric lavage and cytology.

110

Clinical Surgery Pearls

Q 19. If the biopsy is being reported as adenocarcinoma (the most common carcinoma in

stomach), how will you proceed?

In the order of priority, the following investigations

are done to stage the disease and rule out

metastases.

Staging investigations

a. Ultrasound abdomen to rule out metastases and

assess the extent of the disease

b. X-ray chest to rule pulmonary metastases

c. CT abdomen if required to stage the disease

d. Liver function test (LFT) to rule out metastases

e. Hemogram to rule out anemia.

Q 20. In what way does the ultrasound examination

help in evaluating this patient?

a. The presence of liver metastases can be assessed

and if required a sono guided FNAC can be done

b. The tumor thickness can be assessed with the

help of a high resolution probe

c. The presence or absence of regional lymph

node and adjacent organ involvement

d. The presence of free fluid can be ascertained.

Q 21. Is there any role for endoscopic ultrasound

(EUS)?

By ordinary abdominal ultrasound one may not be

able to ascertain the tumor thickness and serosal

involvement which is possible with EUS. To some

extent the presence of nodes can be also assessed

by endoscopic ultrasound.

Q 22. Why is CT required in addition to ultrasound

examination?

The gastric carcinoma appear as areas of mural

thickening. The measurement of the thickness of

the wall will give an idea of transmural extension,

when the thickness is more than 2 cm. In addition, a

blurred serosal contour or strand like densities that

extend into the perigastic fat may be seen. Direct

invasion of the adjacent structures like pancreas,

diaphragm, transverse colon, spleen or left lobe of

liver can be ascertained by CT scan.

Q 23. Is there any role for laparoscopy in such a

patient?

Yes.

Role of laparoscopy in carcinoma stomach

Diagnostic laparoscopy is useful for staging the

disease

Pick up peritoneal metastases (CT will miss

peritoneal metastases)

It can detect occult metastases in 13–37%

Laparoscopic ultrasound can detect metastases in

liver

It can identify adjacent organ invasion

Guided biopsies are possible.

Laparoscopy can help in doing a peritoneal lavage

and cytology

Laparoscopycaneliminate the need for laparotomy.

Q 24. What are the laparoscopic signs of

inoperability?

Laparoscopic signs of inoperability

1. Positive cytology in peritoneal wash

2. Peritoneal deposits

3. Posterior fixation

4. Fixed celiac nodes

5. Para-aortic nodes

6. Liver metastases.

Q 25. What is the staging system for carcinoma

stomach?

In the west, the AJCC TNM system based on the 7th

edition is accepted. But the Japanese Investigators

report their results based on the JRSGC (Japanese

Epigastric Lump

111

Research Society for Gastric Cancer). The N3 and

N4 nodes in the Japanese system correspond to

extra-regional nodes in UICC system that is M1

metastatic nodes.

AJCC 7th edition Staging

Tx: Primary cannot be assessed

T0: No evidence of primary tumor

Tis: Carcinoma in situ – intraepithelial tumor

Without invasion of lamina propria

T1: Tumor invades lamina propria, muscularis

mucosa or submucosa

T1a: Tumor invades lamina propria or

muscularis mucosa

T1b: Tumor invades submucosa

T2: Invades muscularis or propria

T3: Tumor penetrates sub serosal connective

tissue without invasion of visceral peritoneum

or adjacent structures

T4: Tumor invades serosa (visceral peritoneum) or

adjacent structures

T4a: Tumor invades serosa (visceral peritoneum)

T4b: Tumor invades adjacent structures

Nx: Regional node cannot be assessed

N0: No nodes

N1: Metastases in 1–2 regional nodes

N2: Metastases 3–6 regional nodes

N3: Metastases 7 or more regional nodes

N3a: 7–15 regional lymph nodes

N3b: Metastases in 16 or more regional lymph nodes

M0: No distant metastases

M1: Distant metastases (this includes peritoneum and

distant lymph nodes)

Stage 0 Tis N0, M0

Stage IA T1 N0 M0

Stage IB T1 N1 M0/T2 N0 M0

Stage IIA T3 N0 M0 /T2 N1 M0/T1 N2 M0

Stage IIB T4A N0 M0 / T3 N1, M0 / T2 N2 M0 / T1

 N0 M0

Stage IIIA T4aN1 M0/T3 N2 M0/T2 N3 M0

Stage IIIB T4b N0 M0/T4b N1 M0 / T4a N2 M0/

T3 N3 M0

Stage IIIC T4b N2 M0 / T4b N3 M0 / T4a N3 M0

Stage IV Any T, any N, M1

Q 26. What are the nodal stations according to

Japanese Research Society?

1. Right cardiac

2. Left cardiac

3. Lesser curvature

4. Greater curvature

5. Suprapyloric

6. Infrapyloric

7. Nodes along left gastric artery

8. Common hepatic artery

9. Celiac

10. Splenic hilar

11. Splenic artery

12. Hepatic pedicle

13. Retropancreatic

14. Mesenteric root

15. Middle colic artery

16. Para-aortic.

Lymph node stations

1– 6 are consideredN1 nodes (Perigastric nodes

– along the curvatures)

7 – 11 are considered N2 nodes (Along the

named vessels)

12 – 14 are considered N3 nodes (Intraperitoneal nodes)

Contd... • 15 – 16 are considered N4 nodes.

Contd...

112

Clinical Surgery Pearls

Note:

N1 and N2 nodes are regional nodes

N3 nodes are metastatic node stations

Involvement of N3 node station is a

contraindication for radical surgery.

Q 27. What do you mean by D1and D2 resection?

The extent of lymphadenectomy in the Japanese

system is described using a D descriptor.

D0 lymphadenectomy means all the JRSGC

(Japanese Research Society for Gastric Cancer)

N1 nodes have not been completely removed.

D1 lymphadenectomy means all the N1 nodes

have been removed but not all the N2 nodes.

D2 lymphadenectomy means that all the N1 and

N2 nodes have been removed but not all the N3

nodes.

Thus, for example, a D2 lymphadenectomy for

a tumor involving all 3 gastric areas would involve

removal of lymph nodes at stations 1- 11.

The extent of lymphadenectomy before

1993 was classified by the ‘R’ descriptor. Since

1993 in order to avoid confusion with the UICC

R descriptor (which reflects an entirely different

characteristics namely, the presence of known

residual disease after surgical treatment), the extent

of lymphadenectomy is denoted by D-descriptor.

Q 28. What are the bad prognostic factors for CA

stomach?

1. Lymph node involvement is a poor prognostic

factor—four or more lymph node involvement

is a bad prognostic sign.

2. Serosal involvement is the single most important

bad prognostic factor.

3. Free carcinoma cells in the peritoneum.

4. Intestinal type is having a better prognosis than

diffuse (Lauren’s).

Q 29. What are the poor prognostic variables that

relate to surgery?

1. Positive resection margin

2. Inadequate lymphadenectomy

3. Need for splenectomy.

Q 30. What is Lauren’s pathological classification?

DIO

Diffuse

Intestinal

Others.

According to this classification, it is divided into

diffuse and intestinal type. The intestinal type is

having a better prognosis than the diffuse type.

In intestinal gastric cancer, the tumor resembles

a carcinoma elsewhere in the tubular GI tract and

forms polypoid tumors or ulcers. It probably arises

in areas of intestinal metaplasia.

In contrast, diffuse gastric cancer infiltrates deeply

into the stomach without forming obvious mass

lesions, but spreads widely in the gastric wall. A small

proportion of gastric cancers are of mixed morphology.

Thediffusetypemaybe localized or generalized.

The generalized type of diffused is called Linitis

plastica (leather bottle stomach).

Q 31. What is Correa cycle?

Helicobacter seems to be principally associated

with carcinoma of the body and distal stomach.

Helicobacter is associated with gastritis leading

on to gastric atrophy. Similarly exposure to nitroso

compounds will lead on to nitrosamine leading

to gastritis and gastric atrophy which in turn

will produce intestinal metaplasia leading on to

dysplasia, carcinoma in situ and finally carcinoma.

Q 32. What are the other risk factors for carcinoma

stomach?

1. Pernicious anemia

2. Gastric atrophy

Epigastric Lump

113

3. Gastric polyps

4. Heredity

5. Postgastrectomy patients—Post GJ, gastrectomy and post-pyloroplasty patients have

approximately 4 times the average risk

6. Gastritis—Duodenogastric reflex and reflex

gastritis are related to increased risk of

malignancy

7. Diet—Smoked fish, excessive salt intake, chilly

etc.

8. Smoking and dust ingestion

9. Deficiency of antioxidants

10. Exposure to N nitroso compounds.

Q 33. What is Borrmann classification of advanced

gastric cancer?

Once there is involvement of the muscularis, it is

called advanced gastric cancer. The macroscopic

appearance is classified by Borrmann into 4 types:

Type III and IV are commonly incurable.

Type I – Polypoidal

Type II – Ulcerative

Type III – Ulcerative and polypoidal

Type IV – Diffuse

Q 34. What is the most important epidemiological

change in gastric cancer in recent cases?

1. The incidence of gastric cancer continues to

fall at about 1% per year. This is specially seen

in relation to carcinoma of the body and distal

stomach. This may be because of the use of

refrigerator widely for food preservation.

2. In contrast there appears to be an increased

incidence of carcinoma of the proximal stomach.

Particularly the GE junction.

3. The proximal and distal cancers are supposed to

behave differently.

Q 35. What are the differences between proximal

and distal gastric cancer?

Distal gastric cancer Proximal gastric cancer

a. Diet related Not diet related

b. Environmental Not environmental

c. Epidemic variety Endemic variety

d. Arise in the background

of dysplastic mucosa

No such background

e. Intestinal histology Diffuse type

f. Better prognosis Bad prognosis (morbidity

and mortality high)

g. Resectability is 30% Resectability is 20%

h. Needs distal subtotal

gastrectomy

Needs total gastrectomy

Q 36. What is the classification of esophagogastric

junction tumors (OG junction tumors)?

Sievert classification of OG junction tumor

Type I – Esophageal cancer involving OG junction

Type II – Primary OG junction growth

Type III – Stomach lesion involving OG junction

Note: Type I needs esophageal resection with 10 cm

clearance proximally.

Q 37. What are the modes of spread of carcinoma

stomach?

Five modes of spread of carcinoma stomach

1. Horizontal spread along the stomach wall –

Submucosal spread

2. Vertical – invasion of the stomach wall and to the

adjacent structures like colon, pancreas, etc.

3. Lymphatic spread

4. Peritoneal dissemination

5. Bloodstream spread – to the liver.

Two types of horizontal extensions are met with.

a. Infiltrative growth

b. Expansive growth.

114

Clinical Surgery Pearls

Q 38. What are the preoperative preparations in

a patient with carcinoma stomach?

Preoperative preparations

a. Correction of anemia

b. Correction of nutritional status

c. Correction of fluid and electrolyte disturbance

d. Assessment of cardiac, respiratory and renal status

e. Arrange adequate blood

f. Preoperative stomach wash

g. Prophylactic antibiotic.

The stomach in normal individual is sterile

because of the low pH. Patients with gastric

cancer, however, often have an increased pH and

colonization of the stomach. There is higher risk of

wound complications in patients with high gastric

pH and heavily colonized gastric secretions. The

most common organism isolated from gastric juice

are Streptococcus faecalis, E.coli, bacteroides, S.

albus, etc. For this reason, a prophylactic antibiotic

with a 3rd generation cephalosporin is advocated.

Q 39. What are the controversies regarding the

surgical treatment of gastric adenocarcinoma?

The points of controversy are:

1. The extent of gastric resection

2. The extent of lymph node dissection.

Generally radical gastrectomies are performed

for carcinoma stomach. A radical gastrectomy is one

where the stomach along with the entire greater

omentum, lesser omentum and lymph nodes are

removed en-bloc.

1. The extent of gastric resection

The distal division line is always placed at the

duodenal bulb at least 1cm from the tumor. As

the diagnostic accuracy in infiltrative type is very

low, the macroscopic proximal margin should be

greater than 5 cm.

In the case of Scirrhous carcinoma or Borrmann

type IV, the horizontal extension includes

the whole stomach in the majority of cases

and therefore a total gastrectomy is always

indicated. In contrast, the diagnostic accuracy

is high in expansive growth type tumors and

also in early gastric cancer. A margin of 2 cm

is sufficient for these types and total or distal

radical gastrectomy can be selected.

Total versus subtotal radical gastrectomy

The term radical is vague and lacks proper

definition and may mean different operations

to different surgeons.

Surgical treatment is curative only in stage I

and II diseases. A prospective randomized study

by the French compared total versus subtotal

gastrectomy and found that the morbidity and

mortality were similar for both groups, but the

five year survival was not improved. The morbidity

and mortality appeared to be associated with

distal pancreatectomy and splenectomy.

2. The extent of lymph node dissection

The controversies regarding the extent of

lymphadenectomy relate to D1 vs D2 resection.

D2 gastrectomy has 2 essential components.

a. Adequate 5 cm clearance in the stomach

b. E x t e n s i v e l y m p h a d e n e c t o m y — t h e

omentum, superior leaf of mesocolon, and

the pancreatic capsule are removed en-bloc,

along with all N2 nodes (JRSGC) that is up to

station number 12.

In order for a patient to have pathologically

confirmed D2 lymphadenectomy more than 26

nodes has to be identified in the specimen. Much

of the mortality and morbidity in D2 resection

is related to the distal pancreatectomy and

splenectomy resulting in left subphrenic abscess.

Epigastric Lump

115

So now a days, the D2 gastrectomy will preserve

spleen and pancreas. Overall the oncological

outcome may be better following D2 gastrectomy.

The results of surgical treatment stage for stage in

Japan are much better than commonly reported in

the West and they attribute this to the staging and

quality of surgery in Japan.

In general D1 resection involves the removal

of perigastric nodes and D2 resection involves the

clearance of the major arterial trunks.

Q 40. What is the ideal treatment for operable

cancers of stomach ?

The ideal treatment consists of D2 Gastrectomy

and chemoradiation.

Q 41. What is D2 gastrectomy?

Structures removed in D2 gastrectomy

Removal of the stomach with the growth

Omental bursa

Entire greater omentum

Lesser omentum

Anterior layer of mesocolon

Anterior pancreatitic capsule

Lymphadenectomy up to D2 station.

Q 42. What are the divisions of the stomach from

therapeutic point of view?

The most appropriate operation for a given patient

with gastric cancer must take into account the

location of the lesion and the known pattern of

spread at that site. For this purpose the stomach

can be divided into 3rds.

a. The proximal 3rd includes GE junction and the

fundus.

b. The middle 3rd is the body of stomach and

extends from the fundus to the incisura angularis

of the lesser curvature.

c. The distal 3rd is the pyloric antrum and extends

from the incisura angularis to the pylorus.

Q 43. What is the surgical treatment for middle

third malignancy?

Lesions in this location are asymptomatic until

they have become quite bulky with metastases to

regional nodes. Three procedures are commonly

performed in an attempt to encompass all gross

and microscopic disease in the stomach and its

lymphatic drainage network.

1. High radical subtotal gastrectomy

2. Radical total gastrectomy

3. Extended total gastrectomy with distal

pancreatectomy and splenectomy.

As per the rules of the Japanese Research

Society, tumors encroaching upon or crossing the

line extending from the bare area on the greater

curvature (between the portions of the stomach

supplied by the gastroepiploic artery and the

areas supplied by the short gastric) to a point 5

cm below the cardioesophageal junction on

the lesser curvature required total gastrectomy.

Pathoanatomic studies have demonstrated that

carcinoma of the mid stomach metastasize to all

regional lymph node basins of the stomach. There

is, therefore, considerable theoretical basis for a

radical total gastrectomy.

Q 44. How much stomach should be removed in a

distal radical gastrectomy for a distal 3rd growth?

This involves resection of approximately 75%

of the stomach, including most of the lesser

curvature, where the margins of resection will often

be the closest. At least 1 cm of the 1st part of the

duodenum is resected. 5 cm of normal stomach

is removed proximal to the tumor to assure

adequate margin. Frozen section pathological

evaluation of the surgical margin is performed

116

Clinical Surgery Pearls

before reconstruction. As with all gastric resections

for carcinoma, greater and lesser omentectomy and

regional lymph node resection is required in an

attempt to remove all microscopic disease.

Q 45. What is the surgical treatment of proximal

gastric cancer?

Esophagogastrectomy followed by reconstruction

is the treatment of choice.

Q 46. What is the incision for a surgery for gastric

malignancy?

For all types of gastric malignancy a bilateral

subcostal incision (Chevron) is preferred.

Q 47. What is the management of inoperable

lesions?

Without complications and obstructions:

Chemotherapy

With complications: Palliative surgery and

chemotherapy.

Q 48. What are the palliative surgical procedures?

Palliative procedures for carcinoma stomach

a. Palliative gastrectomy Is indicated for uncontrolled

bleeding, obstruction and perforation

A palliative resection is usually a palliative

gastrectomy, which removes the tumor alone

followed by reconstruction

b. A palliative gastrojejunostomy—this is a poor

operation which does not allow proper emptying of

the stomach but may produce additional problem

of bile reflux

c. Palliative esophagojejunostomy

d. Palliative intubation for proximal growths

e. Recanalization procedures.

Q 49. What is combined resection?

The stomach is surrounded by pancreas, transverse

colon, mesocolon, spleen, liver, diaphragm and

omental bursa. All these structures can be surgically

removed together with stomach if necessary.

Combined resection shows survival benefits when

there are no distant metastases.

Q 50. What is para-aortic dissection?

The fatty tissue is completely removed from around

the abdominal aorta between the aortic hiatus and

origin of the inferior mesenteric artery. The left

adrenal gland is frequently removed to achieve

complete dissection around the left renal artery.

The overall incidence of metastases in this area is

approximately 3%.

Q 51. What is the management of adjacent organ

involvement?

Liver involvement : Non anatomical

 resection

Duodenum : 2 cm clearance

Esophagus : 10 cm clearance

Colon : Segmental resection

Pancreas : Distal pancreatectomy

Note: Proximal pancreas is unresectable

Q 52. What is left upper abdominal evisceration?

It is one of the extended combined resections done

in association with total gastrectomy. In addition

to total gastrectomy, pancreatosplenectomy and

transverse colectomy are minimum requirements.

Sometimes it includes left hepatectomy, left

nephrectomy, left adrenalectomy or resection of

the diaphragm. The purpose is to achieve almost

complete resection of the omental bursa which

consists of the stomach, omentum, mesocolon,

transverse colon, spleen and pancreas. The

indications for the procedures are:

1. When the tumor is located in the upper or middle

part of the stomach and the serosa is penetrated

by cancer.

2. When the tumor mainly occupies the posterior

wall of the stomach.

Epigastric Lump

117

3. When there are no remote metastases

4. When there is no severe medical complication

to prevent long aggressive intervention.

Q 53. What is the classification of early gastric

cancer (EGC)?

The EGC is classified as:

Type I Protruding

Type II Superficial

Type III Excavated.

Q 54. What are the minimally invasive procedures

and function preserving procedures?

These procedures are done for early gastric

cancer limited to the mucosa alone.

EGC affecting the submucosa needs radical

surgery.

The procedures are:

1. Endoscopic mucosal resection

2. Laparoscopic wedge resection of the stomach

3. Pylorus preserving gastrectomy.

Q 55. What are the indications for endoscopic

treatment of early gastric cancer (EGC)?

Absolute indications:

EGC small enough to be completely removed by

single endoscopic treatment. They include:

1. Protruding cancers smaller than 1 cm

2. Ulcer-free depressed cancers smaller than 1 cm

3. Well-demarked cancers.

Relative indications:

1. Cancers associated with severe medical illness

2. Occurring in elderly patients

3. Occurring in patients who refuse surgery.

Q 56. What is pylorus preserving gastrectomy?

This is indicated in small cancers located in the middle

third of the stomach. The purpose is to reduce

dumping symptoms, gallbladder dysfunction and

postoperative gallstone development. The hepatic

and pyloric branches of vagal nerves are preserved.

Q 57. What is the prognosis of carcinoma stomach?

It rarely disseminates widely before it involves the

lymph nodes and therefore there is an opportunity

to cure the disease prior to dissemination. Distant

metastases are uncommon in the absence of lymph

node involvement.

a. The prognosis of early gastric cancer is very

good. Early gastric cancer associated with lymph

node involvement has 5 year survival rates in

the region of 90%.

b. In Japan, approximately 75% of the patients

will have a curative resection and the overall

survival will be 50–70%.

c. In the West 25–50% of patients will undergo

curative resection and the five year survival is

25–30%.

Q 58. Is gastric cancer a different disease in

Japan and West? What is the stage migration

phenomenon (Will Roger’s phenomenon)?

This proposition has no basis in evidence. A

combination of differences in the staging and higher

standard of surgery in Japan probably accounts for

the difference. The more thorough staging, the higher

the stage is likely to be and therefore stage for stage,

the outcome seems better in patients adequately

staged pathologically. This phenomenon is called

stage migration (Will Roger’s phenomenon).

The pathologist will have considerable difficulty

in orientating a fixed specimen and finding lymph

node groups and therefore the surgeon should

dissect the nodes from the specimen and send

them separately to the pathologist—a practice

commonly followed in Japan. Only by this method

accurate staging can be achieved.

118

Clinical Surgery Pearls

Q 59. Is there any role for chemoradiation?

The standard adjuvant treatment is chemoradiation.

Chemotherapy is by 5-FU and Leucovorin.

Three doses of bolus 5-FU during first and last

weeks of radiotherapy.

Radiotherapy: Dose is 4,500 Rads.

Currently Epirubicin, Cisplatin and 5FU combination is favored

Q 60. Is there any role for neoadjuvant chemotherapy?

This is found to be beneficial to downstage cancer

of the lower esophagus and occasionally stomach.

Q 61. What is the most common site for recurrence

after surgery?

The most common site of recurrence is stomach

bed followed by lymph nodes and anastomotic site.

Q 62. What is the treatment for recurrence?

Self-expandable metalstents(SEMS) is used for

obstruction.

New chemotherapeutic regimens

– Cisplatin based

– Taxane based

– Irinotecan based.

Q 63. What is the follow-up after surgery?

Follow-up after surgery for carcinoma stomach

Clinical examination every three months × 2 years

CT scan/chest X-ray every 6 months

Endoscopy every year

Laparoscopy - SOS.

For PG’s—What is new?

Current ICMR guidelines for gastric cancer surgery

R0 Resection

Resection of adjacent involved structures if resectable

Minimum 15 lymph nodes should be evaluated

Splenectomy and pancreatectomy are not done routinely. Ifstation 10 nodes are involved splenectomy is done

If station 11 nodes are involved pancreatectomy may be done

Staging is done by CT, endoscopic ultrasound and staging laparoscopy. PET scan also may be done

If gross serosal disease is present there is no use of lymphadenectomy.

8 Right Hypochondrial

Lump Without Jaundice

Case

Case Capsule

A 50-year-old male patient with history of chronic

alcoholism and previous jaundice, now presents

with loss of appetite, generalized weakness

and loss of weight of 6 months duration. On

examination there is no jaundice but pallor is

present. There are no stigmata of liver disease.

On inspection of the abdomen, there is right

hypochondrial fullness. The rest of the abdomen

is normal and the abdomen moves with respiration.

There are no dilated veins and no visible peristalsis.

On palpation there is a right hypochondrial mass

that is moving with respiration. It is about 6 × 10

cm size and hard in consistency. The mass is seen

to descend from below the right costal margin. One

cannot get above the mass and fingers cannot be

insinuated between the mass and costal margin.

There is dullness on percussion over the swelling

and the dullness is continuous with liver dullness

that is dull up to the 8th rib in the midaxillary

line. The edge of the swelling is irregular and

rounded and the surface of the mass is irregular.

On auscultation a bruit is heard over the swelling.

There is no splenomegaly, there is no ascites. There

is no evidence of encephalopathy (Fig. 8.1).

Refer the checklist for examination in the previous

chapter.

1. Look for Jaundice

2. Look for stigmata of liver disease—palmar

erythema, spider nevi, testicular atrophy, absence

of pubic and axillary hair, caput medusae, etc.

3. Rule out Riedel’s lobe

4. Palpate for spleen

5. Rule out ascites

6. Look for acquired umbilical hernia in severe ascites.

120

Clinical Surgery Pearls

Q 1. What is the probable anatomical diagnosis

in this case?

It is a liver swelling.

Q 2. What are your points in favor of a liver

swelling?

Clinical points in favor of liver swelling

a. Mass in the right hypochondrium moving with

respiration

b. You cannot get above the swelling

c. Fingers cannot be insinuated between the swelling

and the costal margin

d. It is dull on percussion and the dullness is continuous

with liver dullness

e. The liver swelling will have an edge that may be

sharp or round and a surface that will be smooth

or irregular.

Q 3. What are the causes for a knobby generalized

enlargement of liver without jaundice?

Causes for generalized knobby liver enlargement

without jaundice

a. Secondary carcinoma (liver metastases)

b. Primary liver carcinoma

c. Polycystic disease

d. Macronodular cirrhosis.

Q 4. What are the causes for a knobby generalized

enlargement of liver with jaundice?

a. Extensive secondary carcinoma

b. Cirrhosis of liver.

Q 5. What are the causes for localized swellings

from liver?

Localized swelling from liver

a. Riedel’s lobe

b. Secondary carcinoma

c. Liver abscess

d. Hydatid cysts

e. Primary liver carcinoma.

Q 6. What is Riedel’s lobe?

This is an extension of the right lobe of the liver

down below the costal margin along the anterior

axillary line. It is often mistaken for a pathological

enlargement of liver or gallbladder. It is a normal

anatomical variation.

Q 7. What is the probable pathological diagnosis

in this case?

 A liver malignancy may be primary or metastases.

Q 8. What are the points in favor of a primary

malignancy in this case?

a. Chronic alcoholism that can lead onto cirrhosis

and a primary malignancy may arise in cirrhotic

liver.

Fig. 8.1: Liver mass

Right Hypochondrial Lump without Jaundice

121

b. Previous history of jaundice—hepatitis B and

hepatitis C lead on to primary malignancy.

c. Cirrhosis itself will predispose to primary

malignancy.

Q 9. What are the points against metastases in

the liver?

a. Usually multiple nodules are seen in metastases

even though multifocal hepatocellular carcinoma

is possible.

b. The classical sign of umbilication will be

palpable in the liver nodule.

c. Other clinical swellings suggestive of a primary

in another organ may or may not be palpable in

the abdominal cavity.

Q 10. What is the most common malignant

swelling in the liver?

Most common malignant swelling is metastases

in the liver. Metastatic cancer is 20 times more

common than primary tumors in the liver. The

cirrhotic liver, which often gives rise to primary

hepatic tumors, is less susceptible than normal liver

to implantation of metastases.

Q 11. What are the usual presentations of primary

malignancy of the liver?

Clinical presentation of primary

malignancy of the liver

a. Pain abdomen with or without hepatomegaly

b. Sudden deterioration of the condition of cirrhotic

patient owing to the appearance of hepatic failure,

bleeding varices or ascites

c. Sudden massive intraperitoneal hemorrhage

d. Acute illness with fever and abdominal pain

e. Distant metastases

f. No clinical finding

Q 12. Can there be associated metabolic or endocrine abnormalities in hepatocellular carcinoma

(primary)? (PG)

Metabolic and endocrine abnormalities in

hepatocellular carcinoma

a. Erythrocytosis

b. Hypercalcemia

c. Hypoglycemic attack

d. Cushing’s syndrome

e. Virilization

Q 13. What are the complications of hepatocellular

carcinoma? (PG)

a. Sudden intra-abdominal hemorrhage because

of spontaneous bleeding

b. Obstruction of the portal vein resulting in portal

hypertension

c. Obstruction of hepatic veins resulting in BuddChiari syndrome

d. Liver failure.

Q 14. What is the classification of liver tumors?

The broad classification is into benign and malignant.

Benign Malignant

a. Hepatic adenoma

(Hepatic adenoma,

Cholangioadenoma

or a combination)

A. Primary

 a. Hepatocellular

carcinoma

b. Focal nodular

hyperplasia

 b. Cholangiocarcinoma

c. Hemangioma c. Combination of

hepato and cholangiocarcinomas

 d. Adenosquamous

carcinoma

Contd...

122

Clinical Surgery Pearls

 e. Squamous carcinoma

of bile duct

 f. Signet ring carcinoma

 g. Undifferentiated

carcinoma

 h. Bile duct cystadenocarcinoma

B. Metastases

Q 15. What is the classification of metastases?

Metastases may be classified as:

a. Carcinomatous, e.g. stomach, colon, pancreas,

kidney, prostate, breast, etc.

b. Carcinoid metastases—Carcinoid syndrome

will occur when the liver is metastasized and

therefore 5HT cannot be metabolized to 5-

hydroxyindoleacetic acid.

c. Melanomatous metastases—beware of

“missing toes and glass eyes”

d. Sarcomatous metastases.

e. Lymphomatous deposits.

Q 16. What are the salient features of hepatic

adenoma?

Salient features of hepatic adenoma

Occur almost exclusively in women

Associated with the use of oral contraceptives

Well-circumscribed soft yellow tan tumors of 2–15

cm size

2/3rd are solitary and remaining are multiple

Transition from adenoma to carcinoma may occur

Pain, intra-abdominal hemorrhage and shock may

occur

Bleeding episode with menstruation is seen

Regressionispossiblewhenthe size islessthan6 cm

and when the oral contraceptive pill is withdrawn

Tumor should be followed-up periodically will

ultrasound/CT

When in doubt remove the hepatic adenoma.

Q 17: What are the stigmata of liver disease?

Stigmata of liver disease

Spider nevi (> 9 new crops indicate progressive

disease)

Palmar erythema (thenar and hypothenar

eminences)

Clubbing, leukonychia

Dupuytren’s contracture

Body hair loss (axillary and pubic)

Fetor hepaticus

Gynecomastia

Testicular atrophy

Liver flap—flapping tremor

Encephalopathy—cog wheel limb rigidity.

Q 18. How to manage this patient with liver mass?

Management includes investigations and treatment.

The investigations may be classified as:

1. Investigations for diagnosis

2. Investigations for staging

3. Investigations for surgical treatment.

*The most important investigations is imaging.

Q 19. What is the role of ultrasound for imaging

liver?

It is a screening test

Can identify mass lesions

Can identify bile duct dilatation

Useful for guiding biopsy.

Note: But it is operator dependent.

Q 20. What is the role of CT in liver mass?

Triple phase, multislice spiral CT is the investigation

of choice for liver mass.

A mass lesion up to 1 cm size can be identified

Hepatocellular carcinoma in contrast enhanced

CT shows:

Early arterial phase: vascular enhancement.

The venous phase: maps out portal vein and

hepatic veins.

Contd...

Right Hypochondrial Lump without Jaundice

123

Q 21. What are the CT findings of various mass

lesions?

Type of lesion Imaging finding Mode of

treatment

Hemangioma Hyperechoic in US

Hypodense in CT

without contrast

Halo edge

enhancement

initially, then fills

the center (late,

slow venous

enhancement)

No need for

surgery

Hepatic

adenoma

Well-circumscribed

vascular solid tumor.

Enlarged peripheral

vessels in arteriography—needle

biopsy will produce

bleeding

Premalignant

lesion.

Resection

is the treatment

of choice.Withdrawal of oral

contraceptives

will produce

regression

Focal nodular

hyperplasia

(FNH) focal

overgrowth of

functioning

liver with

fibrous

stroma

Central scarring in CT.

Well-vascularized

Sulfur colloid liver

scan is the

investigation of

choice – the

lesion includes both

hepatocytes and

Kupffer cells

No need for

surgery

Metastases Nonenhancing

lesion after IV

contrast in CT

Surgery for

colorectal

metastases

Hepatocellular

carcinoma

Early arterial phase

enhancing lesion

with IV contrast

Liver

resection/liver

transplantation

Q 22. What is the gold standard investigation for

the diagnosis of hepatocellular carcinoma?

Lipiodol enhanced CT scan is the goldstandard

investigation. The practice is an injection of lipiodol

into the hepatic artery during selective angiogram

followed by CT after 2 weeks. The lipiodol will be

taken up by the tumor.

Q 23. What is the role of magnetic resonance

imaging (MRI)?

1. It is useful for patients with iodine sensitivity,

who cannot undergo a lipiodol enhanced CT.

2. Imaging of the biliary tract (MRCP) is possible.

3. Hepatic artery and portal vein can be welldelineated.

Q 24. What is the role of needle biopsy / FNAC in

mass lesions of liver?

Percutaneous biopsy of lesions in patients with

potentially operable disease is contraindicated

because of the high incidence of needle track

recurrence and high mortality associated with this

procedure (spontaneous bleeds).

Q 25. If so what is the indication for a needle

biopsy?

1. Unresectable tumors

2. Diagnostic dilemma.

Q 26. What are the needles for liver biopsy?

1. Chiba needle (Chiba is the name of a University)

20 cm long, 22 gauge, malleable needle with

short bevel

2. Vim silverman needle

3. Menghini needle.

Q 27. What are the precautions to be taken before

biopsy from liver?

The coagulation profile should be checked and

rectified (PT, aPTT and platelet count).

124

Clinical Surgery Pearls

In cases of jaundice (in coagulation failure also)

give vitamin K injections 10 mg IM/IV for 3 days.

Note: The vitamin K1 (phytomenadione) only can

be given IV. Regular vitamin K is oily and cannot

be given intravenously.

Q 28. What are the macroscopic types of

hepatocellular carcinoma? (PG)

Macroscopic types of hepatocellular carcinoma

a. Unifocal

1.  Expanding – Sharp demarcation between the

tumor mass and compressed surrounding

parenchyma

2. Pedunculated - type 1—intrahepatic - type

2—extrahepatic (nourished by a branch of right

hepatic artery) – not a true HCC

3. Spreading (Engel’s massive form)—lack of

demarcation between tumor and normal liver—

may involve the whole liver as nodules

b. Multifocal—synchronous swellings

c. Indeterminate—different features in different parts

d. Diffuse

Another macroscopic classification:

a. Pushing

b. Hanging

c. Infiltrative or diffuse

NCCN classification:

a. Nodular

b. Massive

c. Diffuse

Q 29. What are the pathological variants of

HCC? (PG)

1. Fibrolamellar HCC—good prognosis

2. Mixed hepatocellular cholangiocellular—worse

prognosis

3. Clear cell variant—better prognosis

4. Giant cell variant

5. Childhood HCC (bad prognosis)

6. Carcinosarcoma (sarcomatoid variant).

Q 30. What are the features of fibrolamellar

carcinoma?

1. Younger age group (median 25 years)

2. Without history of cirrhosis

3. Well-circumscribed

4. Betterresectability (50 – 75%incontrastto25%

in ordinary HCC)

5. Better prognosis

6. Left lobe is more commonly affected

7. Lymph node involvement is there

8. Does not produce AFP (α fetoprotein)

9. Neurotensin B and vitamin B12 binding

proteins are the tumor marker

10. Equal sex incidence

11. CT finding—stellate scarring.

Q 31. What are the etiological factors for HCC?

1. Cirrhosis—Cirrhomimetic (develops in the

background of cirrhosis) Noncirrhomimetic

2. HBV (hepatitis B virus)

3. HCV (hepatitis C virus)

4. Alcoholic liver disease

5. Hemochromatosis

6. Hereditary tyrosinemia

7. Mycotoxins (afla toxin B)

8. Hepatic helminthiasis – clonorchis sinensis for

cholangiocarcinoma.

Q 32. What is the tumor marker for the HCC?

1. AFP (α fetoprotein)

2. PIVKA II (protein induced by vitamin K

abnormality by antagonism) – it is positive in

80% HCC

3. Lens culinaris Agglutinin reactive AFP (isoform

of alpha fetoprotein)

Q 33. What is small HCC? (PG)

A tumor that is < 2 cm as per Japanese General

Rules. But a tumor < 3 – 5 cm may be considered

Right Hypochondrial Lump without Jaundice

125

small by some authorities. They are nodular and

well-differentiated.

Q 34. What are the investigations for staging?

1. CT abdomen with oral and IV contrast

2. CT chest—to rule out metastases to lung

3. Bone scan—to rule out metastases

4. Laparoscopy.

Q 35. What is the role of Child-Pugh grading?

This is very important if one is planning for surgical

resection. Resection should be contemplated in

Child-Pugh A patients and occasionally in ChildPugh B patients.

Child-Pugh classification of functional

status of liver

Class: A

Risk: Low

Score 1

B

Moderate

Score 2

C

High

Score 3

Ascites Absent Slight to

moderate

Tense

Encephalopathy None Grades I – II Grades

III – IV

Serum albumin

(g/dL)

> 3.5 2.8 – 3.5 < 2.8

Serum bilirubin

(mg/dL)

< 2.0 2.0 – 3.0 > 3.0

Prothrombin

time (seconds

above control)

< 4.0 4.0 – 6.0 > 6.0

A – 5 to 6, B – 7 to 9, C – 10 to 15

Q 36. What are the surgical options available in

HCC?

The two options are:

1. Resection

2. Liver transplantation.

Q 37. What are the investigations required for

surgery?

Liver function test

Assessment of cardiac status

Pulmonary status

Renal status.

Q 38. What is the TNM staging? (PG)

Tx Primary tumor cannot be assessed

T0 No evidence of primary

T1 Solitary tumor without vascular invasion

T2 Solitary tumor with vascular invasion or

multiple tumors none more than 5 cm

T3a Multiple tumors more than 5 cm

T3b Single tumor or multiple tumors of any size

involving a major branch of portal vein or

hepatic vein

T4 Tumor(s) with direct invasion of adjacent

organs other than the gallbladder or with

perforation of visceral peritoneum

Regional nodes:

Nx Regional node cannot be assessed

N0 No regional node metastasis

N1 Regional node metastasis

Metastasis

M0 No distal metastasis

M1 Distal metastasis

Q 39. What is MELD (Model for End-Stage Liver

Disease) score? (PG)

The following factors are considered for MELD

Creatinine

Bilirubin

INR.

Q 40. What are the bad prognostic factors? (PG)

In addition to MELD, alpha fetoprotein, fibrosis score

and hepatitis serology

126

Clinical Surgery Pearls

Q 41. What is Okuda staging? (PG)

Four factors are considered in this staging:

Tumor size

Ascites

Albumin

Bilirubin.

Q 42. What is CLIP (Cancer of Liver Italia Program)

staging? (PG)

It consists of:

Child-Pugh and Alpha Fetoprotein and Portal vein

thrombosis and Morphology

Q 43. What is CUPI (Chines University Prognostic

Index)? (PG)

The following are considered in this:

TNM and ascites and Bilirubin and ALP and Symptoms

Q 44. What are the indications for resection in

HCC? (PG)

Milan criteria:

1. Tumor less than 5 cm

2. Number less than 3 cm

3. Each less than 3 cm

University of California Sanfrancisco Criteria:

1. Single tumor less than 6.5 cm

2. Less than three tumors

3. Total diameter less than 8 cm

4. Largest less than 4.5 cm

Note: More than 30%ofremnantliver must be kept.

At least two contiguous liver segments must be

retained and it should be a resection with tumor

free negative margin.

Q 45. What are the contraindications for liver

resection in HCC? (PG)

1. Extrahepatic disease

2. Lymph nodes

3. Peritoneal involvement

4. Distant metastases

5. 1 cm clear margin is not possible

6. Multifocal hepatoma (> 2 nodules)—Treatment

is transplantation.

Q 46. How much liver should be left behind in case

of liver resection? (PG)

Two segments of healthy liver must be left behind.

Q 47. What type of anesthesia is preferred for liver

resection? (PG)

Low CVP anesthesia

The inflow to the liver can be blocked by clamping

the free edge of the lesser omentum

Bleeding resultsfrom hepatic vein during resection

If the CVP is decreased to zero, there will be little

bleeding from hepatic veins

As a result, there is no need for blood transfusion.

Q 48. What are the indications for transplantation?

 (PG)

Outlook is better for transplantation than resection

provided:

Tumor nodules are 3 cm or less in diameter

Nodules are 3 or less in number.

Q 49. What are the nonsurgical ablative therapies

available? (PG)

1. Radiofrequency ablation—complications are

needle track recurrence, hemorrhage and liver

failure.

2. Percutaneous alcohol injection—destruction of

tumors up to 3 cm size is possible—ineffective

in colorectal metastases.

3. TACE (Transartenal Chemo Embolization)—for

small tumours only. A mixture of lipiodol and

doxorubicin is injected directly into the hepatic

artery. The complications are post embolization

Right Hypochondrial Lump without Jaundice

127

syndrome, liver failure, and liver abscess. 50%

reduction in tumor size is seen 50% of cases.

4. TARE (Transarterial Radioembolization).

Q 50. What is the role of chemotherapy in HCC?

 (PG)

The results of chemotherapy are disappointing.

Therefore, there is no role for systemic chemotherapy. Subcutaneous Octreotide is being tried

in advanced HCC.

Q 51. What is the prognosis of HCC?

It is a tumor with poor prognosis

Median survival is in the region of 1 year

Symptomatic patients survive for 6 months

Resectedpatients(ifsuitable)the 5 yearssurvival

is around 30%

Transplantation gives better results

Life-expectancy is decided by the liver disease.

9 Right Iliac Fossa Mass

(Suspected Ileocecal Tuberculosis)

Case

Case Capsule

A 40-year-old female patient presents with colicky

abdominal pain with intermittent diarrhea of 1

year duration. The pain is exacerbated by eating

and relieved by vomiting. The stool is watery,

small in amount and mixed with blood. She has

got flatulence and borborygmi with frequent

distension of abdomen. She also gives history of

loss of appetite and weight loss. She gets mild

fever towards the evening. There is a history

of exposure to tuberculosis (her father-in-law

had treatment for pulmonary tuberculosis). On

general examination, she has pallor and she is

emaciated and malnourished. The abdomen

shows visible peristalsis and distended small

bowel loops. The abdomen is found to have

“doughy” feel on palpation and there is a mass

situated in the right iliac fossa extending to the

lumbar region of 12 × 8 cm size which is firm in

consistency. There is no intrinsic mobility for the

mass. All the borders are well-defined except

the lateral border. There is no evidence of free

fluid in the abdomen. There are no other masses.

No supraclavicular lymph nodes. There is no

evidence of pulmonary tuberculosis on chest

examination (Fig. 9.1).

Refer the checklist for examination of abdominal lump.

Checklist for right iliac fossa mass

Look for a primary growth in the drainage area of

inguinal and iliac nodes (it may be a metastatic iliac

group of nodes) including the genitalia, perineum,

gluteal region and a per rectal examination

Fig. 9.1: Right iliac fossa mass

Contd...

Right Iliac Fossa Mass (Suspected Ileocecal Tuberculosis)

129

Look for edema of the right lower limb—due to

compression of iliac vein by a malignant lymph

node mass

Examine the spine—to rule out a gibbus as a result

of tuberculosis (the mass may be an iliopsoas cold

abscess)

Rectal examination to rule out rectal shelf of

Blummer and rectal growth

Examination of genitaliato rule out undescended

testes

Examine for left supraclavicular nodes

Look for swelling below the inguinalligament,

if the right iliac fossa mass is soft. Look for cross

fluctuation above and below the inguinal ligament.

The psoas abscess will track down to the thigh

behind the inguinal ligament

In case of a soft swelling look for a space between

the swelling and ilium. The space will be there in

appendicular abscess and it will be absent in iliac

abscess

Examine the sacroiliac joint—tuberculous abscess

from sacroiliac joint can present as right iliac fossa mass

There are innumerable causes for mass in the right

iliac fossa.

The important causes for mass right iliac fossa

Appendicular mass

Ileocecal tuberculosis

Carcinoma cecum

Right ovarian cyst/tumor

Pelvic inflammatory disease

Iliac lymphadenopathy

Psoas abscess

Tumor in undescended testis

Unascended kidney

Retroperitoneal tumors

Chondrosarcoma of the ilium

Crohn’s disease

Actinomycosis of the cecum

Tubo-ovarian mass.

Q 1. What are the clinical characteristics of a

swelling arising from the cecum?

Even though cecum has a complete peritoneal

covering, it is relatively fixed and therefore the

mass arising from the cecum will be fixed. There

will be no intrinsic mobility. The mass will be

intraabdominal and intraperitoneal. One can get

above and below the swelling and there won’t be

any movement.

Q 2. What are the characteristic features of

appendicular lump?

There will be typical history—the Murphy’s triad –

migratory pain (from umbilical region to the right

iliac fossa), vomiting/nausea and fever. The right

iliac fossa mass will be tender. The duration of

the symptoms and the lump will be short. In late

stages there will be local rise of temperature and

reddening of the overlying skin. The patient may

also have intermittent fever and tachycardia.

Q 3. What are the characteristic features of

tuberculosis?

The patient often has central abdominal pain for

months with general ill health, weight loss, and

change in bowel habit. The mass is firm, the surface

and edges are difficult to define. Ascites may or may

not be there. The typical doughy feel may be there.

The patient may have evening rise of temperature.

Q 4. What are the characteristic features of

carcinoma cecum?

There won’t be any history of acute pain. Some

patients present with anemia, diarrhea, or intestinal

obstruction. The right iliac fossa mass will be firm,

or hard, distinct and fixed it is not tender, does not

resolvewith observation.The patient’stemperature

and pulse are normal.

Contd...

130

Clinical Surgery Pearls

Q 5. Comparison of the clinical features of three

most important causes for right iliac fossa mass?

Appendicular

lump

Ileocecal

tuberculosis

Carcinoma

cecum

Age Younger

age group

Middle age Above 50

years

Onset Acute – 3

days after

the initial

symptoms

Subacute

- recurrent

attacks

Insidious/

asymptomatic

Symptoms Murphy’s

triad

• Migratory

pain

• Nausea/

vomiting

• Fever

Recurrent

attacks of

abdominal

pain,

diarrhea,

loss of

weight,

evening

rise of

temperature

Lump

abdomen

Nature of

mass

Intrinsic

movement

Irregular,

firm and

tender

Fixed

Irregular,

doughy

Fixed

Hard

Fixed

Response to

antibiotic

Become

smaller and

then

disappears

No response No

response

Radiological

signs

No specific

radiological

signs

Pulled up

cecum

Fleischner

sign, Sterlin

sign, obtus

eileocecal

angle

Irregular

filling

defects in

the

cecum

which is

in the

normal

position

Q 6. What is the most probable diagnosis in this

case?

May be a case of ileocecal tuberculosis, one should

rule out carcinoma cecum.

Q 7. What are the points in favor of ileocecal

tuberculosis?

The long duration of symptoms

Recurrent attacksofpainabdomen(centraltype)

History of diarrhea

Loss of weight

Evening rise of temperature.

Q 8. How to confirm the diagnosis?

The investigations for confirming the diagnosis are:

1. ESR—will be raised in tuberculosis (complete

Hemogram is done)

2. Leukocyte count—Leukopeniawithlymphocytosis

3. Hemoglobin—Anemia may be present

4. Mantoux test

Mantoux is highly nonspecific

Mantoux can be negative in severely immunocompromised individuals

It canbepositive inBCGvaccinatedindividuals.

5. X-ray chest—May show pulmonary tuberculosis

6. Plain X-ray abdomen—May show calcified

lymph nodes

Evidence of intestinal dilatation

7. Ultrasound abdomen—May show mass

lesion, fluid collection, enlargement of lymph

nodes, etc. A tense tender tubular structure of

more than 7 mm if identified is suggestive of

appendicular mass, along with fluid collection

8. Double contrast barium enema—For visualizing

the cecum barium enema is the investigation

of choice. For visualizing the terminal ileum a

barium meal follow through is the investigation

of choice.

Right Iliac Fossa Mass (Suspected Ileocecal Tuberculosis)

131

9. CT scan if required

10. Peritoneal biopsy

a. Needle biopsy (not safe)—Abram’s

needle/Cope’s needle.

b. Laparoscopic biopsy (ideal)—Peritoneum

loses it s glistening and smooth

appearance. Becomes dull, rough and

irregular.

c. Open biopsy (under local anesthesia).

11. Ascitic fluid study

a. Adenosine De Aminase (ADA)—more than

32 units L—in ascitic fluid—indicatesthe

degree of stimulation of lymphocytes (not

significant in HIV patients and cirrhotic

patients)

b. Ascitic fluid total protein—more than 2.5

gm/L

c. SAAG (Serum ascitic fluid albumin

gradient)—more than 1.1

d. Ascitic fluid blood glucose ratio of less

than 0.96.

12. PCR.

Q 9. What are the radiological findings in favour

of tuberculosis in barium meal follow through?

1. The cecum will be pulled up (normally it is seen

in relation to the bone ilium)

2. Fleischner sign—Thickening of ileocecal valve

and it is wide open

3. Sterlin sign—Fibrotic terminal ileum opening

into a contracted cecum

4. Obtuse ileocecal angle.

Q 10. Which part of the intestine is commonly

affected by tuberculosis?

It can affect any part of the GI tract. The commonly

affected areas are ileum and proximal colon (57%).

Q 11. Why ileocecal region is more affected in

tuberculosis?

a. More lymphatic follicles are seen in terminal

ileum

b. Stasis in terminal ileum

c. Spread from adjacent fallopian tube (genital TB

will spread via fallopian tube) in females.

Q 12. What are the other forms of abdominal

tuberculosis?

Types of abdominal tuberculosis

a. Intestine

b. Peritoneum (38%)—6 types

1. Chronic peritonitis

2. Acute peritonitis

3. Ascitic type

4. Encysted type

5. Fibrous type

6. Purulent type

c. Mesentery

d. Lymph nodes (6%)

e. Omentum

f. Liver and spleen.

Q 13. What are the pathological types of intestinal

tuberculosis?

Four pathological types of intestinal tuberculosis:

1. Ulcerative

2. Hypertrophic

3. Fibrotic

4. Ulcerofibrotic.

The first two account for majority of cases.

132

Clinical Surgery Pearls

Q 14. What are the differences between ulcerative

type and hypertrophic type of tuberculosis?

Q 15. What is the mechanism of involvement of

peritoneum, mesenteric nodes and omentum?

They are involved by the following mechanisms:

a. Involvement during the bacteremic phase of

pulmonary tuberculosis

b. From adjacent organ fallopian tube

c. From intestinal tuberculosis.

Note: When nodes and intestine nodes are involved,

the node is considered primary because the earliest

intestinal lesion is found to be submucosal.

Q 16. What are the complications of iliocecal

tuberculosis?

1. Intestinal obstruction

2. Perforation.

Ulcerative Hypertrophic

1. Secondary to pulmonary tuberculosis by swallowing

of tubercle bacilli

Itis primary—due to ingestion of low virulent organisms

by a person with high resistance

2. Patient is very ill Not very ill

3. Multiple transverse ulcers in the ileum* Thickening ofintestinalwall and narrowing of the lumen

4. Clinical features—Diarrhea and bleeding, weightloss,

night sweats, anorexia and evening rise of temperature

Mass right iliac fossa, intestinal obstruction, doughy

feel, matted coils of intestine, anemia steatorrhea and

weight loss

5. A primary will be there in the chest No primary in the chest

6. Absence of filling of ileum, cecum and ascending colon Filling defect in the terminal ileum, pulled up cecum,

obtuse ileocecal angle

7. Presence of gross caseation Absence of gross caseation

8. Obstruction not seen usually Obstruction common

*Crohn’s and typhoid are longitudinal ulcers

Q 17. What is abdominal cocoon?

The plastic adhesions in abdominal tuberculosis,

completely obliterate the peritoneal cavity forming a

cocoon. Here the intestines are encased in a sheath.

Q 18. What is the importance of pelvic examination

in abdominal tuberculosis?

1. To identify the associated genital TB in female

2. To take endometrial biopsy.

Q 19. What is the type of stricture in intestinal TB?

Napkin-ring stricture.

Q 20. What is the treatment for intestinal

tuberculosis?

If the diagnosis is certain and in the absence of

obstruction, antituberculous treatment alone is

required.

Right Iliac Fossa Mass (Suspected Ileocecal Tuberculosis)

133

Antituberculous regime

Drug Adult dose Bactericidal/Bacteriostatic Important toxicity

1. Rifampicin 450–600 mg/day

(Before food) orally

Bactericidal Hepatotoxicity

(LFT monthly)

2. INH 300 mg OD orally Bactericidal Neuritis(Give pyridoxine 10 mg/day)

3. Ethambutol 800 mg OD after food Bacteriostatic Retrobulbar neuritis (Check vision)

4. Pyrazinamide

(25 mg/kg/day)

750 mg BD after food Bacteriocidal, acts on caseous

materials, macrophages

Hepatotoxicity

Note: All four drugs are given for first two months. Next four months 1 and 2 are given.

Rifampicin can cause failure of oral contraception. It interferes with kinetics of the estrogen component.

Note: Pyridoxine 2 mg for 100 mg of INH is given to

mother and infant.

Q 24. What is the treatment of choice when there

is obstruction?

a. Ileocecal resection in the form of a right

hemicolectomy, when the disease involves ileum

and cecum. When there is doubt regarding the

nature of the mass, resection is recommended.

b. Strictureplasty when the lesion is confined to

the ileum and the numbers of strictures are

isolated and limited.

Q 25. If the investigations are found to be negative

for tuberculosis, how one should proceed?

Carcinoma of the cecum should be ruled out by

the following investigations:

1. Digital examination of the rectum

2. Rigid/fiberoptic sigmoidoscopy

3. Colonoscopy to rule out synchronous lesions

4. Carcinoembryonic antigen (CEA)—this will be

elevated in cancer colon

5. Ultrasoundabdomentoruleoutlivermetastases

6. CT of the abdomen to find out the extent of

involvement of the surrounding structures

7. X-ray chest to rule out pulmonary metastases.

Q 21. How will you classify antituberculous drugs?

Primary agents

(Bactericidal)

Secondary agents

(Bacteriostatic)

Minor agents

(Bacteriostatic)

INH Ethambutol Kanamycin

Rifampicin Ethionamide Thiacetazone

Pyrazinamide Cyclosporine

Streptomycin Viomycin

Q 22. How will you classify Mycobacterium tuberculosis?

They are classified as:

Rapidly dividing—Susceptible to bacteriocidal

drugs

Intermittent metabolizers (not more than a few

hours)—Killed by Rifampicin because of the

rapidity of action

Organismsin the acid environment of macrophages

- Killed by Pyrazinamide

Some are dormant and not affected by drugs

Note: The INH will take one day for the action and

therefore not effective for intermittent metabolizers.

Q 23. What are the drugs secreted in the milk?

Rifampicin

Pyrazinamide.

10 Suspected Carcinoma

of the Cecum

Case

Case Capsule

A male patient of 50 years presents with abdominal

pain, anorexia and loss of weight of eight months

duration. It is a dull ache in the right iliac fossa.

He also complaints of melena. There is no history

of fever or vomiting. There is no family history of

any malignancy. On examination the patient is thin

built and pale. There is fullness of the abdomen in

the right iliac fossa region. On palpation there is a

hard, irregular nontender mass which is fixed, of

the size of 16 × 12 cm size extending to the lumbar

region. All the borders are well-delineated except

the lateral border. It is possible to get below the

swelling. The mass is dull to percussion. The liver is

not palpable. There is no other palpable mass. There

is no evidence of free fluid. The bowel sounds are

normal. The left supraclavicular lymph nodes are

not enlarged. The rectal examination revealed feces

with blood staining.

Checklist for examination:

Take a thorough family history

Digital rectal examination

Left supraclavicular nodes

Look for lumps in the breast

Do vaginal examination to rule out endometrial

and ovarian malignancies

Look for sebaceous cyst

Look for osteomas in skull, mandible and tibia

Look for desmoid tumor in the abdomen

Look for lipomas and fibromas

Look for pigmented spots in retina.

Q 1. What are the diagnostic points in favor of

carcinoma cecum?

1. Hard mass in the right iliac fossa

2. The mass is fixed

3. Appears to arise from the cecum (intra-abdominal)

4. Anemia

5. Melena.

Q 2. What is the importance of family history in

carcinoma colon?

1. Familial adenomatous polyposis (FAP) is an

inherited, autosomal, dominant condition

affecting the chromosome 5 p 21 (APC GENE)–

presenting as multiple adenomatous polyposis

mainly involving large bowel, but it can also

involve stomach duodenum and small intestine.

It is clinically defined by the presence of more

than 100 colorectal adenomas.

2. HNPCC (Hereditary nonpolyposis colorectal

cancer) is also autosomal dominant problem

where patient presents with colon cancers at

an early age. It is also associated with cancers

Suspected Carcinoma of the Cecum

135

of the endometrium, ovary, stomach and small

intestine.The mutation isin the MLH1 and MSH2

genes. (It is also called Lynch Syndrome).

Q 3. What are the syndromes associated with

polyps?

1. Gardner’s syndrome—

FAP

Desmoid tumour of abdominal wall

Osteoma of skull, mandible, tibia

Epidermal cyst

Fibroma

Lipoma

Bone cyst

Impacted molars.

2. Turcot’s syndrome—It is autosomal recessive

Polyposis

Childhood cerebellar medulloblastoma.

Q 4. What is the risk for malignancy in FAP?

1. All patients will develop colorectal carcinoma

10 to 20 years after the onset of polyp.

2. Inadditionthere is a riskofduodenal carcinoma,

ampullary carcinoma.

Q 5. What are the clinical features of FAP?

They may be symptomatic or asymptomatic

Symptoms:

Loose stools

Lower abdominal pain

Weight loss

Diarrhea

Blood and mucus with stools.

Q 6. What are the investigations done to rule out

FAP?

Sigmoidoscopy

Double contrast barium enema

Colonoscopy and biopsy.

*More than 100 adenomas must be there to make

a diagnosis of FAP.

Q 7. What is the screening policy for FAP? (PG)

1. The screening is by colonoscopy:

All members ofthe family (cousins, nephews,

and nieces) are screened at the age of 10 to

12 years and thereafter once in 1–2 years. This

should be continued upto the age of 20 years.

If no polyps till the age of 20, continue

screening at 5 yearly intervals until 50 years.

At 50 years if there are no polyps—then no

inherited gene is present.

2. Pigmented spots in retina (Congenital

hypertrophy of retinal pigment epithelium > 4

in number on each eye).

3. DNAtestfor FAP (Genetic testing for atrisk family

members).

Q 8. Can you prevent carcinoma in FAP? (PG)

At present, the only means of prevention is by

prophylactic resection of the colon—total

proctocolectomy and ileoanal pouch anastomosis.

Q 9. Is there any role for upper GI endoscopy in

FAP? (PG)

Yes. The patient can develop duodenal ampullary

and stomach malignancies and therefore upper GI

endoscopy is indicated.

Q 10. Is there any medical treatment for polyp? (PG)

Sulindac and Celecoxibare being tried.

Q 11. What are the features of HNPCC?

It is also an autosomal dominant condition affecting

the chromosome 2P (short arm of 2) and there are

2 syndromes associated with this—

a. Lynch I syndrome—this produces multiple

colonic cancers in the proximal colon at an early

age (hereditary site specific colon caner)

b. Lynch II syndrome—here in addition to colon

cancer extracolonic adenocarcinomas affecting

(Cancer family syndrome)the following organs

are seen:

136

Clinical Surgery Pearls

Breast, ovary, endometrium, pancreas,stomach,

bile duct, ureter and renal pelvis, etc.

Q 12. What is Amsterdam criteria for HNPCC?(PG)

Amsterdam criteria 1 (Lynch 1)—at least 3 relatives

must have histologically verified colorectal cancer.

1. One must be a 1st degree relative of the other

two

2. At least two successive generations must be

affected

3. At least one of the relatives with colorectal

cancer must have received the diagnosis before

the age of 50 years.

Amsterdam criteria 2 (Lynch 2)—at least 3 relatives

must have a cancer associated with hereditary

nonpolyposis colonic cancer namely colorectal,

endometrial, ovarian, stomach, pancreas, hepatobiliary, etc.

1. One must be a 1st degree relative of the other

two

2. At least two successive generations must be

affected

3. At least one of the relatives with HNPCC

associated cancer must have received the

diagnosis before the age of 50 years.

Q 13. What is synchronous carcinoma?

Inall casesofsuspectedcolorectal cancerlook forthe

presence of a second malignancy in another location

in the colon. The development of simultaneous

cancer is called synchronous carcinoma. If it is

developing after the resection of one malignancy

then it is called metachronous cancer. Therefore,

even after identification of one malignancy the

rest of the colon must be studied thoroughly by

colonoscopy and double contrast barium enema

to rule out a second malignancy (Synchronous).

Q 14. What are the macroscopical types of

colorectal cancer?

There are 4 types:

Type 1 – Annular

Type 2 – Tubular

Type 3 – Ulcerative

Type 4 – Cauliflower—least malignant.

Q 15. Which type of lesion is seen on the right

side of colon?

Ulcerative and polypoidal lesions are seen on the

right side.

Q 16. What are the investigations for making a

diagnosis in this case?

A. Investigations for diagnosis:

1. Rigid sigmoidoscopy

2. Flexible sigmoidoscopy (60 cm length)

3. Colonoscopy—can visualize the growth and

take biopsy.

Can demonstrate synchronous carcinoma

(seen in 5%) and polyps

4. Double contrast barium enema—

– irregular filling defects

– apple core appearance

– shouldering

– demonstrate polyps

B. Investigations for staging:

1. Chest X-ray

2. LFT

3. Ultrasound

– for liver metastases

– for free fluid

for knowing the extent ofthe caecal mass

– obstruction of ureter and subsequent

hydronephrosis

4. CECT (contrast enhanced)

– for local invasion

– for adjacent structure involvement

Suspected Carcinoma of the Cecum

137

5. CEA

C. Investigations for surgery:

1. Cardiac status

2. Renal status

3. Hematological investigations.

Q 17. What is virtual colonoscopy?

High resolution 3D images of the colon can be

made with a multislice CT scanner with improved

software. It has a sensitivity and specificity of 94%.

There is no need for sedation as in the case of

colonoscopy.

Q 18. What are the methods of spread of carcinoma

colon?

1. Local spread—may be vertical, horizontal and

radial.

Vertical is to the adjacent structures and horizontal

is submucosal. Radial is circumferential.

a. It will spread around—it takes one year for

spreading 3/4th circumference and result in

intestinal obstruction.

b. Ulcerated type produce fistulization to

urinary bladder

Local perforation

External fecal fistula.

2. Lymphatic spread

Read the lymphatic drainage (below).

3. Bloodstream spread

Produce occult hepatic metastases

– Responsible for late death

Metastasesin lung, liver,bone,brain and skin.

4. Transperitoneal spread

Peritoneal metastases—Ascites

Carcinomatosis peritoneum

Rectal shelf of Blumer.

5. Intraluminal spread—This is responsible for

anastomotic site recurrence.

Q 19. What are the predisposing causes for

colorectal cancer?

Predisposing causes for colorectal cancer

Ulcerative colitis—Inflammatory bowel disease of

> 10 years

Crohn’s colitis

Schistosomal colitis

Exposure to radiation

Ureterosigmoidostomy

Breast carcinoma (small increased risk)

Gallstone (postchole cystectomy-increased risk for

right colonic malignancy)

Diet—fiber diet protects, cooked meat—carcinogenic, bile acid—carcinogenic.

Q 20. What is the commonest site for colorectal

cancer?

Incidence of colorectal cancer in various sites

Maximum incidence in rectum 38%

Sigmoid 21%

Cecum 12%

Transverse colon 5.5%

Ascending colon 5%

Descending colon 4%

Splenic flexure 3%

Hepatic flexure 2%

Q 21. What are the types of adenomatous polyps

of colon?

There are 3 types

1. Tubular – 5% chance for malignancy

2. Tubulovillous – 22% chance for malignancy

3. Villous – 40% chance for malignancy

Q 22. What is the significance of adenomatous

polyp and how will you manage polyp?

Adenomas are premalignant

They are usually multiple

138

Clinical Surgery Pearls

Synchronous cancer may occur

Adenomas > 5 mm size should be removed

because of malignant potential.

Q 23. What is the peculiarity of villous adenoma?

They are sessile frond-like spreading lesion.

They produce mucus discharge, diarrhea and

hypokalemia. Unlike other polyp, removal by

colonoscopic snaring and diathermy obliteration

with hot biopsy forceps are not possible in villous

adenoma. Proctectomy may be required.

Q 24. What is adenoma-carcinoma sequence? (PG)

Fearon-Vogelstein Adenoma-carcinoma multi-step

model of carcinogenesis

Normal epithelium

↓← 5q loss

Dysplasia

Early adenoma

↓←12p activation K-ras

Intermediate adenoma

↓←18q loss

Late adenoma

↓←17p loss, p53

Carcinoma

Metastases

Q 25. What is the staging for carcinoma colon?

Duke’s classification originally described for rectum

is used for colon with modification.

Duke’s A: Tumor restricted, but not through the bowel

wall

Duke’s B: Indicating penetration through the bowel

wall

Duke’s C: Spread to local and regional nodes

 C1: Nodes in the immediate vicinity

 C2: Along the vessels

Duke’s D: Distant metastases.

The staging system by Astler and Coller is a

modification of Duke’s.

TNM-AJCC 6th edition is the staging system

followed now.

Q 26. What is the AJCC 7th Edition - TNM staging?

 (PG)

Tx Primary tumor cannot be assessed

T0 No evidence of primary tumor

Tis Carcinoma in situ, intraepithelial or invasion of

lamina propria

T1 Tumor invades submucosa

T2 Tumor invades muscularis propria

T3 Tumor invades through the muscularis propria,

into the subserosa or into non-peritonealized

pericolic or perirectal tissues

T4a Tumor penetrates to the surface of the visceral

peritoneum

T4b Tumor directly invades or is adherent to other

organs or structures

Nx Regional lymph nodes cannot be assessed

N0 No regional lymph node metastases

N1 Metastases in 1 to 3 nodes

N1a Metastasis in 1 regional lymph node

N1b Metastases in 2–3 regional lymph nodes

N1c Tumor deposits in the subserosa, mesentery, or

non-peritonealized pericolic or perirectal tissue

without regional nodal metastases

N2 Metastases in 4 or more nodes

N2a Metastases in 4 - 6 regional lymph nodes

N2b Metastases in 7 or more regional lymph nodes

Mx Distant metastases cannot be assessed

M0 No distant metastases

M1 Distant metastases

M1a Metastases confined to 1 organ or site (e.g. liver,

lung, ovary, non regional node)

M1b Metastasis in more than 1 organ/site or the

peritoneum

Suspected Carcinoma of the Cecum

139

Q 27. Comparative chart showing TNM, Duke’s and

modified Astler-Coller staging? (PG)

Stage T N M Dukes MAC

0 Tis N0 M0

I T1 N0 M0 A A

T2 N0 M0 A B1

IIA T3 N0 M0 B B2

IIB T4a N0 M0 B B3

IIC T4b N0 M0

IIIA T1-T2 N1/N1c M0 C C1

T1 N2a M0

IIIB T3-T4a N1/N1c M0 C C2/C3

T2-T3 N2a M0

T1-T2 N2b M0

IIIC T4a N2a M0 C C1/C2/C3

T3-T4a N2b M0

T4b N1-N2 M0

IVA Any T Any N M1a D D

IVB Any T Any N M1b

Q 28. What are the differences between right and

left sided colonic malignancy?

Right sided Left sided

1. Right side colonic

diameter is large

Narrow

2. On right side the

content is fluid (fecal)

Content is solid

3. Growth is ulcerated/

cauliflower like

Stenosing growth

4. Present with pallor Present with intestinal

obstruction

5. Ill defined pain Colicky pain

6. Bowel symptom -

Melena

Alternating constipation

and diarrhea

7. Late diagnosis Because of obstruction

early diagnosis

8. For right colectomy

only the branches of

superior mesenteric are

ligated

Flush ligation of inferior

mesenteric artery done

for left colectomy

9. Bad prognosis because

of late diagnosis

Good prognosis because

of early diagnosis

Q 29. What is the cause for alternating constipation

and diarrhea in left sided colon cancer?

The left sided cancers are usually tubular or annular

producing obstruction. As a result of obstruction

therewillbeconstipation.Subsequenttoconstipation

there will be stasis of fecal matter proximal to the

stenosis. This will produce stercoral enteritis which

in turn will produce diarrhea—the so-called spurious

diarrhea where the liquid fecal matter will escape

through the narrow segment. Again the patient will

go in for constipation. This is a vicious cycle.

Q 30. What is the site of perforation of the colon

in cases of obstruction?

The perforation can occur in the following places.

a. At the site of the growth

b. Just proximal to the growth as a result of

stercoral enteritis

c. At the cecum for any distal growth if the ileocecal

valve is competent.

Q 31. What are the acute manifestations of colonic

cancer?

Intestinal obstruction

Perforation

Peritonitis

Bleeding

Contd...

Contd...








clinical surgery pearls

clinical surgery pearls 3rd edition pdf

clinical surgery pearls 3rd edition pdf free download

clinical surgery pearls dayananda babu pdf

clinical surgery pearls pdf

clinical surgery pearls pdf google drive

clinical surgery pearls latest edition

clinical surgery pearls 2nd edition pdf free download

clinical surgery pearls pdf free download

clinical surgery pearls 3rd edition free pdf

clinical surgery pearls and co




clinical surgery pearls and beauty

clinical surgery pearls and skin

clinical surgery pearls clinic

clinical surgery pearls canada

clinical surgery pearls course

clinical surgery pearls clinical

clinical surgery pearls clinical trials

clinical surgery pearls dayananda babu pdf free download

clinical surgery pearls pdf download

clinical surgery pearls hotel

clinical surgery pearls hospital

clinical surgery pearls harvard

clinical surgery pearls harris

clinical pearls surgery

clinical surgery pearls journal

clinical surgery pearls japan

clinical surgery pearls jewellery

clinical surgery pearls jewelry

clinical surgery pearls kit

clinical surgery pearls ksa

clinical surgery pearls ks

clinical surgery pearls meaning

clinical surgery pearls mdpi

clinical surgery pearls medical

clinical surgery pearls md

clinical surgery pearls mcgill

clinical surgery pearls near me

clinical surgery pearls ncbi

clinical surgery pearls nursery

general surgery pearls of wisdom pdf

clinical surgery pearls qatar

clinical surgery pearls questions

clinical surgery pearls quiz

clinical surgery pearls review

clinical surgery pearls reviews

clinical surgery pearls spain

clinical surgery pearls spa

clinical surgery pearls sc

clinical surgery pearls st

clinical surgery pearls test

clinical surgery pearls therapy

clinical surgery pearls thesis

clinical surgery pearls video

clinical surgery pearls vessel

clinical surgery pearls valley

clinical surgery pearls xl

clinical surgery pearls youtube

clinical surgery pearls yellow

clinical surgery pearls yahoo finance

clinical surgery pearls yahoo

clinical surgery pearls yugioh

clinical surgery pearls zambia

clinical surgery pearls zurich

clinical surgery pearls zone

clinical surgery pearls zanzibar

can clinical surgery pearls and co

can clinical surgery pearls and beauty

can clinical surgery pearls and skin

can clinical surgery pearls benefits

can clinical surgery pearls before and after

can clinical surgery pearls before swine

can clinical surgery pearls black

can clinical surgery pearls canada

can clinical surgery pearls clinic

can clinical surgery pearls cost

can clinical surgery pearls costco

can clinical surgery pearls co

can clinical surgery pearls experience

can clinical surgery pearls free

can clinical surgery pearls for sale

can clinical surgery pearls for dogs

can clinical surgery pearls guide

can clinical surgery pearls good

can clinical surgery pearls growth

can clinical surgery pearls good or bad

can clinical surgery pearls guidelines

can clinical surgery pearls hospital

can clinical surgery pearls harvard

can clinical surgery pearls how to use

can clinical surgery pearls inc

can clinical surgery pearls in india

can clinical surgery pearls in china

can clinical surgery pearls in canada

can clinical surgery pearls journal

can clinical surgery pearls jobs

can clinical surgery pearls job description

can clinical surgery pearls job

can clinical surgery pearls journals

can clinical surgery pearls kit

can clinical surgery pearls kopen

can clinical surgery pearls ksa

can clinical surgery pearls meaning

can clinical surgery pearls mdpi

can clinical surgery pearls medical

can clinical surgery pearls md

can clinical surgery pearls mcgill

can clinical surgery pearls near me

can clinical surgery pearls ncbi

can clinical surgery pearls nursery

can clinical surgery pearls online

can clinical surgery pearls of the world

can clinical surgery pearls quotes

can clinical surgery pearls questions

can clinical surgery pearls quiz

can clinical surgery pearls quora

can clinical surgery pearls review

can clinical surgery pearls reviews

can clinical surgery pearls reddit

can clinical surgery pearls test

can clinical surgery pearls thesis

can clinical surgery pearls uk

can clinical surgery pearls uses

can clinical surgery pearls us

can clinical surgery pearls use

can clinical surgery pearls up

can clinical surgery pearls video

can clinical surgery pearls valley

can clinical surgery pearls work

can clinical surgery pearls worth

can clinical surgery pearls worth it

can clinical surgery pearls with

can clinical surgery pearls works

can clinical surgery pearls xl

can clinical surgery pearls xray

can clinical surgery pearls youtube

can clinical surgery pearls yellow

can clinical surgery pearls you

can clinical surgery pearls zurich

can clinical surgery pearls zambia

can clinical surgery pearls zanzibar

how clinical surgery pearls are

how clinical surgery pearls are made

how clinical surgery pearls benefits

how clinical surgery pearls before and after

how clinical surgery pearls canada

how clinical surgery pearls cost

how clinical surgery pearls cancer

how clinical surgery pearls come from

how clinical surgery pearls experience

how clinical surgery pearls experiment

how clinical surgery pearls free

how clinical surgery pearls foundation

how clinical surgery pearls good

how clinical surgery pearls grow

how clinical surgery pearls goodreads

how clinical surgery pearls good or bad

how clinical surgery pearls goods

how clinical surgery pearls have

how clinical surgery pearls harvard

how clinical surgery pearls inc

how clinical surgery pearls is

how clinical surgery pearls journal

how clinical surgery pearls jewellery

how clinical surgery pearls job description

how clinical surgery pearls journals

how clinical surgery pearls jewelry

how clinical surgery pearls kit

how clinical surgery pearls ksa

how clinical surgery pearls ks

how clinical surgery pearls knowledge

how clinical surgery pearls meaning

how clinical surgery pearls mean

how clinical surgery pearls means

how clinical surgery pearls made

how clinical surgery pearls make

how clinical surgery pearls near me

how clinical surgery pearls needed

how clinical surgery pearls norway

how clinical surgery pearls open

how clinical surgery pearls on

how clinical surgery pearls out

how clinical surgery pearls opened

how clinical surgery pearls of the world

how clinical surgery pearls questions

how clinical surgery pearls quiz

how clinical surgery pearls quotes

how clinical surgery pearls quora

how clinical surgery pearls quizlet

how clinical surgery pearls review

how clinical surgery pearls start

how clinical surgery pearls soap

how clinical surgery pearls started

how clinical surgery pearls test

how clinical surgery pearls thesis

how clinical surgery pearls use

how clinical surgery pearls uses

how clinical surgery pearls up

how clinical surgery pearls used

how clinical surgery pearls video

how clinical surgery pearls valley

how clinical surgery pearls work

how clinical surgery pearls works

how clinical surgery pearls worth

how clinical surgery pearls worked

how clinical surgery pearls worth it

how clinical surgery pearls xl

how clinical surgery pearls xray

how clinical surgery pearls youtube

how clinical surgery pearls you

how clinical surgery pearls yellow

how clinical surgery pearls yeast

how clinical surgery pearls zurich

how clinical surgery pearls zap

how clinical surgery pearls zundert

how to do clinical surgery pearls

which clinical surgery pearls are best

which clinical surgery pearls are made

which clinical surgery pearls are the same

which clinical surgery pearls benefits

which clinical surgery pearls before and after

which clinical surgery pearls better

which clinical surgery pearls canada

which clinical surgery pearls cost

which clinical surgery pearls come from

which clinical surgery pearls everyday

which clinical surgery pearls end

which clinical surgery pearls fit

which clinical surgery pearls foundation

which clinical surgery pearls found

which clinical surgery pearls fellow

which clinical surgery pearls fellows

which clinical surgery pearls good

which clinical surgery pearls grow

which clinical surgery pearls have

which clinical surgery pearls is best

which clinical surgery pearls is good

which clinical surgery pearls journal

which clinical surgery pearls jewellery

which clinical surgery pearls joint

which clinical surgery pearls killed

which clinical surgery pearls mean

which clinical surgery pearls made

which clinical surgery pearls means

which clinical surgery pearls make

which clinical surgery pearls needed

which clinical surgery pearls near me

which clinical surgery pearls need

which clinical surgery pearls open

which clinical surgery pearls on

which clinical surgery pearls offer

which clinical surgery pearls of the world

which clinical surgery pearls on a plane

which clinical surgery pearls quora

which clinical surgery pearls quiz

which clinical surgery pearls quality

which clinical surgery pearls questions

which clinical surgery pearls quote

which clinical surgery pearls review

which clinical surgery pearls real

which clinical surgery pearls replacement

which clinical surgery pearls reviews

which clinical surgery pearls support

which clinical surgery pearls start

which clinical surgery pearls started

which clinical surgery pearls test

which clinical surgery pearls the same

which clinical surgery pearls use

which clinical surgery pearls uses

which clinical surgery pearls valley

which clinical surgery pearls work

which clinical surgery pearls works

which clinical surgery pearls worth it

which clinical surgery pearls worth

which clinical surgery pearls xl

which clinical surgery pearls x ray

which clinical surgery pearls you

which clinical surgery pearls youtube

which clinical surgery pearls zurich

which clinical surgery pearls zap

which clinical surgery pearls zen

surgical examination

asmbs clinical pearls for emergency care of the bariatric surgery patient

clinical pearls for emergency care of the bariatric surgery patient

surgery examination

kaplan obstetrics and gynecology videos

طبيب سوداني باثولوجي

best of clinical surgery pearls

best and clinical surgery pearls

clinical surgery pearls شرح

clinical surgery pearls مترجم

clinical surgery pearls arabic

clinical surgery pearls download

clinical surgery pearls en arabe

clinical surgery pearls egybest

clinical surgery pearls greek

clinical surgery pearls gel شرح

clinical surgery pearls how to use in hindi

clinical surgery pearls hindi

clinical surgery pearls in hindi

clinical surgery pearls in شرح

clinical surgery pearls in arabic

clinical surgery pearls kurdish

clinical surgery pearls plus شرح

clinical surgery pearls review شرح

clinical surgery pearls uses in hindi

clinical surgery pearls uses in urdu

clinical surgery pearls us شرح

clinical surgery pearls white شرح

clinical surgery pearls x شرح

clinical surgery pearls xl شرح

can clinical surgery pearls arabic

can clinical surgery pearls download

can clinical surgery pearls egybest

can clinical surgery pearls greek

can clinical surgery pearls gel شرح

can clinical surgery pearls how to use in hindi

can clinical surgery pearls in hindi

can clinical surgery pearls in شرح

can clinical surgery pearls in arabic

can clinical surgery pearls like شرح

can clinical surgery pearls pdf

can clinical surgery pearls para que sirve

can clinical surgery pearls turkish

can clinical surgery pearls uses in hindi

can clinical surgery pearls uses in urdu

can clinical surgery pearls white شرح

can clinical surgery pearls x شرح

can clinical surgery pearls yapımı

how clinical surgery pearls arabic

how clinical surgery pearls download

how clinical surgery pearls does شرح

how clinical surgery pearls egybest

how clinical surgery pearls greek

how clinical surgery pearls gel شرح

how clinical surgery pearls hindi

how clinical surgery pearls how to use in hindi

how clinical surgery pearls in hindi

how clinical surgery pearls in hindi شرح

how clinical surgery pearls in شرح

how clinical surgery pearls in arabic

how clinical surgery pearls kurdish

how clinical surgery pearls like شرح

how clinical surgery pearls pdf

how clinical surgery pearls uses in hindi

how clinical surgery pearls uses in urdu

how clinical surgery pearls use in hindi

how clinical surgery pearls works شرح

how clinical surgery pearls work شرح

how clinical surgery pearls x شرح

how clinical surgery pearls zinc شرح

which clinical surgery pearls arabic

which clinical surgery pearls download

which clinical surgery pearls egybest

which clinical surgery pearls in hindi

which clinical surgery pearls pdf

which clinical surgery pearls quail

which clinical surgery pearls uses in hindi

which clinical surgery pearls use in hindi

which clinical surgery pearls uses in urdu


Comments

Search This Blog

Archive

Show more

Popular posts from this blog

TRIPASS XR تري باس

CELEPHI 200 MG, Gélule

ZENOXIA 15 MG, Comprimé

VOXCIB 200 MG, Gélule

Kana Brax Laberax

فومي كايند

بعض الادويه نجد رموز عليها مثل IR ، MR, XR, CR, SR , DS ماذا تعني هذه الرموز

NIFLURIL 700 MG, Suppositoire adulte

Antifongiques مضادات الفطريات

Popular posts from this blog

علاقة البيبي بالفراولة بالالفا فيتو بروتين

التغيرات الخمس التي تحدث للجسم عند المشي

إحصائيات سنة 2020 | تعداد سكَان دول إفريقيا تنازليا :

ما هو الليمونير للأسنان ؟

ACUPAN 20 MG, Solution injectable

CELEPHI 200 MG, Gélule

الام الظهر

VOXCIB 200 MG, Gélule

ميبستان

Popular posts from this blog

TRIPASS XR تري باس

CELEPHI 200 MG, Gélule

Popular posts from this blog

TRIPASS XR تري باس

CELEPHI 200 MG, Gélule

ZENOXIA 15 MG, Comprimé

VOXCIB 200 MG, Gélule

Kana Brax Laberax

فومي كايند

بعض الادويه نجد رموز عليها مثل IR ، MR, XR, CR, SR , DS ماذا تعني هذه الرموز

NIFLURIL 700 MG, Suppositoire adulte

Antifongiques مضادات الفطريات

Popular posts from this blog

Kana Brax Laberax

TRIPASS XR تري باس

PARANTAL 100 MG, Suppositoire بارانتال 100 مجم تحاميل

الكبد الدهني Fatty Liver

الم اسفل الظهر (الحاد) الذي يظهر بشكل مفاجئ bal-agrisi

SEDALGIC 37.5 MG / 325 MG, Comprimé pelliculé [P] سيدالجيك 37.5 مجم / 325 مجم ، قرص مغلف [P]

نمـو الدمـاغ والتطـور العقـلي لـدى الطفـل

CELEPHI 200 MG, Gélule

أخطر أنواع المخدرات فى العالم و الشرق الاوسط

Archive

Show more